IASbaba's Flagship Course: Integrated Learning Programme (ILP) - 2024  Read Details

Posts

[DAY 47] 60 DAY RAPID REVISION (RaRe) SERIES for UPSC Prelims 2024 – SCIENCE AND TECHNOLOGY, CURRENT AFFAIRS & CSAT TEST SERIES!

Archives Hello Friends The 60 Days Rapid Revision (RaRe) Series is IASbaba’s Flagship Initiative recommended by Toppers and loved by the aspirants’ community every year. It is the most comprehensive program which will help you complete the syllabus, revise and practice tests on a daily basis. The Programme on a daily basis includes Daily Prelims MCQs from Static (Monday – Saturday) Daily Static Quiz will cover all the topics of static subjects – Polity, History, Geography, Economics, Environment and Science and technology. 20 questions will be posted daily and these questions are framed from the topics mentioned in the schedule. It will ensure timely and streamlined revision of your static subjects. Daily Current Affairs MCQs (Monday – Saturday) Daily 5 Current Affairs questions, based on sources like ‘The Hindu’, ‘Indian Express’ and ‘PIB’, would be published from Monday to Saturday according to the schedule. Daily CSAT Quiz (Monday – Friday) CSAT has been an Achilles heel for many aspirants. Daily 5 CSAT Questions will be published. Note – Daily Test of 20 static questions, 10 current affairs, and 5 CSAT questions. (35 Prelims Questions) in QUIZ FORMAT will be updated on a daily basis. To Know More about 60 Days Rapid Revision (RaRe) Series – CLICK HERE   60 Day Rapid Revision (RaRe) Series Schedule – CLICK HERE  Important Note Comment your Scores in the Comment Section. This will keep you accountable, responsible and sincere in days to come. It will help us come out with the Cut-Off on a Daily Basis. Let us know if you enjoyed today’s test 🙂  You can post your comments in the given format  (1) Your Score (2) Matrix Meter (3) New Learning from the Test Time limit: 0 Test-summary 0 of 35 questions completed Questions: 1 2 3 4 5 6 7 8 9 10 11 12 13 14 15 16 17 18 19 20 21 22 23 24 25 26 27 28 29 30 31 32 33 34 35 Information The following Test is based on the syllabus of 60 Days Plan-2023 for UPSC IAS Prelims 2022. To view Solutions, follow these instructions: Click on – ‘Start Test’ button Solve Questions Click on ‘Test Summary’ button Click on ‘Finish Test’ button Now click on ‘View Questions’ button – here you will see solutions and links. You have already completed the test before. Hence you can not start it again. Test is loading... You must sign in or sign up to start the test. You have to finish following test, to start this test: Results 0 of 35 questions answered correctly Your time: Time has elapsed You have scored 0 points out of 0 points, (0) Average score     Your score     Categories Not categorized 0% Your result has been entered into leaderboard Loading Name: E-Mail: Captcha: maximum of 70 points Pos. Name Entered on Points Result Table is loading No data available 1 2 3 4 5 6 7 8 9 10 11 12 13 14 15 16 17 18 19 20 21 22 23 24 25 26 27 28 29 30 31 32 33 34 35 Answered Review Question 1 of 35 1. Question The compound used in black and white photography is? a) AgBr b) MgSO4 c) CaCO3 d) NaCl Correct Solution (a) Silver bromide (AgBr) is a soft, pale-yellow, water-insoluble salt well known (along with other silver halides) for its unusual sensitivity to light. This property has allowed silver halides to become the basis of modern photographic materials. AgBr is widely used in photographic film. Incorrect Solution (a) Silver bromide (AgBr) is a soft, pale-yellow, water-insoluble salt well known (along with other silver halides) for its unusual sensitivity to light. This property has allowed silver halides to become the basis of modern photographic materials. AgBr is widely used in photographic film. Question 2 of 35 2. Question Bisphenol A (BPA), a cause of concern, is a structural/key component in the manufacture of which of the following kinds of plastics? a) Polycarbonate b) Low-density polyethylene c) Polyvinylchloride d) Polyethylene Terephthalate Correct Solution (a) Bisphenol A (BPA) is a chemical compound primarily used in the manufacturing of various plastics. It is a colourless solid which is soluble in most common organic solvents, but has very poor solubility in water. BPA’s largest single application is as a co-monomer in the production of polycarbonates, which accounts for 65-70% of all BPA production. Incorrect Solution (a) Bisphenol A (BPA) is a chemical compound primarily used in the manufacturing of various plastics. It is a colourless solid which is soluble in most common organic solvents, but has very poor solubility in water. BPA’s largest single application is as a co-monomer in the production of polycarbonates, which accounts for 65-70% of all BPA production. Question 3 of 35 3. Question Which one among the following is a chemical process? a) Distillation of sea water b) Crystallization of impure salt c) Production of iodine from seaweeds d) Sublimation of iodine Correct Solution (c) All the given processes are physical processes except production of iodine from seaweeds. Incorrect Solution (c) All the given processes are physical processes except production of iodine from seaweeds. Question 4 of 35 4. Question With respect to Ethylene Glycol, consider the following statements: Ethylene Glycol is used as an anti-freezing agent in automobiles. Ethylene Glycol is highly poisonous in nature. Ethylene Glycol is an important precursor to polyester fibers and resins How many of the above statements are correct? a) Only one b) Only two c) All three d) None Correct Solution (c) Ethylene Glycol, also called ethane-1,2-diol, the simplest member of the glycol family of organic compounds. Ethylene glycol is a clear, sweet, slightly viscous liquid that boils at 198 °C (388.4 °F). Its most common use is as an automotive antifreeze. In addition to its use in antifreeze, ethylene glycol is used as an ingredient in hydraulic fluids, printing inks, and paint solvents. It is also used as a reagent in making polyesters, explosives, alkyd resins, and synthetic waxes. In the plastic industry, ethylene glycol is an important precursor to polyester fibers and resins. Polyethylene terephthalate, used to make plastic bottles for soft drinks, is prepared from ethylene glycol. Incorrect Solution (c) Ethylene Glycol, also called ethane-1,2-diol, the simplest member of the glycol family of organic compounds. Ethylene glycol is a clear, sweet, slightly viscous liquid that boils at 198 °C (388.4 °F). Its most common use is as an automotive antifreeze. In addition to its use in antifreeze, ethylene glycol is used as an ingredient in hydraulic fluids, printing inks, and paint solvents. It is also used as a reagent in making polyesters, explosives, alkyd resins, and synthetic waxes. In the plastic industry, ethylene glycol is an important precursor to polyester fibers and resins. Polyethylene terephthalate, used to make plastic bottles for soft drinks, is prepared from ethylene glycol. Question 5 of 35 5. Question Consider the following statements with respect to ‘Alloys’: Alloys are defined as mixtures of only two or more metals. They can be separated into their components by physical methods. Which of the statement/s given above is/are correct? a) 1 only b) 2 only c) Both 1 and 2 d) Neither 1 nor 2 Correct Solution (d) Statement 1 Statement 2 Incorrect Incorrect Alloys are mixtures of two or more metals or a metal and a non-metal. For example, brass is a mixture of approximately 30% zinc and 70% copper. They cannot be separated into their components by physical methods. But still, an alloy is considered as a mixture because it shows the properties of its constituents and can have variable composition. Incorrect Solution (d) Statement 1 Statement 2 Incorrect Incorrect Alloys are mixtures of two or more metals or a metal and a non-metal. For example, brass is a mixture of approximately 30% zinc and 70% copper. They cannot be separated into their components by physical methods. But still, an alloy is considered as a mixture because it shows the properties of its constituents and can have variable composition. Question 6 of 35 6. Question Consider the following statements with reference to properties of a colloid: A colloid is a homogenous mixture. The size of particles of a colloid are big enough to be individually seen by naked eyes. Colloids do not settle down when left undisturbed. How many of the above statements are correct? a) Only one b) Only two c) All three d) None Correct Solution (a) Statement 1 Statement 2 Statement 3 Incorrect Incorrect Correct The particles of a colloid are uniformly spread throughout the solution. Due to the relatively smaller size of particles, as compared to that of a suspension, the mixture appears to be homogeneous. But actually, a colloidal solution is a heterogeneous mixture, for example, milk. Because of the small size of colloidal particles, we cannot see them with naked eyes. But these particles can easily scatter a beam of visible light. Thus, the size of particles of a colloid is too small to be individually seen by naked eyes.   They do not settle down when left undisturbed, that is, a colloid is quite stable. They cannot be separated from the mixture by the process of filtration. But, a special technique of separation known as centrifugation, can be used to separate the colloidal particles. Incorrect Solution (a) Statement 1 Statement 2 Statement 3 Incorrect Incorrect Correct The particles of a colloid are uniformly spread throughout the solution. Due to the relatively smaller size of particles, as compared to that of a suspension, the mixture appears to be homogeneous. But actually, a colloidal solution is a heterogeneous mixture, for example, milk. Because of the small size of colloidal particles, we cannot see them with naked eyes. But these particles can easily scatter a beam of visible light. Thus, the size of particles of a colloid is too small to be individually seen by naked eyes.   They do not settle down when left undisturbed, that is, a colloid is quite stable. They cannot be separated from the mixture by the process of filtration. But, a special technique of separation known as centrifugation, can be used to separate the colloidal particles. Question 7 of 35 7. Question What do you understand by the term ‘Chromatography’? a) The study of the absorption and emission of light. b) The process of separation of components of a mixture. c) The process in which one or more substances are converted to one or more different substances. d) The study of physical interaction that occurs between electrically charged particles. Correct Solution (b) Statement 1 Statement 2 Statement 3 Statement 4 Incorrect Correct Incorrect Incorrect Spectroscopy is the study of the absorption and emission of light and other radiation by matter. It involves the splitting of light (or more precisely electromagnetic radiation) into its constituent wavelengths (a spectrum), which is done in much the same way as a prism splits light into a rainbow of colours. In chemical analysis, chromatography is a laboratory technique for the separation of a mixture into its components. The mixture is dissolved in a fluid solvent called the mobile phase, which carries it through a system on which a material called the stationary phase is fixed. Chemical reaction is a process in which one or more substances, the reactants, are converted to one or more different substances, the products. Substances are either chemical elements or compounds. A chemical reaction rearranges the constituent atoms of the reactants to create different substances as products. Electromagnetism is a branch of physics involving the study of the electromagnetic force, a type of physical interaction that occurs between electrically charged particles. The electromagnetic force is carried by electromagnetic fields composed of electric fields and magnetic fields, and it is responsible for electromagnetic radiation such as light. Incorrect Solution (b) Statement 1 Statement 2 Statement 3 Statement 4 Incorrect Correct Incorrect Incorrect Spectroscopy is the study of the absorption and emission of light and other radiation by matter. It involves the splitting of light (or more precisely electromagnetic radiation) into its constituent wavelengths (a spectrum), which is done in much the same way as a prism splits light into a rainbow of colours. In chemical analysis, chromatography is a laboratory technique for the separation of a mixture into its components. The mixture is dissolved in a fluid solvent called the mobile phase, which carries it through a system on which a material called the stationary phase is fixed. Chemical reaction is a process in which one or more substances, the reactants, are converted to one or more different substances, the products. Substances are either chemical elements or compounds. A chemical reaction rearranges the constituent atoms of the reactants to create different substances as products. Electromagnetism is a branch of physics involving the study of the electromagnetic force, a type of physical interaction that occurs between electrically charged particles. The electromagnetic force is carried by electromagnetic fields composed of electric fields and magnetic fields, and it is responsible for electromagnetic radiation such as light. Question 8 of 35 8. Question It was known by 1900 that the atom was indivisible particle but contained at least one sub-atomic particle i.e., the electron. This was identified by? a) J.J. Thomson b) E. Goldstein c) Ernest Rutherford d) John Dalton Correct Solution (a) Statement 1 Statement 2 Statement 3 Statement 4 Correct Incorrect Incorrect Incorrect J.J. Thomson (1856-1940), a British physicist, was born in Cheetham Hill, a suburb of Manchester, on 18 December 1856. He was awarded the Nobel prize in Physics in 1906 for his work on the discovery of electrons. He directed the Cavendish Laboratory at Cambridge for 35 years and 7 of his research assistants subsequently won Nobel prizes. Eugen Goldstein was a German physicist. He was an early investigator of discharge tubes, the discoverer of anode rays or canal rays, later identified as positive ions in the gas phase including the hydrogen ion. These are positively charged ions that are accelerated toward and through a perforated cathode in an evacuated tube. Ernest Rutherford was a New Zealand physicist who came to be known as the father of nuclear physics. Encyclopedia Britannica considers him to be the greatest experimentalist since Michael Faraday. He discovered the nucleus of the atom in 1911. John Dalton FRS was an English chemist, physicist and meteorologist. He is best known for introducing the atomic theory into chemistry, and for his research into colour blindness, sometimes referred to as Daltonism in his honour. Incorrect Solution (a) Statement 1 Statement 2 Statement 3 Statement 4 Correct Incorrect Incorrect Incorrect J.J. Thomson (1856-1940), a British physicist, was born in Cheetham Hill, a suburb of Manchester, on 18 December 1856. He was awarded the Nobel prize in Physics in 1906 for his work on the discovery of electrons. He directed the Cavendish Laboratory at Cambridge for 35 years and 7 of his research assistants subsequently won Nobel prizes. Eugen Goldstein was a German physicist. He was an early investigator of discharge tubes, the discoverer of anode rays or canal rays, later identified as positive ions in the gas phase including the hydrogen ion. These are positively charged ions that are accelerated toward and through a perforated cathode in an evacuated tube. Ernest Rutherford was a New Zealand physicist who came to be known as the father of nuclear physics. Encyclopedia Britannica considers him to be the greatest experimentalist since Michael Faraday. He discovered the nucleus of the atom in 1911. John Dalton FRS was an English chemist, physicist and meteorologist. He is best known for introducing the atomic theory into chemistry, and for his research into colour blindness, sometimes referred to as Daltonism in his honour. Question 9 of 35 9. Question Steel is an alloy of? a) Iron and Chromium b) Carbon and Nickel c) Iron and Carbon d) Carbon and Aluminium Correct Solution (c) Steel is an alloy made up of iron with typically a few tenths of a percent of carbon to improve its strength and fracture resistance compared to other forms of iron. Incorrect Solution (c) Steel is an alloy made up of iron with typically a few tenths of a percent of carbon to improve its strength and fracture resistance compared to other forms of iron. Question 10 of 35 10. Question With reference to the Acids and its sources in our daily life, select the correct pair: Name of acid                  Material found in Acetic acid –                  Vinegar Formic acid –                  Curd Oxalic acid –                 Spinach Select the answer: a) 1 only b) 1 and 2 only c) 2 and 3 only d) 1 and 3 only Correct Solution (d) Statement 1 Statement 2 Statement 3 Correct Incorrect Correct Acetic acid, systematically named ethanoic acid, is an acidic, colourless liquid and organic compound with the chemical formula CH₃COOH. Vinegar is at least 4% acetic acid by volume, making acetic acid the main component of vinegar apart from water and other trace elements. Formic acid is the simplest carboxylic acid, containing a single carbon. Occurs naturally in various sources including the venom of bee and ant stings, and is a useful organic synthetic reagent. Principally used as a preservative and antibacterial agent in livestock feed. Oxalic acid is an organic compound found in many plants, including leafy greens, vegetables, fruits, cocoa, nuts, seeds and spinach. In plants, it’s usually bound to minerals, forming oxalate. The terms “oxalic acid” and “oxalate” are used interchangeably in nutrition science. Incorrect Solution (d) Statement 1 Statement 2 Statement 3 Correct Incorrect Correct Acetic acid, systematically named ethanoic acid, is an acidic, colourless liquid and organic compound with the chemical formula CH₃COOH. Vinegar is at least 4% acetic acid by volume, making acetic acid the main component of vinegar apart from water and other trace elements. Formic acid is the simplest carboxylic acid, containing a single carbon. Occurs naturally in various sources including the venom of bee and ant stings, and is a useful organic synthetic reagent. Principally used as a preservative and antibacterial agent in livestock feed. Oxalic acid is an organic compound found in many plants, including leafy greens, vegetables, fruits, cocoa, nuts, seeds and spinach. In plants, it’s usually bound to minerals, forming oxalate. The terms “oxalic acid” and “oxalate” are used interchangeably in nutrition science. Question 11 of 35 11. Question Turmeric stains on the shirt turn red when washed with soap because? a) Soap water reacts with cloth of shirt to make it red. b) Soap solution is basic in nature which changes the yellow colour of turmeric into red. c) Material of shirt is exposed to liquid medium, which makes it change the colour. d) Turmeric has inherent property to turn red when it comes in contact with any solution. Correct Solution (b) Turmeric stains on the shirt turn red when washed with soap because the soap solution is basic in nature which changes the yellow colour of Turmeric into red colour. It is because turmeric contains tartaric acid, while the soap contains a sodium hydroxide base. Incorrect Solution (b) Turmeric stains on the shirt turn red when washed with soap because the soap solution is basic in nature which changes the yellow colour of Turmeric into red colour. It is because turmeric contains tartaric acid, while the soap contains a sodium hydroxide base. Question 12 of 35 12. Question With reference to classification of ‘Carbohydrates’, consider the following statements: Carbohydrates which yield a large number of monosaccharide units on hydrolysis are called oligosaccharides. Carbohydrates that yield two to ten monosaccharide units, on hydrolysis, are called polysaccharides. From the above given statements, identify the incorrect one/s: a) 1 only b) 2 only c) Both 1 and 2 d) Neither 1 nor 2 Correct Solution (c) Statement 1 Statement 2 Incorrect Incorrect A carbohydrate that cannot be hydrolysed further to give simpler unit of polyhydroxy aldehyde or ketone is called a monosaccharide. Carbohydrates that yield two to ten monosaccharide units, on hydrolysis, are called oligosaccharides. Carbohydrates which yield a large number of monosaccharide units on hydrolysis are called polysaccharides. Some common examples are starch, cellulose, glycogen, gums, etc. Polysaccharides are not sweet in taste, hence they are also called non-sugars. Incorrect Solution (c) Statement 1 Statement 2 Incorrect Incorrect A carbohydrate that cannot be hydrolysed further to give simpler unit of polyhydroxy aldehyde or ketone is called a monosaccharide. Carbohydrates that yield two to ten monosaccharide units, on hydrolysis, are called oligosaccharides. Carbohydrates which yield a large number of monosaccharide units on hydrolysis are called polysaccharides. Some common examples are starch, cellulose, glycogen, gums, etc. Polysaccharides are not sweet in taste, hence they are also called non-sugars. Question 13 of 35 13. Question Consider the following statements: Aqua regia is a mixture of nitric acid and hydrochloric acid. Aqua regia is a highly corrosive, fuming liquid. It is able to dissolve gold and platinum. How many of the above statements are correct? a) Only one b) Only two c) All three d) None Correct Solution (c) Statement 1 Statement 2 Statement 3 Correct Correct Correct Aqua regia, (Latin for ‘royal water’) is a freshly prepared mixture of concentrated hydrochloric acid and concentrated nitric acid in the ratio of 3:1. Aqua regia is a highly corrosive, fuming liquid. It can dissolve gold, even though neither of these acids can do so alone. It is one of the few reagents that is able to dissolve gold and platinum. Aqua regia is a mixture of nitric acid and hydrochloric acid, optimally in a molar ratio of 1:3. Aqua regia is a fuming liquid.   Hydrogen gas is not evolved when a metal reacts with nitric acid. It is because HNO3 is a strong oxidising agent. It oxidises the H2 produced to water and itself gets reduced to any of the nitrogen oxides (N2O, NO, NO2). But magnesium (Mg) and manganese (Mn) react with very dilute HNO3 to evolve H2 gas. Incorrect Solution (c) Statement 1 Statement 2 Statement 3 Correct Correct Correct Aqua regia, (Latin for ‘royal water’) is a freshly prepared mixture of concentrated hydrochloric acid and concentrated nitric acid in the ratio of 3:1. Aqua regia is a highly corrosive, fuming liquid. It can dissolve gold, even though neither of these acids can do so alone. It is one of the few reagents that is able to dissolve gold and platinum. Aqua regia is a mixture of nitric acid and hydrochloric acid, optimally in a molar ratio of 1:3. Aqua regia is a fuming liquid.   Hydrogen gas is not evolved when a metal reacts with nitric acid. It is because HNO3 is a strong oxidising agent. It oxidises the H2 produced to water and itself gets reduced to any of the nitrogen oxides (N2O, NO, NO2). But magnesium (Mg) and manganese (Mn) react with very dilute HNO3 to evolve H2 gas. Question 14 of 35 14. Question Consider the following statements: The chemical name of the compound ‘Baking Soda’ is sodium chloride. Carbon dioxide is produced when baking powder is heated or mixed in water. Which of the given statement/s is/are correct? a) 1 only b) 2 only c) Both 1 and 2 d) Neither 1 nor 2 Correct Solution (b) Statement 1 Statement 2 Incorrect Correct The baking soda is commonly used in the kitchen for making tasty crispy pakoras, etc. Sometimes it is added for faster cooking. The chemical name of the compound is sodium hydrogencarbonate (NaHCO3). It is produced using sodium chloride as one of the raw materials. Sodium Chloride (NaCl) is an ingredient in common salt. For making baking powder, which is a mixture of baking soda (sodium hydrogencarbonate) and a mild edible acid such as tartaric acid. When baking powder is heated or mixed in water, the following reaction takes place – NaHCO3 + H+ = CO2 + H2O + Sodium salt of acid Carbon dioxide produced during the reaction can cause bread or cake to rise making them soft and spongy. Incorrect Solution (b) Statement 1 Statement 2 Incorrect Correct The baking soda is commonly used in the kitchen for making tasty crispy pakoras, etc. Sometimes it is added for faster cooking. The chemical name of the compound is sodium hydrogencarbonate (NaHCO3). It is produced using sodium chloride as one of the raw materials. Sodium Chloride (NaCl) is an ingredient in common salt. For making baking powder, which is a mixture of baking soda (sodium hydrogencarbonate) and a mild edible acid such as tartaric acid. When baking powder is heated or mixed in water, the following reaction takes place – NaHCO3 + H+ = CO2 + H2O + Sodium salt of acid Carbon dioxide produced during the reaction can cause bread or cake to rise making them soft and spongy. Question 15 of 35 15. Question The p in pH stands for ‘potenz’ in German, meaning power. In this regard, consider the following statements with reference to ‘pH’: The pH of human stomach is acidic. Tooth decay starts when the pH of the mouth is lower than 5.5. Human body works within the pH range of 7.0 to 7.8. How many of the above statements are correct? a) Only one b) Only two c) All three d) None Correct Solution (c) Statement 1 Statement 2 Statement 3 Correct Correct Correct The pH of our stomach varies, but its natural state is between 1.5 and 3.5. This level rises when food enters the stomach; it can reach up to six, but it lowers again throughout digestion as stomach acid is secreted. It is very interesting to note that our stomach produces hydrochloric acid. It helps in the digestion of food without harming the stomach. Tooth decay starts when the pH of the mouth is lower than 5.5. Tooth enamel, made up of calcium hydroxyapatite (a crystalline form of calcium phosphate) is the hardest substance in the body. It does not dissolve in water, but is corroded when the pH in the mouth is below 5.5. Bacteria present in the mouth produce acids by degradation of sugar and food particles remaining in the mouth after eating. Human body works within the pH range of 7.0 to 7.8. Living organisms can survive only in a narrow range of pH change. When pH of rain water is less than 5.6, it is called acid rain. When acid rain flows into the rivers, it lowers the pH of the river water. The survival of aquatic life in such rivers becomes difficult. Incorrect Solution (c) Statement 1 Statement 2 Statement 3 Correct Correct Correct The pH of our stomach varies, but its natural state is between 1.5 and 3.5. This level rises when food enters the stomach; it can reach up to six, but it lowers again throughout digestion as stomach acid is secreted. It is very interesting to note that our stomach produces hydrochloric acid. It helps in the digestion of food without harming the stomach. Tooth decay starts when the pH of the mouth is lower than 5.5. Tooth enamel, made up of calcium hydroxyapatite (a crystalline form of calcium phosphate) is the hardest substance in the body. It does not dissolve in water, but is corroded when the pH in the mouth is below 5.5. Bacteria present in the mouth produce acids by degradation of sugar and food particles remaining in the mouth after eating. Human body works within the pH range of 7.0 to 7.8. Living organisms can survive only in a narrow range of pH change. When pH of rain water is less than 5.6, it is called acid rain. When acid rain flows into the rivers, it lowers the pH of the river water. The survival of aquatic life in such rivers becomes difficult. Question 16 of 35 16. Question Consider the following statements: Diamond and Graphite are crystalline forms of carbon. Crystallization is a technique used to purify solid compounds. Crystallization is based on the principles of solubility How many of the above statements are correct? a) Only one b) Only two c) All three d) None Correct Solution (c) Statement 1 Statement 2 Statement 3 Correct Correct Correct The crystalline forms of carbon are diamond, graphite and fullerene. Crystallization is the process by which a solid form, where the atoms or molecules are highly organized into a structure known as a crystal.   Crystallization is a technique which chemists use to purify solid compounds. Crystallization is based on the principles of solubility: compounds (solutes) tend to be more soluble in hot liquids (solvents) than they are in cold liquids.   If a saturated hot solution is allowed to cool, the solute is no longer soluble in the solvent and forms crystals of pure compound.   Impurities are excluded from the growing crystals and the pure solid crystals can be separated from the dissolved impurities by filtration.   Incorrect Solution (c) Statement 1 Statement 2 Statement 3 Correct Correct Correct The crystalline forms of carbon are diamond, graphite and fullerene. Crystallization is the process by which a solid form, where the atoms or molecules are highly organized into a structure known as a crystal.   Crystallization is a technique which chemists use to purify solid compounds. Crystallization is based on the principles of solubility: compounds (solutes) tend to be more soluble in hot liquids (solvents) than they are in cold liquids.   If a saturated hot solution is allowed to cool, the solute is no longer soluble in the solvent and forms crystals of pure compound.   Impurities are excluded from the growing crystals and the pure solid crystals can be separated from the dissolved impurities by filtration.   Question 17 of 35 17. Question Which one of the following is not a semiconductor? a) Germanium b) Silicon c) Quartz d) Gallium Arsenide Correct Solution (c) A semiconductor material has an electrical conductivity value falling between that of a conductor, such as metallic copper, and an insulator, such as glass. Its resistivity falls as its temperature rises; metals behave in the opposite way. Some examples of semiconductors are silicon, germanium and gallium arsenide. After silicon, gallium arsenide is the second-most common semiconductor and is used in laser diodes, solar cells, microwave-frequency integrated circuits, and others. Silicon is a critical element for fabricating most electronic circuits. Incorrect Solution (c) A semiconductor material has an electrical conductivity value falling between that of a conductor, such as metallic copper, and an insulator, such as glass. Its resistivity falls as its temperature rises; metals behave in the opposite way. Some examples of semiconductors are silicon, germanium and gallium arsenide. After silicon, gallium arsenide is the second-most common semiconductor and is used in laser diodes, solar cells, microwave-frequency integrated circuits, and others. Silicon is a critical element for fabricating most electronic circuits. Question 18 of 35 18. Question Which one of the following polymers is widely used for making bullet proof material? a) Polycarbonate b) Polyvinyl Chloride c) Polyethylene d) Polyamides Correct Solution (a) Polycarbonates (PC) are a group of thermoplastic polymers containing carbonate groups in their chemical structures. Polycarbonates used in engineering are strong, tough materials, and some grades are optically transparent. Much of the bullet-proof and shatter-proof glass that you see and experience in real life is actually made of bullet-resistant polycarbonate sheets. Incorrect Solution (a) Polycarbonates (PC) are a group of thermoplastic polymers containing carbonate groups in their chemical structures. Polycarbonates used in engineering are strong, tough materials, and some grades are optically transparent. Much of the bullet-proof and shatter-proof glass that you see and experience in real life is actually made of bullet-resistant polycarbonate sheets. Question 19 of 35 19. Question Magnetite is a mineral whose primary component is an: a) Iron Oxide b) Copper Oxide c) Aluminium Oxide d) Magnesium Oxide Correct Solution (a) Magnetite is a mineral whose primary component is an iron oxide that contains equal amounts of iron (II) and iron (III). Its empirical formula is Fe3O4, and it is often expressed as iron (II, III) oxide. Magnetite is found in igneous, metamorphic, and sedimentary rocks. As its name implies, it is magnetic; it and other inherently magnetic iron-containing minerals are described as being ferrimagnetic. Incorrect Solution (a) Magnetite is a mineral whose primary component is an iron oxide that contains equal amounts of iron (II) and iron (III). Its empirical formula is Fe3O4, and it is often expressed as iron (II, III) oxide. Magnetite is found in igneous, metamorphic, and sedimentary rocks. As its name implies, it is magnetic; it and other inherently magnetic iron-containing minerals are described as being ferrimagnetic. Question 20 of 35 20. Question Who of the following was the founder of the ‘Madras school’ of conformational analysis of biopolymers? a) Ramakrishnan Nagaraj b) G.N. Ramachandran c) Salim Ali d) CV Raman Correct Solution (b) G.N. Ramachandran, an outstanding figure in the field of protein structure, was the founder of the ‘Madras school’ of conformational analysis of biopolymers. His discovery of the triple helical structure of collagen in 1955 and his analysis of the allowed conformations of proteins through the use of the ‘Ramachandran plot’ ranks among the most outstanding contributions in structural biology. Ramakrishnan Nagaraj (born 1953) is an Indian biochemist, molecular biologist and the leader of a team of scientists working in the field of peptide biochemistry at Centre for Cellular and Molecular Biology (CCMB). He is known for his studies on hemolytic and antibacterial properties in synthetic analogs of bacterial toxins. Sálim Moizuddin Abdul Ali (12 November 1896 – 20 June 1987) was an Indian ornithologist and naturalist. Sometimes referred to as the “Birdman of India”, Salim Ali was the first Indian to conduct systematic bird surveys across India and wrote several bird books that popularized ornithology in India. He became a key figure behind the Bombay Natural History Society. Chandrasekhara Venkata Raman won the Nobel Prize for Physics in 1930 for his pioneering work on scattering of light. Born in Tiruchirapalli on November 7, 1888, he was the first Asian and first non-White to receive any Nobel Prize in the sciences. Incorrect Solution (b) G.N. Ramachandran, an outstanding figure in the field of protein structure, was the founder of the ‘Madras school’ of conformational analysis of biopolymers. His discovery of the triple helical structure of collagen in 1955 and his analysis of the allowed conformations of proteins through the use of the ‘Ramachandran plot’ ranks among the most outstanding contributions in structural biology. Ramakrishnan Nagaraj (born 1953) is an Indian biochemist, molecular biologist and the leader of a team of scientists working in the field of peptide biochemistry at Centre for Cellular and Molecular Biology (CCMB). He is known for his studies on hemolytic and antibacterial properties in synthetic analogs of bacterial toxins. Sálim Moizuddin Abdul Ali (12 November 1896 – 20 June 1987) was an Indian ornithologist and naturalist. Sometimes referred to as the “Birdman of India”, Salim Ali was the first Indian to conduct systematic bird surveys across India and wrote several bird books that popularized ornithology in India. He became a key figure behind the Bombay Natural History Society. Chandrasekhara Venkata Raman won the Nobel Prize for Physics in 1930 for his pioneering work on scattering of light. Born in Tiruchirapalli on November 7, 1888, he was the first Asian and first non-White to receive any Nobel Prize in the sciences. Question 21 of 35 21. Question “Sagar Aankalan Guidelines” recently in the news is? a) These guidelines aim to promote deep-sea exploration and resource extraction. b) These guidelines focus on developing sustainable fishing practices and conserving marine resources. c) These guidelines aim to map and benchmark the logistics performance of Indian ports. d) These guidelines aim for information sharing and cooperation between maritime agencies. Correct Solution (c) The Sagar Aankalan Guidelines were launched by the Indian government to establish a comprehensive framework for assessing and improving the performance of Indian ports. Hence option c is correct. It focuses on the National Benchmarking of Indian Ports’ Performance to enhance the efficiency and effectiveness of seaports across India. It would apply to all Indian seaports. It will transform the performance assessment of Indian ports. It will help in fulfilling “Maritime Amrit Kaal Vision 2047”. It will help in port development, modernization, green hydrogen and ammonia, port-led development, the cruise sector, business and commerce, shipbuilding, and knowledge sharing. Incorrect Solution (c) The Sagar Aankalan Guidelines were launched by the Indian government to establish a comprehensive framework for assessing and improving the performance of Indian ports. Hence option c is correct. It focuses on the National Benchmarking of Indian Ports’ Performance to enhance the efficiency and effectiveness of seaports across India. It would apply to all Indian seaports. It will transform the performance assessment of Indian ports. It will help in fulfilling “Maritime Amrit Kaal Vision 2047”. It will help in port development, modernization, green hydrogen and ammonia, port-led development, the cruise sector, business and commerce, shipbuilding, and knowledge sharing. Question 22 of 35 22. Question Consider the following statements regarding the Central Electricity Regulatory Commission (CERC): It is a statutory body established under the provisions of the Electricity Regulatory Commissions Act, of 1998. The Chairperson of the Central Electricity Authority is an ex-officio member of the CERC. It regulates the tariff of generating companies owned or controlled by the central government. It regulates the inter-state transmission of electricity and determines tariffs for inter-state transmission of electricity. How many of the above statements are correct? a) Only one b) Only two c) Only three d) All four Correct Solution (d) The Central Electricity Regulatory Commission (CERC) is a statutory body established under the provisions of the Electricity Regulatory Commissions Act, of 1998. Hence statement 1 is correct. It is the central commission for the purposes of the Electricity Act, of 2003, which has repealed the ERC Act, of 1998. The Commission consists of a Chairperson and four other members including the Chairperson, Central Electricity Authority, who is an ex-officio Member of the Commission. The Chairperson of the Central Electricity Authority is an ex-officio member of the CERC. Hence statement 2 is correct. It intends to promote competition, efficiency, and economy in bulk power markets, improve the quality of supply, promote investments, and advise the government on the removal of institutional barriers to bridge the demand-supply gap and thus foster the interests of consumers. It regulates the tariff of generating companies owned or controlled by the central government. Hence statement 3 is correct. It regulates the inter-state transmission of electricity and determines tariffs for inter-state transmission of electricity. Hence statement 4 is correct. It regulates the tariff of generating companies other than those owned or controlled by the central government specified in clause (a) if such generating companies enter into or otherwise have a composite scheme for the generation and sale of electricity in more than one state. It issues licences to persons to function as transmission licensees and electricity traders with respect to their interstate operations. Incorrect Solution (d) The Central Electricity Regulatory Commission (CERC) is a statutory body established under the provisions of the Electricity Regulatory Commissions Act, of 1998. Hence statement 1 is correct. It is the central commission for the purposes of the Electricity Act, of 2003, which has repealed the ERC Act, of 1998. The Commission consists of a Chairperson and four other members including the Chairperson, Central Electricity Authority, who is an ex-officio Member of the Commission. The Chairperson of the Central Electricity Authority is an ex-officio member of the CERC. Hence statement 2 is correct. It intends to promote competition, efficiency, and economy in bulk power markets, improve the quality of supply, promote investments, and advise the government on the removal of institutional barriers to bridge the demand-supply gap and thus foster the interests of consumers. It regulates the tariff of generating companies owned or controlled by the central government. Hence statement 3 is correct. It regulates the inter-state transmission of electricity and determines tariffs for inter-state transmission of electricity. Hence statement 4 is correct. It regulates the tariff of generating companies other than those owned or controlled by the central government specified in clause (a) if such generating companies enter into or otherwise have a composite scheme for the generation and sale of electricity in more than one state. It issues licences to persons to function as transmission licensees and electricity traders with respect to their interstate operations. Question 23 of 35 23. Question Consider the following statements: The 14th Conference of the Parties to the Convention on the United Nations Convention on Biological Diversity (UNCBD) adopted the Single Species Action Plan for the conservation of the Hawksbill Turtle. The International Single Species Action Plan is developed under the Agreement on the Conservation of African-Eurasian Migratory Waterbirds (AEWA). AEWA is an intergovernmental treaty dedicated to the conservation of migratory waterbirds and their habitats across Africa, Europe, the Middle East, Central Asia, Greenland, and the Canadian Archipelago. How many of the above statements are correct? a) Only one b) Only two c) All three d) None Correct Solution (a) The 14th Conference of the Parties to the Convention on the Conservation of Migratory Species of Wild Animals (CMS COP14) adopted the Single Species Action Plan for the conservation of the Hawksbill Turtle. Hence statement 1 is incorrect. The International Single Species Action Plan is developed under the Agreement on the Conservation of African-Eurasian Migratory Waterbirds (AEWA). Hence statement 2 is correct. It is to implement coordinated measures to restore migratory waterbird species to favourable conservation status. AEWA is an intergovernmental treaty dedicated to the conservation of migratory waterbirds and their habitats across Africa, Europe, the Middle East, Central Asia, Greenland, and the Canadian Archipelago. Hence statement 3 is correct. It was developed under the framework of the Convention on Migratory Species (CMS) and administered by the United Nations Environment Programme (UNEP). It brings together countries and the wider international conservation community to establish coordinated conservation and management of migratory waterbirds throughout their entire migratory range. Incorrect Solution (a) The 14th Conference of the Parties to the Convention on the Conservation of Migratory Species of Wild Animals (CMS COP14) adopted the Single Species Action Plan for the conservation of the Hawksbill Turtle. Hence statement 1 is incorrect. The International Single Species Action Plan is developed under the Agreement on the Conservation of African-Eurasian Migratory Waterbirds (AEWA). Hence statement 2 is correct. It is to implement coordinated measures to restore migratory waterbird species to favourable conservation status. AEWA is an intergovernmental treaty dedicated to the conservation of migratory waterbirds and their habitats across Africa, Europe, the Middle East, Central Asia, Greenland, and the Canadian Archipelago. Hence statement 3 is correct. It was developed under the framework of the Convention on Migratory Species (CMS) and administered by the United Nations Environment Programme (UNEP). It brings together countries and the wider international conservation community to establish coordinated conservation and management of migratory waterbirds throughout their entire migratory range. Question 24 of 35 24. Question Consider the following statements regarding Papua New Guinea: It is an island country that lies in the south-western Atlantic. It has Indonesia to the west, Australia to the south, and the Solomon Islands to the southeast. It is the world’s most linguistically diverse country, with more than 800 languages. It is a constitutional monarchy and a member of the Commonwealth. How many of the above statements are correct? a) Only one b) Only two c) Only three d) All four Correct Solution (c) Papua New Guinea is an island country that lies in the south-western Pacific. Hence statement 1 is incorrect. Its capital is Port Moresby. It has Indonesia to the west, Australia to the south, and the Solomon Islands to the southeast. Hence statement 2 is correct. It is mainly mountainous but has low-lying plains in southern New Guinea. The islands that constitute Papua New Guinea were settled over a period of 40,000 years by a mixture of peoples who are generally referred to as Melanesians. It is the world’s most linguistically diverse country, with more than 800 languages. Hence statement 3 is correct. English is the main language of government and commerce. In most everyday contexts, the most widely spoken language is Tok Pisin. The majority of Papua New Guinea’s people are at least nominally Christian. It is a constitutional monarchy and a member of the Commonwealth. Hence statement 4 is correct. Papua New Guinea became self-governing on December 1, 1973, and achieved independence on September 16, 1975. Incorrect Solution (c) Papua New Guinea is an island country that lies in the south-western Pacific. Hence statement 1 is incorrect. Its capital is Port Moresby. It has Indonesia to the west, Australia to the south, and the Solomon Islands to the southeast. Hence statement 2 is correct. It is mainly mountainous but has low-lying plains in southern New Guinea. The islands that constitute Papua New Guinea were settled over a period of 40,000 years by a mixture of peoples who are generally referred to as Melanesians. It is the world’s most linguistically diverse country, with more than 800 languages. Hence statement 3 is correct. English is the main language of government and commerce. In most everyday contexts, the most widely spoken language is Tok Pisin. The majority of Papua New Guinea’s people are at least nominally Christian. It is a constitutional monarchy and a member of the Commonwealth. Hence statement 4 is correct. Papua New Guinea became self-governing on December 1, 1973, and achieved independence on September 16, 1975. Question 25 of 35 25. Question Consider the following statements regarding the Director General of Civil Aviation (DGCA): Its primary function is to ensure the safety of passengers and crew members on all flights operating in India. It is responsible for the regulation of air transport services to/from/within India and for the enforcement of civil air regulations, air safety, and airworthiness standards. It investigates any incidents or accidents that occur within the Indian airspace and takes appropriate action to prevent similar incidents from happening in the future. How many of the above statements are correct? a) Only one b) Only two c) All three d) None Correct Solution (c) The Director General of Civil Aviation (DGCA) is the regulatory body in the field of civil aviation primarily dealing with safety issues. It is an attached office of the Ministry of Civil Aviation. Its primary function is to ensure the safety of passengers and crew members on all flights operating in India. Hence statement 1 is correct. In addition to safety, the DGCA also plays a crucial role in the growth and development of the Indian aviation industry. The organization works closely with airlines and airport operators to promote the industry’s growth and improve the overall travel experience for passengers. It also coordinates all regulatory functions with the International Civil Aviation Organisation. It is responsible for the regulation of air transport services to/from/within India and for the enforcement of civil air regulations, air safety, and airworthiness standards. Hence statement 2 is correct. It plays a crucial role in developing new airports and modernizing existing facilities to ensure that they can meet the growing demands of the aviation industry. It investigates any incidents or accidents that occur within the Indian airspace and takes appropriate action to prevent similar incidents from happening in the future. Hence statement 3 is correct.   Incorrect Solution (c) The Director General of Civil Aviation (DGCA) is the regulatory body in the field of civil aviation primarily dealing with safety issues. It is an attached office of the Ministry of Civil Aviation. Its primary function is to ensure the safety of passengers and crew members on all flights operating in India. Hence statement 1 is correct. In addition to safety, the DGCA also plays a crucial role in the growth and development of the Indian aviation industry. The organization works closely with airlines and airport operators to promote the industry’s growth and improve the overall travel experience for passengers. It also coordinates all regulatory functions with the International Civil Aviation Organisation. It is responsible for the regulation of air transport services to/from/within India and for the enforcement of civil air regulations, air safety, and airworthiness standards. Hence statement 2 is correct. It plays a crucial role in developing new airports and modernizing existing facilities to ensure that they can meet the growing demands of the aviation industry. It investigates any incidents or accidents that occur within the Indian airspace and takes appropriate action to prevent similar incidents from happening in the future. Hence statement 3 is correct.   Question 26 of 35 26. Question Consider the following statements regarding the Melghat Tiger Reserve: It is a critical tiger habitat located in Vindhya Hill in northern Madhya Pradesh. It is a catchment area for the Khandu, Khapra, Sipna, Gadga, and Dolar, all of which are tributaries of the River Tapti. The Korkus are the largest tribal community in Melghat. How many of the above statements are correct? a) Only one b) Only two c) All three d) None Correct Solution (b) The Melghat Tiger Reserve is located in the Amaravati district of Maharashtra. Hence statement 1 is incorrect. It is located on the southern offshoot of the Satpura Hill Range in Central India, called Gavilgarh Hill. It was established as a wildlife sanctuary in 1967 and was declared a tiger reserve in 1974. It was the first tiger reserve in Maharashtra. It was among the first nine tiger reserves notified in 1973-74 under Project Tiger. The name ‘Melghat’ means the confluence of various ‘ghats’ or valleys, as is typical of the landscape of this Tiger Reserve. It is a catchment area for the Khandu, Khapra, Sipna, Gadga, and Dolar, all of which are tributaries of the River Tapti. Hence statement 2 is correct. The forest is tropical dry deciduous, and dominated by teak. The Tapti River and the Gawilgadh ridge of the Satpura Range form the boundaries of the reserve. The Korkus are the largest tribal community in Melghat. Hence statement 3 is correct. Other communities include the Gawli community, the Gond tribe, and several other smaller tribal communities. Incorrect Solution (b) The Melghat Tiger Reserve is located in the Amaravati district of Maharashtra. Hence statement 1 is incorrect. It is located on the southern offshoot of the Satpura Hill Range in Central India, called Gavilgarh Hill. It was established as a wildlife sanctuary in 1967 and was declared a tiger reserve in 1974. It was the first tiger reserve in Maharashtra. It was among the first nine tiger reserves notified in 1973-74 under Project Tiger. The name ‘Melghat’ means the confluence of various ‘ghats’ or valleys, as is typical of the landscape of this Tiger Reserve. It is a catchment area for the Khandu, Khapra, Sipna, Gadga, and Dolar, all of which are tributaries of the River Tapti. Hence statement 2 is correct. The forest is tropical dry deciduous, and dominated by teak. The Tapti River and the Gawilgadh ridge of the Satpura Range form the boundaries of the reserve. The Korkus are the largest tribal community in Melghat. Hence statement 3 is correct. Other communities include the Gawli community, the Gond tribe, and several other smaller tribal communities. Question 27 of 35 27. Question Consider the following statements: National Electronic Funds Transfer (NEFT) is suitable for both small and large-value transactions. Real Time Gross Settlement (RTGS) system does not operate on weekends. Under Immediate Payment Service (IMPS), a minimum transaction amount is ₹2 lakhs. How many of the above statements are correct? a) Only one b) Only two c) All three d) None Correct Solution (a) National Electronic Funds Transfer (NEFT) is suitable for both small and large-value transactions. It allows individuals to transfer any amount to the recipient’s account on a one-on-one transfer basis without a maximum limit for funds that can be transferred in a single day. Hence statement 1 is correct. The Real Time Gross Settlement (RTGS) system is available 24 hours a day, 7 days a week, including weekends and bank holidays. Hence statement 2 is incorrect. Immediate Payment Service allows for both peer-to-peer transfers and merchant payments. IMPS is available 24/7 throughout the year, including bank holidays, and enables real-time instant inter-bank funds to transfer with no upper limit for transactions. The upper limit usually varies from bank to bank. There is also no lower limit or minimum transaction amount specified for IMPS. Hence statement 3 is incorrect. Incorrect Solution (a) National Electronic Funds Transfer (NEFT) is suitable for both small and large-value transactions. It allows individuals to transfer any amount to the recipient’s account on a one-on-one transfer basis without a maximum limit for funds that can be transferred in a single day. Hence statement 1 is correct. The Real Time Gross Settlement (RTGS) system is available 24 hours a day, 7 days a week, including weekends and bank holidays. Hence statement 2 is incorrect. Immediate Payment Service allows for both peer-to-peer transfers and merchant payments. IMPS is available 24/7 throughout the year, including bank holidays, and enables real-time instant inter-bank funds to transfer with no upper limit for transactions. The upper limit usually varies from bank to bank. There is also no lower limit or minimum transaction amount specified for IMPS. Hence statement 3 is incorrect. Question 28 of 35 28. Question Consider the following statements: Cryptocurrencies in India fall under the Virtual Digital Assets (VDAs) category and are not subject to taxation. Bitcoin is an open-source digital currency, that facilitates instant payments without central authority issuance. Choose the correct code: a) 1 only b) 2 only c) Both 1 and 2 d) Neither 1 nor 2 Correct Solution (b) Cryptocurrencies in India fall under the virtual digital assets (VDAs) category and are subject to taxation. Hence statement 1 is incorrect. The profits generated from cryptocurrency trading are taxed at a rate of 30%, with an additional 4% cess (Union budget 2022-23). In 2022, the RBI launched its own Central Bank Digital Currency (CBDC) known as the e-Rupee which is based on blockchain technology. Bitcoin is an open-source digital currency, that facilitates instant payments without central authority issuance. Hence statement 2 is correct. Note: A bitcoin halving is an event where the reward for mining new blocks is halved, meaning miners receive 50% fewer bitcoins for verifying transactions. Bitcoin halving is scheduled to occur once every 210,000 blocks – roughly every four years – until the maximum supply of 21 million bitcoins has been generated by the network.   Incorrect Solution (b) Cryptocurrencies in India fall under the virtual digital assets (VDAs) category and are subject to taxation. Hence statement 1 is incorrect. The profits generated from cryptocurrency trading are taxed at a rate of 30%, with an additional 4% cess (Union budget 2022-23). In 2022, the RBI launched its own Central Bank Digital Currency (CBDC) known as the e-Rupee which is based on blockchain technology. Bitcoin is an open-source digital currency, that facilitates instant payments without central authority issuance. Hence statement 2 is correct. Note: A bitcoin halving is an event where the reward for mining new blocks is halved, meaning miners receive 50% fewer bitcoins for verifying transactions. Bitcoin halving is scheduled to occur once every 210,000 blocks – roughly every four years – until the maximum supply of 21 million bitcoins has been generated by the network.   Question 29 of 35 29. Question Consider the following statements about Google DeepMind’s Genie: It is the first generative interactive environment that has been trained in an unsupervised manner from unlabelled internet videos. It can generate an endless variety of playable (action-controllable) worlds from synthetic images, photographs, and even sketches. Choose the correct code: a) 1 only b) 2 only c) Both 1 and 2 d) Neither 1 nor 2 Correct Solution (c) Google DeepMind is a British-American AI research laboratory that is a subsidiary of Google. DeepMind is based in London and has research centres in Canada, France, Germany, and the US. It has introduced Genie AI (Artificial Intelligence), a new model that can generate interactive video games from just a text or image prompt. It is the first generative interactive environment that has been trained in an unsupervised manner from unlabelled internet videos. Hence statement 1 is correct. Generative Interactive Environments (Genie) is a foundation world model that is trained on videos sourced from the Internet. The model can “generate an endless variety of playable (action-controllable) worlds from synthetic images, photographs, and even sketches”. Hence statement 2 is correct. Incorrect Solution (c) Google DeepMind is a British-American AI research laboratory that is a subsidiary of Google. DeepMind is based in London and has research centres in Canada, France, Germany, and the US. It has introduced Genie AI (Artificial Intelligence), a new model that can generate interactive video games from just a text or image prompt. It is the first generative interactive environment that has been trained in an unsupervised manner from unlabelled internet videos. Hence statement 1 is correct. Generative Interactive Environments (Genie) is a foundation world model that is trained on videos sourced from the Internet. The model can “generate an endless variety of playable (action-controllable) worlds from synthetic images, photographs, and even sketches”. Hence statement 2 is correct. Question 30 of 35 30. Question Consider the following statements about Uyghurs: They are a predominantly Muslim minority Turkic ethnic group, whose origins can be traced to Central and East Asia. Currently, the largest population of the Uighur ethnic community lives in the Xinjiang region of Turkey. Choose the correct code: a) 1 only b) 2 only c) Both 1 and 2 d) Neither 1 nor 2 Correct Solution (a) Uyghurs are a predominantly Muslim minority Turkic ethnic group, whose origins can be traced to Central and East Asia. Hence statement 1 is correct. The Uighurs speak their own language, similar to Turkish, and see themselves as culturally and ethnically close to Central Asian nations. The Uighurs are considered to be one of the 55 officially recognized ethnic minority communities in China. However, China recognises the community only as a regional minority and rejects that they are an indigenous group. Currently, the largest population of the Uighur ethnic community lives in the Xinjiang region of China. Hence statement 2 is incorrect. Incorrect Solution (a) Uyghurs are a predominantly Muslim minority Turkic ethnic group, whose origins can be traced to Central and East Asia. Hence statement 1 is correct. The Uighurs speak their own language, similar to Turkish, and see themselves as culturally and ethnically close to Central Asian nations. The Uighurs are considered to be one of the 55 officially recognized ethnic minority communities in China. However, China recognises the community only as a regional minority and rejects that they are an indigenous group. Currently, the largest population of the Uighur ethnic community lives in the Xinjiang region of China. Hence statement 2 is incorrect. Question 31 of 35 31. Question At 5’O clock, clock ticks 5 times. The time between first and last tick was 36sec. How much time it takes at 11’O clock? a) 60 seconds b) 90 seconds c) 120 seconds d) 30 seconds Correct Solution (b) At 5’o clock it ticks 5 times so there are total 4-time intervals. Total time taken = 36 seconds Hence, the time taken between any two consecutive ticks = 36/4 seconds = 9 seconds Similarly, clock will tick 11 times at 11 o’ clock. At 11o’clock there will be 10 intervals. Hence total time needed at 11 o’clock = 10 × 9 = 90 seconds. Incorrect Solution (b) At 5’o clock it ticks 5 times so there are total 4-time intervals. Total time taken = 36 seconds Hence, the time taken between any two consecutive ticks = 36/4 seconds = 9 seconds Similarly, clock will tick 11 times at 11 o’ clock. At 11o’clock there will be 10 intervals. Hence total time needed at 11 o’clock = 10 × 9 = 90 seconds. Question 32 of 35 32. Question Out of the four annual examinations, each with a total of 200 marks, a student secured average marks of 45%, 50% and 55% in the first, second and third annual examinations. To have an overall average of 60%, how many marks does the student need to secure in the fourth annual examination? a) 160 b) 170 c) 180 d) 190 Correct Solution (c)              Let the average marks in the third Annual examination be x. Total marks = (Marks in first + second + third + forth) Annual examination 4(60)( 200/100) = (45/100) (200) + ( 50/100) (200) + ( 55/100) (200) + ( x/100) (200) 4(60) = 45 + 50 +55 + x x = 90 So, the student must score 90% in the fourth annual examination to secure 60% overall average. ∴ Average marks in the fourth annual examination (90/100) × 200 = 180 marks. Incorrect Solution (c)              Let the average marks in the third Annual examination be x. Total marks = (Marks in first + second + third + forth) Annual examination 4(60)( 200/100) = (45/100) (200) + ( 50/100) (200) + ( 55/100) (200) + ( x/100) (200) 4(60) = 45 + 50 +55 + x x = 90 So, the student must score 90% in the fourth annual examination to secure 60% overall average. ∴ Average marks in the fourth annual examination (90/100) × 200 = 180 marks. Question 33 of 35 33. Question The sum of 3 numbers is 85. If third number be one fifth of the first and the first number be twice the second, find the second number. a) 20 b) 25 c) 10 d) 15 Correct Solution (b) From the details given in the question, the Ratio of the numbers = 1: 1/2: 1/5 = 10: 5: 2 Second number = 5/17 × 85 = 25. Incorrect Solution (b) From the details given in the question, the Ratio of the numbers = 1: 1/2: 1/5 = 10: 5: 2 Second number = 5/17 × 85 = 25. Question 34 of 35 34. Question 3ab0 is a four-digit number divisible by 25. If the number formed from the two digits ab is a multiple of 17, then ab a) 51 b) 34 c) 85 d) 17 Correct Solution (c) We have given that the number 3ab0 is divisible by 25. Any number divisible by 25 ends with the last two digits 00, 25, 50, or 75. So, b0 should equal 00 or 50. Hence, b=0 or 5. Since a is now free to take any digit from 0 through 9, ab can have multiple values. We also have that ab is divisible by 17. The multiples of 17 are 17, 34, 51, 68, 85, 102, 119, 136, 153 and 170. Among these, the only number ending with 0 or 5 is 85. Hence, ab=85   Incorrect Solution (c) We have given that the number 3ab0 is divisible by 25. Any number divisible by 25 ends with the last two digits 00, 25, 50, or 75. So, b0 should equal 00 or 50. Hence, b=0 or 5. Since a is now free to take any digit from 0 through 9, ab can have multiple values. We also have that ab is divisible by 17. The multiples of 17 are 17, 34, 51, 68, 85, 102, 119, 136, 153 and 170. Among these, the only number ending with 0 or 5 is 85. Hence, ab=85   Question 35 of 35 35. Question A 40 litre mixture of oil and water contains oil and water in the ratio 3 : 2. Now, 10 litres of the mixture is removed and replaced with pure oil. This operation is done one more time. What is the ratio of water and oil in the resultant mixture? a) 12 : 31 b) 7 : 19 c) 9 : 31 d) 11 : 31 Correct Solution (c) Original quantity of oil and water is 24 litres and 16 litres. Let the final quantity of water be a. 10 litres is being removed from 40 litres twice. ∴ a/16 = [1 – (10/40)]^2 = 9/16 ∴ a = 9 Hence, the final solution has 9 litres of water and 31 litres of oil. ∴ Required ratio = 9 : 31 Hence, option c. Incorrect Solution (c) Original quantity of oil and water is 24 litres and 16 litres. Let the final quantity of water be a. 10 litres is being removed from 40 litres twice. ∴ a/16 = [1 – (10/40)]^2 = 9/16 ∴ a = 9 Hence, the final solution has 9 litres of water and 31 litres of oil. ∴ Required ratio = 9 : 31 Hence, option c. window.wpProQuizInitList = window.wpProQuizInitList || []; window.wpProQuizInitList.push({ id: '#wpProQuiz_3678', init: { quizId: 3678, mode: 1, globalPoints: 70, timelimit: 1800, resultsGrade: [0], bo: 704, qpp: 0, catPoints: [70], formPos: 0, lbn: "Test-summary", json: {"33017":{"type":"single","id":33017,"catId":0,"points":2,"correct":[1,0,0,0]},"33018":{"type":"single","id":33018,"catId":0,"points":2,"correct":[1,0,0,0]},"33019":{"type":"single","id":33019,"catId":0,"points":2,"correct":[0,0,1,0]},"33022":{"type":"single","id":33022,"catId":0,"points":2,"correct":[0,0,1,0]},"33024":{"type":"single","id":33024,"catId":0,"points":2,"correct":[0,0,0,1]},"33027":{"type":"single","id":33027,"catId":0,"points":2,"correct":[1,0,0,0]},"33029":{"type":"single","id":33029,"catId":0,"points":2,"correct":[0,1,0,0]},"33030":{"type":"single","id":33030,"catId":0,"points":2,"correct":[1,0,0,0]},"33032":{"type":"single","id":33032,"catId":0,"points":2,"correct":[0,0,1,0]},"33034":{"type":"single","id":33034,"catId":0,"points":2,"correct":[0,0,0,1]},"33037":{"type":"single","id":33037,"catId":0,"points":2,"correct":[0,1,0,0]},"33039":{"type":"single","id":33039,"catId":0,"points":2,"correct":[0,0,1,0]},"33042":{"type":"single","id":33042,"catId":0,"points":2,"correct":[0,0,1,0]},"33044":{"type":"single","id":33044,"catId":0,"points":2,"correct":[0,1,0,0]},"33045":{"type":"single","id":33045,"catId":0,"points":2,"correct":[0,0,1,0]},"33047":{"type":"single","id":33047,"catId":0,"points":2,"correct":[0,0,1,0]},"33049":{"type":"single","id":33049,"catId":0,"points":2,"correct":[0,0,1,0]},"33051":{"type":"single","id":33051,"catId":0,"points":2,"correct":[1,0,0,0]},"33053":{"type":"single","id":33053,"catId":0,"points":2,"correct":[1,0,0,0]},"33055":{"type":"single","id":33055,"catId":0,"points":2,"correct":[0,1,0,0]},"33057":{"type":"single","id":33057,"catId":0,"points":2,"correct":[0,0,1,0]},"33059":{"type":"single","id":33059,"catId":0,"points":2,"correct":[0,0,0,1]},"33061":{"type":"single","id":33061,"catId":0,"points":2,"correct":[1,0,0,0]},"33062":{"type":"single","id":33062,"catId":0,"points":2,"correct":[0,0,1,0]},"33063":{"type":"single","id":33063,"catId":0,"points":2,"correct":[0,0,1,0]},"33064":{"type":"single","id":33064,"catId":0,"points":2,"correct":[0,1,0,0]},"33065":{"type":"single","id":33065,"catId":0,"points":2,"correct":[1,0,0,0]},"33068":{"type":"single","id":33068,"catId":0,"points":2,"correct":[0,1,0,0]},"33070":{"type":"single","id":33070,"catId":0,"points":2,"correct":[0,0,1,0]},"33073":{"type":"single","id":33073,"catId":0,"points":2,"correct":[1,0,0,0]},"33076":{"type":"single","id":33076,"catId":0,"points":2,"correct":[0,1,0,0]},"33077":{"type":"single","id":33077,"catId":0,"points":2,"correct":[0,0,1,0]},"33078":{"type":"single","id":33078,"catId":0,"points":2,"correct":[0,1,0,0]},"33079":{"type":"single","id":33079,"catId":0,"points":2,"correct":[0,0,1,0]},"33081":{"type":"single","id":33081,"catId":0,"points":2,"correct":[0,0,1,0]}} } }); All the Best IASbaba

DAILY CURRENT AFFAIRS IAS | UPSC Prelims and Mains Exam – 25th April 2024

Archives (PRELIMS & MAINS Focus)   Ethylene Oxide Syllabus Prelims – Current Event Context: Food Safety and Standards Authority of India (FSSAI) will conduct checks on products of spice brands MDH and Everest Group after authorities in Hong Kong and Singapore last week determined that four of their spice mixes contained high levels of carcinogen ethylene oxide. Background:- FSSAI doesn’t permit use of ethylene oxide in any food product. About Ethylene Oxide Ethylene oxide is a pesticide that has been classified as a Group 1 carcinogen by the International Agency for Research on Cancer, meaning there is sufficient evidence from human studies that it can cause cancer. It is used by the spice industry as a fumigant to reduce microbial contamination, such as E. coli and Salmonella. It is a colourless, highly flammable and very reactive gas that kills bacteria, viruses and fungi. It is an industrial chemical. Ethylene oxide can damage DNA during sterilisation procedures. While the risk from occasional, low-level exposure may be minimal, spices and spice blends like those flagged are commonly used in household cooking across multiple dishes. This can lead to chronic, persistent exposure over time, which has been linked to an increased risk of cancers like leukemia, stomach cancer and breast cancer. It can even lead to respiratory irritation and lung injury, headache, nausea, vomiting, diarrhoea and shortness of breath. Ethylene oxide was banned by the European Union (EU) in 2011 for fumigation of food and animal feed during transport and storage. The use of ethylene oxide is now only permitted in disinfection and sterilisation of medical devices. Additional information Escherichia coli (E.coli) is a bacterial strain that is commonly found in the intestines of people and animals, fecal waste of cattle and humans. Salmonella is a group of bacteria that can cause food-borne illnesses known as salmonellosis. The World Health Organisation (WHO) identifies Salmonella as one of four key global causes of diarrhoeal diseases.Individuals who develop salmonellosis may show symptoms such as nausea, diarrhoea, fever, and abdominal cramps 12-72 hours after contracting the infection. Source: Indian Express Ongoing Israel - Palestine Conflict Syllabus Prelims & Mains – International Event Context: Six months after the October 7 attack, Israel’s war to eliminate Hamas has resulted in the near-complete destruction of Gaza and a dire humanitarian crisis with the area on the verge of famine.Iran and its allies have militarily united behind the Palestinian cause and expanded the confrontation to other parts of the region. Background: The dynamics of war have drastically changed following the massive Iranian drone and missile attacks on military targets in Israel on April 14. This was in retaliation to Israeli aerial strikes on an Iranian diplomatic compound in Damascus on April 1 in which top Iranian generals were killed. Israel had not seen direct attacks by a state on its soil since 1991 when Saddam Hussein fired 42 Scud missiles. A volatile Middle East: The geopolitics of the region has already undergone a sea change since October 7 in four substantive ways: First, thawing of the hitherto icy Shia-Sunni relationship, with Hezbollah, Houthis and Iran (all Shia entities) extending open support to “Sunni” Palestinians, presenting regional unity via the Axis of Resistance, opposing US-Israel domination. The first signs of the Resistance surfaced early on in the war, with conflict with Hezbollah in northern Israel leading to the displacement of thousands of Israeli civilians who are yet to return. This active front is likely to simmer. Second, the war has expanded across the region, pushing the US to become the fulcrum of efforts to contain expansion and making it a primary target. Third, the Saudi-aligned Sunni Arab States have adopted the political and diplomatic route, advocating a ceasefire, supporting humanitarian assistance, and calling for a two-state solution while remaining in line with US-led initiatives. Fourth, the Yemen-based Houthis — a non-state actor devoid of a navy — have managed to successfully disrupt a critical sea lane of communication for five months, impairing trade substantively. Additional Information The expansion of the war has already caused economic distress to the Israeli economy .The Israeli economy has reportedly shrunk by 20 per cent in the last quarter of 2023. Two crucial Israeli ports, Eilat on the Red Sea and Haifa on the eastern Mediterranean have been sporadically attacked.  The Eilat port is said to be down to a quarter of its business, while Israeli media reports an acute shortage of labour in Israel, primarily on account of military conscription and volunteerism. Source: Indian Express GREEN FINANCING Syllabus Prelims & Mains – Economy & Environment Context: National Bank for Agriculture and Rural Development (NABARD) recently unveiled its Climate Strategy 2030 document which aims to address India’s need for green financing. Background: NABARD’s Climate Strategy 2030 is structured around four key pillars which include accelerating green lending across sectors, playing a broader market-making role, internal green transformation and strategic resource mobilisation. What is Green financing Green financing refers to any structured financial activity – a product or service – that has been designed to ensure a better environmental outcome. It encompasses a variety of loans, debt mechanisms, and investments aimed at promoting the development of green projects or minimizing the climate impact of regular projects. Purpose and Importance: Green finance aims to align financial activities with environmental goals. It plays a crucial role in achieving several of the United Nations’ Sustainable Development Goals by supporting sustainable development priorities. Examples of Green Finance Projects: Renewable Energy and Energy Efficiency: Funding projects related to clean energy sources and energy-saving technologies. Pollution Prevention and Control: Financing initiatives that reduce pollution and promote cleaner practices. Biodiversity Conservation: Supporting efforts to protect and preserve biodiversity. Circular Economy Initiatives: Investing in projects that promote resource efficiency and waste reduction. Sustainable Use of Natural Resources and Land: Financing projects that balance economic development with environmental conservation. Green Bonds: Green bonds are a common green finance instrument. They adhere to specific criteria, including the use of proceeds, project evaluation, proper management, and detailed reporting. The United States, China, and France are the three largest issuers of green bonds. Source: Hindu ENGLISH CHANNEL Syllabus Prelims – Geography Context: Recently, five asylum seekers died while crossing the English Channel from France to Britain in an overcrowded small boat. Background: In recent years, the English Channel has become a focal point for migrants and asylum seekers attempting to reach the UK. Many undertake dangerous journeys in small boats, hoping to find safety and better opportunities.   About English Channel: The English Channel, also known as “La Manche,” is an arm of the Atlantic Ocean that separates Southern England from northern France. It links to the southern part of the North Sea via the Strait of Dover at its northeastern end. Notably, it is the busiest shipping area in the world. Historically, the channel played a pivotal role in Britain’s naval supremacy and served as a natural defence mechanism against invasions during significant events like the Napoleonic Wars and World War II. Source: Reuters VOYAGER 1 SPACECRAFT Syllabus Prelims – Science Context: NASA Voyager 1 spacecraft has begun sending readable communications again after months of transmitting gibberish back to Earth. Background: NASA’s Voyager flight team detected a glitch in November and worked to fix the problem via code, leading to the successful return of readable data. About VOYAGER 1 SPACECRAFT : Voyager 1 was launched by NASA on September 5, 1977. Voyager 1 is part of the Voyager program, which also includes Voyager 2. Mission Objective: To study the outer Solar System and beyond, including flybys of Jupiter and Saturn. Discoveries: It discovered a thin ring around Jupiter and two new Jovian moons: Thebe and Metis. At Saturn, it found five new moons and a new ring called the G-ring. Interstellar Achievement: Voyager 1 was the first human-made object to cross the heliosphere and enter interstellar space in August 2012. Current Status: It is the most distant human-made object from Earth and is expected to send data until at least 2025. Golden Record: Carries a golden record with sounds and images representing life and culture on Earth. Source: Live Science Climate Change and Human Rights Syllabus Prelims – GS 2 & GS 3 Context: The Supreme Court of India acknowledged the right to protection from climate change impacts as part of the fundamental rights to life (Article 21) and equality (Article 19) enshrined in the Indian Constitution. Background: The arguments were a part of a verdict by a three-judge bench headed by the Chief Justice of India DY Chandrachud hearing a case on the conservation of the great India bustard and the lesser florican. The intersection of climate change and human rights: Climate change can directly affect people’s right to life by causing extreme weather events like hurricanes or floods, which can lead to loss of life and property. For example, in low-lying coastal areas, rising sea levels due to climate change can threaten people’s homes and livelihoods, forcing them to relocate. Climate change can impact water sources, leading to water scarcity or contamination. This affects people’s right to clean water and sanitation. In regions where droughts are becoming more frequent due to climate change, communities may struggle to access safe drinking water, leading to health issues. Climate change can exacerbate health problems, especially for vulnerable populations. For instance, increased heat waves can lead to heat-related illnesses and deaths, affecting the right to health. Climate change-induced events such as sea-level rise, extreme weather events, or desertification can force people to migrate or be displaced from their homes. This intersects with human rights, particularly the right to residence and the right to seek asylum. Climate change can disproportionately affect indigenous communities that rely heavily on natural resources for their livelihoods and cultural practices. For instance, changes in ecosystems due to climate change can threaten traditional livelihoods like farming or fishing, impacting indigenous peoples’ rights to land, resources, and cultural heritage. The Supreme Court’s interpretation of constitutional provisions concerning climate change: Article 48A which mandates environmental protection and Article 51A(g) which promotes wildlife conservation, implicitly guarantee a right to be safeguarded from climate change. Article 21 recognises the right to life and personal liberty while Article 14 indicates that all persons shall have equality before the law and the equal protection of laws. These articles are important sources of the right to a clean environment and the right against the adverse effects of climate change. In MC Mehta vs Kamal Nath Case, 2000, the Supreme Court stated that the right to a clean environment is an extension of the right to life. Issues/Challenges in balancing climate change mitigation with human rights protection: Some climate mitigation measures may conflict with human rights, such as restrictions on land use for conservation projects or displacement due to renewable energy infrastructure development. Finding solutions that minimise negative impacts while maximising benefits is challenging. Climate actions like transitioning to renewable energy or implementing carbon pricing can impact access to essential resources like energy, water, and food, especially for marginalised communities. Climate-induced migration can strain social systems and lead to conflicts over resources and rights in host communities. Managing migration flows in a way that respects the rights of both migrants and host populations is a multifaceted challenge. Balancing efforts to reduce greenhouse gas emissions (mitigation) with investments in adaptation to climate impacts can be challenging. Prioritising one over the other can have implications for human rights, particularly for communities already facing climate-related risks. Climate change is a global issue requiring international cooperation. Balancing national climate goals with global responsibilities and ensuring that climate actions do not undermine the rights of vulnerable communities across borders is a complex task. Source: Down To Earth Practice MCQs Daily Practice MCQs Q1.) With reference to the Voyager 1 spacecraft, consider the following statements: Voyager 1 was launched by ISRO. The primary mission of Voyager 1 was to explore the outer planets of solar system. Which of the statements given above is/are correct? 1 only 2 only Both 1 and 2 Neither 1 nor 2 Q2.) English Channel separates England from Spain France Germany Poland Q3.) With reference to the Green Financing, consider the following statements: Green financing aims to align financial activities with environmental goals. It encompasses a variety of debt mechanisms and investments aimed at promoting the development of green projects. Green bonds are a common green finance instrument. Which of the statements given above are correct? Only one Only two All three None Comment the answers to the above questions in the comment section below!! ANSWERS FOR ’  25th April  2024 – Daily Practice MCQs’ will be updated along with tomorrow’s Daily Current Affairs.st ANSWERS FOR  24th April – Daily Practice MCQs Answers- Daily Practice MCQs Q.1) – b Q.2) – c Q.3) – d

Daily Prelims CA Quiz

UPSC Quiz – 2024 : IASbaba’s Daily Current Affairs Quiz 25th April 2024

The Current Affairs questions are based on sources like ‘The Hindu’, ‘Indian Express’ and ‘PIB’, which are very important sources for UPSC Prelims Exam. The questions are focused on both the concepts and facts. The topics covered here are generally different from what is being covered under ‘Daily Current Affairs/Daily News Analysis (DNA) and Daily Static Quiz’ to avoid duplication. The questions would be published from Monday to Saturday before 2 PM. One should not spend more than 10 minutes on this initiative. Gear up and Make the Best Use of this initiative. Do remember that, “the difference between Ordinary and EXTRA-Ordinary is PRACTICE!!” Important Note: Don’t forget to post your marks in the comment section. Also, let us know if you enjoyed today’s test 🙂After completing the 5 questions, click on ‘View Questions’ to check your score, time taken, and solutions.To take the Test Click Here

[DAY 46] 60 DAY RAPID REVISION (RaRe) SERIES for UPSC Prelims 2024 – ECONOMY, CURRENT AFFAIRS & CSAT TEST SERIES!

Archives Hello Friends The 60 Days Rapid Revision (RaRe) Series is IASbaba’s Flagship Initiative recommended by Toppers and loved by the aspirants’ community every year. It is the most comprehensive program which will help you complete the syllabus, revise and practice tests on a daily basis. The Programme on a daily basis includes Daily Prelims MCQs from Static (Monday – Saturday) Daily Static Quiz will cover all the topics of static subjects – Polity, History, Geography, Economics, Environment and Science and technology. 20 questions will be posted daily and these questions are framed from the topics mentioned in the schedule. It will ensure timely and streamlined revision of your static subjects. Daily Current Affairs MCQs (Monday – Saturday) Daily 5 Current Affairs questions, based on sources like ‘The Hindu’, ‘Indian Express’ and ‘PIB’, would be published from Monday to Saturday according to the schedule. Daily CSAT Quiz (Monday – Friday) CSAT has been an Achilles heel for many aspirants. Daily 5 CSAT Questions will be published. Note – Daily Test of 20 static questions, 10 current affairs, and 5 CSAT questions. (35 Prelims Questions) in QUIZ FORMAT will be updated on a daily basis. To Know More about 60 Days Rapid Revision (RaRe) Series – CLICK HERE   60 Day Rapid Revision (RaRe) Series Schedule – CLICK HERE  Important Note Comment your Scores in the Comment Section. This will keep you accountable, responsible and sincere in days to come. It will help us come out with the Cut-Off on a Daily Basis. Let us know if you enjoyed today’s test 🙂  You can post your comments in the given format  (1) Your Score (2) Matrix Meter (3) New Learning from the Test Time limit: 0 Test-summary 0 of 34 questions completed Questions: 1 2 3 4 5 6 7 8 9 10 11 12 13 14 15 16 17 18 19 20 21 22 23 24 25 26 27 28 29 30 31 32 33 34 Information The following Test is based on the syllabus of 60 Days Plan-2023 for UPSC IAS Prelims 2022. To view Solutions, follow these instructions: Click on – ‘Start Test’ button Solve Questions Click on ‘Test Summary’ button Click on ‘Finish Test’ button Now click on ‘View Questions’ button – here you will see solutions and links. You have already completed the test before. Hence you can not start it again. Test is loading... You must sign in or sign up to start the test. You have to finish following test, to start this test: Results 0 of 34 questions answered correctly Your time: Time has elapsed You have scored 0 points out of 0 points, (0) Average score     Your score     Categories Not categorized 0% Your result has been entered into leaderboard Loading Name: E-Mail: Captcha: maximum of 68 points Pos. Name Entered on Points Result Table is loading No data available 1 2 3 4 5 6 7 8 9 10 11 12 13 14 15 16 17 18 19 20 21 22 23 24 25 26 27 28 29 30 31 32 33 34 Answered Review Question 1 of 34 1. Question Consider the following statements regarding deficit financing External grants are the best means to finance the deficit. External borrowings are preferred over internal borrowings due to the ‘crowding out effect’. Internal borrowings to finance the deficit might hamper the economy’s investment level. How many of the above statements are correct? a) Only one b) Only two c) All three d) None Correct Solution (c) Statement 1 Statement 2 Statement 3 Correct Correct Correct External Grants are the best mode to finance the deficit as they do not create any obligation- neither interest payments nor any repayment. They are essentially free. External borrowings are preferred over internal borrowings due to the ‘crowding out effect’. If the government goes on borrowing from the banks within the country, the liquidity with banks reduces. Less liquidity with banks leaves private entities with limited money. This further leads to depression in the level of investment. Incorrect Solution (c) Statement 1 Statement 2 Statement 3 Correct Correct Correct External Grants are the best mode to finance the deficit as they do not create any obligation- neither interest payments nor any repayment. They are essentially free. External borrowings are preferred over internal borrowings due to the ‘crowding out effect’. If the government goes on borrowing from the banks within the country, the liquidity with banks reduces. Less liquidity with banks leaves private entities with limited money. This further leads to depression in the level of investment. Question 2 of 34 2. Question Consider the following statements regarding the Balance of Payment: The balance of payments (BOP) is a statement of all transactions made between residents in one country and the rest of the world over a defined period. The industrial revolution increased international economic integration, and balance-of-payment crises became more frequent. Select the correct answer using the code given below. a) 1 only b) 2 only c) Both 1 and 2 d) Neither 1 nor 2 Correct Solution (c) Statement 1 Statement 2 Correct Correct The outcome of the total transactions of an economy with the outside world in one year is known as the balance of payment (BoP) of the economy. Basically, it is the net outcome of the current and capital accounts of an economy. Before the 19th century, international transactions were denominated in gold, providing little flexibility for countries experiencing trade deficits. Growth was low, so stimulating a trade surplus was the primary method of strengthening a nation’s financial position. National economies were not well integrated, however, so steep trade imbalances rarely provoked crises. The industrial revolution increased international economic integration, and balance-of-payment crises began to occur more frequently. Incorrect Solution (c) Statement 1 Statement 2 Correct Correct The outcome of the total transactions of an economy with the outside world in one year is known as the balance of payment (BoP) of the economy. Basically, it is the net outcome of the current and capital accounts of an economy. Before the 19th century, international transactions were denominated in gold, providing little flexibility for countries experiencing trade deficits. Growth was low, so stimulating a trade surplus was the primary method of strengthening a nation’s financial position. National economies were not well integrated, however, so steep trade imbalances rarely provoked crises. The industrial revolution increased international economic integration, and balance-of-payment crises began to occur more frequently. Question 3 of 34 3. Question Consider the following statements concerning Capital Account Convertibility: It refers to the removal of restraints on international flows on a country’s capital account. S. Tarapore committee examined the feasibility of capital account convertibility in India. Select the correct answer using the code given below. a) 1 only b) 2 only c) Both 1 and 2 d) Neither 1 nor 2 Correct Solution (c) Statement 1 Statement 2 Correct Correct Capital Account Convertibility refers to the removal of restraints on international flows on a country’s capital account, enabling full currency convertibility and opening the financial system. Presently, India has current account convertibility. This means one can import and export goods or receive or make payments for services rendered. However, investments and borrowings are restricted. The term Capital Account Convertibility was coined by RBI and this term is almost synonymous with the RBI committee headed by SS Tarapore. The SS Tarapore Committee on Capital Account Convertibility was an experts’ committee to study the feasibility of capital account convertibility in India. It submitted its report in 1997. Incorrect Solution (c) Statement 1 Statement 2 Correct Correct Capital Account Convertibility refers to the removal of restraints on international flows on a country’s capital account, enabling full currency convertibility and opening the financial system. Presently, India has current account convertibility. This means one can import and export goods or receive or make payments for services rendered. However, investments and borrowings are restricted. The term Capital Account Convertibility was coined by RBI and this term is almost synonymous with the RBI committee headed by SS Tarapore. The SS Tarapore Committee on Capital Account Convertibility was an experts’ committee to study the feasibility of capital account convertibility in India. It submitted its report in 1997. Question 4 of 34 4. Question Consider the following statements regarding Nominal Effective Exchange Rates (NEER) NEER is a measure of the value of a currency against a weighted average of several foreign currencies. An increase in NEER indicates an appreciation of the local currency against the weighted basket of currencies of its trading partners. The basket of foreign currencies is selected based on an international standard set by the IMF. How many of the above statements are correct? a) Only one b) Only two c) All three d) None Correct Solution (b) Statement 1 Statement 2 Statement 3 Correct Correct Incorrect The Nominal Effective Exchange Rate (NEER) of the rupee is an unadjusted weighted average of exchange rates before the currencies of India’s major trading partners. An increase in NEER indicates an appreciation of the local currency against the weighted basket of currencies of its trading partners. The basket of foreign currencies basket is chosen based on the domestic country’s most important trading partners as well as other major currencies. There is no international standard for selecting a basket of currencies. Rather it depends upon weighted average of currencies of India’s major trading partners However, many different institutions rely on International Financial Statistics (IFS) published by the IMF Incorrect Solution (b) Statement 1 Statement 2 Statement 3 Correct Correct Incorrect The Nominal Effective Exchange Rate (NEER) of the rupee is an unadjusted weighted average of exchange rates before the currencies of India’s major trading partners. An increase in NEER indicates an appreciation of the local currency against the weighted basket of currencies of its trading partners. The basket of foreign currencies basket is chosen based on the domestic country’s most important trading partners as well as other major currencies. There is no international standard for selecting a basket of currencies. Rather it depends upon weighted average of currencies of India’s major trading partners However, many different institutions rely on International Financial Statistics (IFS) published by the IMF Question 5 of 34 5. Question Consider the following statements regarding Foreign Direct Investment (FDI) in India FDI in India was allowed after the BoP crisis of 1991. FDI inflows are a component of the current account of BoP. In India, FDI is not allowed in the real estate sector. How many of the above statements are correct? a) Only one b) Only two c) All three d) None Correct Solution (b) Statement 1 Statement 2 Statement 3 Correct Incorrect Correct FDI was introduced in India in 1991 after the LPG reforms which opened the economy to the world. India passed Foreign Exchange Management Act (FEMA), 1999 to consolidate and amend the law relating to foreign exchange with the objective of facilitating external trade and payments and promoting the orderly development and maintenance of the foreign exchange market in India. FDI inflows are reported under the capital account of the Balance of payments (BoP) The prohibited sectors include Betting, Gambling, Lottery; Chit funds; Nidhi Company; Real Estate, etc. FDI for most cases can be brought through Automatic Route and for the remaining case through Government approval.   Incorrect Solution (b) Statement 1 Statement 2 Statement 3 Correct Incorrect Correct FDI was introduced in India in 1991 after the LPG reforms which opened the economy to the world. India passed Foreign Exchange Management Act (FEMA), 1999 to consolidate and amend the law relating to foreign exchange with the objective of facilitating external trade and payments and promoting the orderly development and maintenance of the foreign exchange market in India. FDI inflows are reported under the capital account of the Balance of payments (BoP) The prohibited sectors include Betting, Gambling, Lottery; Chit funds; Nidhi Company; Real Estate, etc. FDI for most cases can be brought through Automatic Route and for the remaining case through Government approval.   Question 6 of 34 6. Question Concerning FDI and FII, consider the following statements: Both are types of foreign investments, but there is a stark difference in how they are operated, whom they target, and the returns that can be derived from both. Through FDI, the investors have a good amount of management control over the operations of the business in the host country. Through FII, the investors generally have no control of any companies or banks in the host country in which they are investing. How many of the above statements are correct? a) Only one b) Only two c) All three d) None Correct Solution (c) Statement 1 Statement 2 Statement 3 Correct Correct Correct Both are types of foreign investments, there is, however, a stark difference in the way they are operated, whom they target, and the returns that can be derived from both. FDI being a direct form of investment into a foreign company, the investors are more interested and even get to enjoy a higher control on the management of the company, even if it is in a foreign country. FII just allows for funds to be invested into the financial market of the host country and therefore doesn’t have much hold onto the managerial decisions, or rather are referred to as just passive investors Incorrect Solution (c) Statement 1 Statement 2 Statement 3 Correct Correct Correct Both are types of foreign investments, there is, however, a stark difference in the way they are operated, whom they target, and the returns that can be derived from both. FDI being a direct form of investment into a foreign company, the investors are more interested and even get to enjoy a higher control on the management of the company, even if it is in a foreign country. FII just allows for funds to be invested into the financial market of the host country and therefore doesn’t have much hold onto the managerial decisions, or rather are referred to as just passive investors Question 7 of 34 7. Question Consider the following statements regarding Forex Reserve: Forex Reserves comprise foreign currency assets and Special Drawing Rights only. The adequacy of Forex Reserves can be measured by Import Cover. An economy’s Reserve Tranche in the IMF is not a part of its forex reserve. How many of the above statements are correct? a) Only one b) Only two c) All three d) None Correct Solution (a) Statement 1 Statement 2 Statement 3 Incorrect Correct Incorrect Forex Reserves comprise foreign currency assets added with its gold reserves, SDRs (Special Drawing Rights), and Reserve Tranche in the IMF. The adequacy of Forex Reserves is measured by Import Cover.   A reserve tranche is a portion of the required quota of currency each member country must provide to the International Monetary Fund (IMF) that can be utilized for its purposes—without a service fee or economic reform conditions. Reserve Tranche in IMF also comprises the forex reserves. Incorrect Solution (a) Statement 1 Statement 2 Statement 3 Incorrect Correct Incorrect Forex Reserves comprise foreign currency assets added with its gold reserves, SDRs (Special Drawing Rights), and Reserve Tranche in the IMF. The adequacy of Forex Reserves is measured by Import Cover.   A reserve tranche is a portion of the required quota of currency each member country must provide to the International Monetary Fund (IMF) that can be utilized for its purposes—without a service fee or economic reform conditions. Reserve Tranche in IMF also comprises the forex reserves. Question 8 of 34 8. Question Consider the following statements about types of exchange rates and interventions. Exchange rate       – Interventions Flexible Exchange Rate – Interventions by the Government Managed Exchange Rate – Market-Driven Interventions Fixed Exchange Rate – Currency Pegged system How many of the above pairs are correctly matched? a) Only one b) Only two c) All three d) None Correct Solution (a) Statement 1 Statement 2 Statement 3 Incorrect Incorrect Correct Floating/Flexible Exchange Rates are also called market-driven or based exchange rates, which are regulated by factors such as the demand and supply of the domestic and foreign currencies in the concerned economy. In the floating exchange rate system, a domestic currency is left free to float against a number of foreign currencies in its foreign exchange market and determine its value. Failure of the gold standard and the Bretton Woods Agreement led to the increased popularity of this system. Managed Exchange Rate:  A managed-exchange-rate system is a hybrid or mixture of the fixed and flexible exchange rate systems in which the government of the economy attempts to affect the exchange rate directly by buying or selling foreign currencies or indirectly, through monetary policy (by lowering/raising interest rates on foreign currency bank accounts, etc. Fixed Exchange Rate:  In this system, the government or central bank ties the country’s currency official exchange rate to another country’s currency (currency peg) or the price of gold (gold standard). Fixed rates provide greater certainty for exporters and importers and also help the government maintain low inflation. The purpose of a fixed exchange rate system is to keep a currency’s value within a narrow band. Incorrect Solution (a) Statement 1 Statement 2 Statement 3 Incorrect Incorrect Correct Floating/Flexible Exchange Rates are also called market-driven or based exchange rates, which are regulated by factors such as the demand and supply of the domestic and foreign currencies in the concerned economy. In the floating exchange rate system, a domestic currency is left free to float against a number of foreign currencies in its foreign exchange market and determine its value. Failure of the gold standard and the Bretton Woods Agreement led to the increased popularity of this system. Managed Exchange Rate:  A managed-exchange-rate system is a hybrid or mixture of the fixed and flexible exchange rate systems in which the government of the economy attempts to affect the exchange rate directly by buying or selling foreign currencies or indirectly, through monetary policy (by lowering/raising interest rates on foreign currency bank accounts, etc. Fixed Exchange Rate:  In this system, the government or central bank ties the country’s currency official exchange rate to another country’s currency (currency peg) or the price of gold (gold standard). Fixed rates provide greater certainty for exporters and importers and also help the government maintain low inflation. The purpose of a fixed exchange rate system is to keep a currency’s value within a narrow band. Question 9 of 34 9. Question Which of the following are the possible risk of rising external debt on the Indian economy? Risk due to interest rate fluctuations. Risk due to exchange rate fluctuations. Government will have less flexibility to manipulate currency to boost exports. How many of the above statements are correct? a) Only one b) Only two c) All three d) None Correct Solution (c) Statement 1 Statement 2 Statement 3 Correct Correct Correct There are certain major risks involved in foreign borrowings. One is that, like in the case of domestic borrowings, there could be unexpected changes in the interest rates charged on these loans. This can, for instance, cause widespread default when rates rise as borrowers may not be able to make higher interest payments, thus raising the risks of a systemic crisis. A major risk is unexpected changes in the exchange rates of currencies. An unexpected fall in the value of the rupee (depreciation of the rupee) can cause severe difficulties for Indian companies that need to pay back dollar-denominated loans as they will now have to shell out more rupees than they had previously estimated to buy the necessary dollars. This eventually results in higher debt on the balance sheet which may affect many financial ratios adversely. The government loses the flexibility to manipulate currency to lower levels to boost exports as it will further worsen the debt payment requirements. High levels of government borrowing from international sources can also hamper the domestic industry by crowding out their sources of funds. Incorrect Solution (c) Statement 1 Statement 2 Statement 3 Correct Correct Correct There are certain major risks involved in foreign borrowings. One is that, like in the case of domestic borrowings, there could be unexpected changes in the interest rates charged on these loans. This can, for instance, cause widespread default when rates rise as borrowers may not be able to make higher interest payments, thus raising the risks of a systemic crisis. A major risk is unexpected changes in the exchange rates of currencies. An unexpected fall in the value of the rupee (depreciation of the rupee) can cause severe difficulties for Indian companies that need to pay back dollar-denominated loans as they will now have to shell out more rupees than they had previously estimated to buy the necessary dollars. This eventually results in higher debt on the balance sheet which may affect many financial ratios adversely. The government loses the flexibility to manipulate currency to lower levels to boost exports as it will further worsen the debt payment requirements. High levels of government borrowing from international sources can also hamper the domestic industry by crowding out their sources of funds. Question 10 of 34 10. Question Consider the following statements concerning External Commercial Borrowings (ECBs): These are loans availed by an Indian entity from a non-resident lender in foreign currency. ECBs can be procured from foreign sources for a minimum average maturity period of 8 years. The proceeds from ECB cannot be utilised for real estate and equity investment. How many of the above statements are correct? a) Only one b) Only two c) All three d) None Correct Solution (b) Statement 1 Statement 2 Statement 3 Correct Incorrect Correct External Commercial Borrowings (ECBs) are basically loans availed by an Indian entity from a non-resident lender. Most of these loans are provided by foreign commercial banks and other institutions in foreign currency. It is a loan availed from non-resident lenders with a minimum average maturity of 3 years. It includes commercial bank loans, buyers’ credit, suppliers’ credit, securitized instruments such as Floating Rate Notes and Fixed Rate Bonds, etc., credit from official export credit agencies, and commercial borrowings from Multilateral Financial Institutions. The External Commercial Borrowing (ECB) policy is regularly reviewed by the Ministry of Finance in consultation with the Reserve Bank of India (RBI) to keep it in tune with the evolving macroeconomic situation, changing market conditions, sectoral requirements, etc. The RBI has decided to keep the minimum average maturity period at 3 years (earlier 5 years) for all ECBs, irrespective of the amount of borrowing, except for borrowers specifically permitted to borrow for a shorter period, like manufacturing companies. The ECB proceeds cannot be utilized for real estate activities, investment in the capital market, equity investment, working capital purposes (except foreign equity holder), and repayment of Rupee loans (except foreign equity holder). Incorrect Solution (b) Statement 1 Statement 2 Statement 3 Correct Incorrect Correct External Commercial Borrowings (ECBs) are basically loans availed by an Indian entity from a non-resident lender. Most of these loans are provided by foreign commercial banks and other institutions in foreign currency. It is a loan availed from non-resident lenders with a minimum average maturity of 3 years. It includes commercial bank loans, buyers’ credit, suppliers’ credit, securitized instruments such as Floating Rate Notes and Fixed Rate Bonds, etc., credit from official export credit agencies, and commercial borrowings from Multilateral Financial Institutions. The External Commercial Borrowing (ECB) policy is regularly reviewed by the Ministry of Finance in consultation with the Reserve Bank of India (RBI) to keep it in tune with the evolving macroeconomic situation, changing market conditions, sectoral requirements, etc. The RBI has decided to keep the minimum average maturity period at 3 years (earlier 5 years) for all ECBs, irrespective of the amount of borrowing, except for borrowers specifically permitted to borrow for a shorter period, like manufacturing companies. The ECB proceeds cannot be utilized for real estate activities, investment in the capital market, equity investment, working capital purposes (except foreign equity holder), and repayment of Rupee loans (except foreign equity holder). Question 11 of 34 11. Question Which of the following statements is/are correct about Foreign Exchange Management Act (FEMA)? In FEMA If there was a need for transferring of funds for external operations, then prior approval of the Reserve Bank of India (RBI) was required. FEMA was repealed in 1998 by Vajpayee Government. In FEMA there is no provision for Special Director (Appeals) and Special Tribunal. How many of the above statements are correct? a) Only one b) Only two c) All three d) None Correct Solution (d) Statement 1 Statement 2 Statement 3 Incorrect Incorrect Incorrect For External trade and remittances, there is no need for prior approval from the Reserve Bank of India (RBI) FERA was repealed in 1998 by the government of Atal Bihari Vajpayee and replaced by the Foreign Exchange Management Act, which liberalised foreign exchange controls and restrictions on foreign investment. Section 17 of the FEMA Act provides for the appointment of Special Director (Appeals). Section 17(1) provides that the Central Government shall, by notification, appoint one or more Special Directors (Appeals) to hear appeals against the orders of the Adjudicating Authorities. Hence there is a provision Incorrect Solution (d) Statement 1 Statement 2 Statement 3 Incorrect Incorrect Incorrect For External trade and remittances, there is no need for prior approval from the Reserve Bank of India (RBI) FERA was repealed in 1998 by the government of Atal Bihari Vajpayee and replaced by the Foreign Exchange Management Act, which liberalised foreign exchange controls and restrictions on foreign investment. Section 17 of the FEMA Act provides for the appointment of Special Director (Appeals). Section 17(1) provides that the Central Government shall, by notification, appoint one or more Special Directors (Appeals) to hear appeals against the orders of the Adjudicating Authorities. Hence there is a provision Question 12 of 34 12. Question If external debt vis-s-vis internal debt in the government’s total debt increases, what would be its likely impact on the Indian economy? It may bring down the interest that the government pays on its debt. In the short term, it may lead to the rupee depreciating against the dollar. It may have a negative impact on the Make in India initiative. It can hamper countries’ ability to invest in social and physical infrastructure. How many of the above statements are correct? a) Only one b) Only two c) Only three d) All four Correct Solution (c) Sovereign borrowing means raising money from foreign markets by issuing government bonds by the government/central bank of a country. Statement 1 Statement 2 Statement 3 Statement 4 Correct Incorrect Correct Correct Global interest rates are at historic lows, so the government should tap the potential and allocate needed money for the country’s growth and development. Borrowing in dollars is expected to be cheaper, and hence, bring down the interest that the government pays on its debt. It may lead to the rupee appreciating against the dollar, at least in the short run. When the bonds are sold and the dollars (or any other foreign currency for that matter) are brought back to India, they will have to be converted into rupees. This will push up the demand for rupees and eventually lead to the rupee appreciating in value against the dollar. In the short term, an appreciating rupee will hurt India’s exports, which are struggling already. It will make imports cheaper and hurt domestic producers competing against them. The government of India launched the “Make in India” scheme to promote manufacturing in India. But with the appreciation of the rupee, imports will be cheaper and it may impact the Make-in-India initiative. Excessive levels of foreign debt can hamper countries’ ability to invest in their economic future—whether it be via infrastructure, education, or health care—as their limited revenue goes to servicing their loans. This thwarts long-term economic growth. The United Nations has also linked high levels of foreign debt and a government’s dependency on foreign assistance to human rights abuses. Economic distress causes governments to cut social spending and reduces the resources it has to enforce labour standards and human rights.   Incorrect Solution (c) Sovereign borrowing means raising money from foreign markets by issuing government bonds by the government/central bank of a country. Statement 1 Statement 2 Statement 3 Statement 4 Correct Incorrect Correct Correct Global interest rates are at historic lows, so the government should tap the potential and allocate needed money for the country’s growth and development. Borrowing in dollars is expected to be cheaper, and hence, bring down the interest that the government pays on its debt. It may lead to the rupee appreciating against the dollar, at least in the short run. When the bonds are sold and the dollars (or any other foreign currency for that matter) are brought back to India, they will have to be converted into rupees. This will push up the demand for rupees and eventually lead to the rupee appreciating in value against the dollar. In the short term, an appreciating rupee will hurt India’s exports, which are struggling already. It will make imports cheaper and hurt domestic producers competing against them. The government of India launched the “Make in India” scheme to promote manufacturing in India. But with the appreciation of the rupee, imports will be cheaper and it may impact the Make-in-India initiative. Excessive levels of foreign debt can hamper countries’ ability to invest in their economic future—whether it be via infrastructure, education, or health care—as their limited revenue goes to servicing their loans. This thwarts long-term economic growth. The United Nations has also linked high levels of foreign debt and a government’s dependency on foreign assistance to human rights abuses. Economic distress causes governments to cut social spending and reduces the resources it has to enforce labour standards and human rights.   Question 13 of 34 13. Question Consider the following pairs: Types of Currency – Characteristics Hard Currency – High level of liquidity in the international currency market Heated Currency – Domestic currency which is under the pressure of depreciation Soft Currency – Currency has very high demand in the Forex market How many of the above pairs are correctly matched? a) Only one b) Only two c) All three d) None Correct Solution (b) Statement 1 Statement 2 Statement 3 Correct Correct Incorrect Hard currency is the international currency in which the highest faith is shown and is needed by every economy. The strongest currency in the world is one that has a high level of liquidity. The economy with the highest as well as highly diversified exports that are compulsive imports for other countries (as of high-level technology, defense products, lifesaving medicines, and petroleum products) will also create high demand for its currency in the end become the hard currency. It is always scarce. Heated currency is a term used in the forex market to denote the domestic currency which is under enough pressure (heat) of depreciation due to a hard currency’s high tendency of exiting the economy (since it has become hot). It is also known as currency under heat or hammering. A soft currency is one with a value that fluctuates because there is less demand for that currency in the forex markets. This lack of demand may be driven by a variety of factors, but is most often a result of the country’s political or economic uncertainty. Soft currencies are unlikely to be held by central banks as foreign reserves. Incorrect Solution (b) Statement 1 Statement 2 Statement 3 Correct Correct Incorrect Hard currency is the international currency in which the highest faith is shown and is needed by every economy. The strongest currency in the world is one that has a high level of liquidity. The economy with the highest as well as highly diversified exports that are compulsive imports for other countries (as of high-level technology, defense products, lifesaving medicines, and petroleum products) will also create high demand for its currency in the end become the hard currency. It is always scarce. Heated currency is a term used in the forex market to denote the domestic currency which is under enough pressure (heat) of depreciation due to a hard currency’s high tendency of exiting the economy (since it has become hot). It is also known as currency under heat or hammering. A soft currency is one with a value that fluctuates because there is less demand for that currency in the forex markets. This lack of demand may be driven by a variety of factors, but is most often a result of the country’s political or economic uncertainty. Soft currencies are unlikely to be held by central banks as foreign reserves. Question 14 of 34 14. Question Consider the following statements with reference to the foreign exchange market Depreciation in a currency can only take place if the economy follows the floating exchange rate system. Appreciation rates for different assets are not fixed by any government as they depend upon market mechanisms. Devaluation is the value loss of the domestic currency against a foreign currency, mainly driven by market forces. Revaluation is the official appreciation of domestic currency against any foreign currency. How many of the above statements are correct? a) Only one b) Only two c) Only three d) All four Correct Solution (c) Statement 1 Statement 2 Statement 3 Statement 4 Correct Correct Incorrect Correct In the foreign exchange market, it is a situation when a domestic currency loses its value in front of a foreign currency if it is market-driven. It means depreciation in a currency can only take place if the economy follows the floating exchange rate system. If a free-floating domestic currency increases its value against the value of a foreign currency, it is appreciated. In the domestic economy, if a fixed asset has seen an increase in its value, it is also known as appreciation. Appreciation rates for different assets are not fixed by any government as they depend upon market mechanisms. When the exchange rate of a domestic currency is cut down by its government (and not driven by market forces) against any foreign currency, it is called devaluation. It means official depreciation is devaluation. Revaluation is when a government increases the exchange rate of its currency against any foreign currency. It is official appreciation. Incorrect Solution (c) Statement 1 Statement 2 Statement 3 Statement 4 Correct Correct Incorrect Correct In the foreign exchange market, it is a situation when a domestic currency loses its value in front of a foreign currency if it is market-driven. It means depreciation in a currency can only take place if the economy follows the floating exchange rate system. If a free-floating domestic currency increases its value against the value of a foreign currency, it is appreciated. In the domestic economy, if a fixed asset has seen an increase in its value, it is also known as appreciation. Appreciation rates for different assets are not fixed by any government as they depend upon market mechanisms. When the exchange rate of a domestic currency is cut down by its government (and not driven by market forces) against any foreign currency, it is called devaluation. It means official depreciation is devaluation. Revaluation is when a government increases the exchange rate of its currency against any foreign currency. It is official appreciation. Question 15 of 34 15. Question Current account of the country includes: Export and Import. Interest payments Private remittances and transfers. Foreign currency deposits of banks Which of the above statements is/are correct? a) 1, 3 and 4 only b) 1, 2, 3 and 4 c) 1, 2 and 3 only d) 1 and 4 only Correct Solution (c) Foreign currency deposits of banks are included in the capital accounts. In the external sector, it refers to the account maintained by every government of the world in which every kind of current transaction is shown—basically this account is maintained by the central banking body of the economy on behalf of the government. Current transactions of an economy in foreign currency all over the world are—export, import, interest payments, private remittances, and transfers. All transactions are shown as either inflow or outflow (credit or debit). At the end of the year, the current account might be positive or negative. The positive one is known as a surplus current account, and the negative one is known as a deficit current account. Incorrect Solution (c) Foreign currency deposits of banks are included in the capital accounts. In the external sector, it refers to the account maintained by every government of the world in which every kind of current transaction is shown—basically this account is maintained by the central banking body of the economy on behalf of the government. Current transactions of an economy in foreign currency all over the world are—export, import, interest payments, private remittances, and transfers. All transactions are shown as either inflow or outflow (credit or debit). At the end of the year, the current account might be positive or negative. The positive one is known as a surplus current account, and the negative one is known as a deficit current account. Question 16 of 34 16. Question Which of the following transactions would be related to the BoP account of the economy? Joint production of helicopters by India and Japan. Transfer of technology by Russia to India. USA’s investment in Indian treasury bills. How many of the above statements are correct? a) Only one b) Only two c) All three d) None Correct Solution (c) Statement 1 Statement 2 Statement 3 Correct Correct Correct A joint production of helicopters by India and Japan would involve sharing and transferring of money and other resources across nations. Transfer of technology would also involve money and the transfer of other resources. It is essentially a BoP account transaction of the economy. Investment in treasury bills or government bonds of a country is a kind of Foreign Institutional Investment. So it will come under external sector transactions. Incorrect Solution (c) Statement 1 Statement 2 Statement 3 Correct Correct Correct A joint production of helicopters by India and Japan would involve sharing and transferring of money and other resources across nations. Transfer of technology would also involve money and the transfer of other resources. It is essentially a BoP account transaction of the economy. Investment in treasury bills or government bonds of a country is a kind of Foreign Institutional Investment. So it will come under external sector transactions. Question 17 of 34 17. Question Which of the following would lead to a depreciation of the Indian rupee? Increase in the import bill. Higher fiscal deficit. Increase in interest rates by US federal reserve. Select the correct answer using the codes given below: a) 1 only b) 1 and 2 only c) 1, 2 and 3 d) 2 and 3 only Correct Solution (c) Statement 1 Statement 2 Statement 3 Correct Correct Correct Currency depreciation is a fall in the value of a currency in a floating exchange rate system. The higher import bill could lead to the outflow of the dollar in the economy and increase the value of the dollar as compared to the Indian rupee in the market economy which could lead to the depreciation of the Indian rupee. A higher fiscal deficit would lead to a depreciation of the Indian rupee. A country running a large fiscal deficit is always at risk of seeing the value of the currency fall. If there are insufficient capital flows to finance the deficit, the exchange rate will fall to reflect the imbalance of foreign flows of funds. An increase in interest rates by US federal reserve leads to a depreciation of the Indian rupee. When the Federal Reserve increases the federal funds rate, it increases interest rates throughout the economy. The higher yields attract investment capital from investors abroad seeking higher returns on bonds and interest-rate products. This results in a stronger exchange rate in favor of the U.S. dollar and a weaker exchange rate in the favor of Indian rupee, which could lead to the depreciation of the Indian rupee. Incorrect Solution (c) Statement 1 Statement 2 Statement 3 Correct Correct Correct Currency depreciation is a fall in the value of a currency in a floating exchange rate system. The higher import bill could lead to the outflow of the dollar in the economy and increase the value of the dollar as compared to the Indian rupee in the market economy which could lead to the depreciation of the Indian rupee. A higher fiscal deficit would lead to a depreciation of the Indian rupee. A country running a large fiscal deficit is always at risk of seeing the value of the currency fall. If there are insufficient capital flows to finance the deficit, the exchange rate will fall to reflect the imbalance of foreign flows of funds. An increase in interest rates by US federal reserve leads to a depreciation of the Indian rupee. When the Federal Reserve increases the federal funds rate, it increases interest rates throughout the economy. The higher yields attract investment capital from investors abroad seeking higher returns on bonds and interest-rate products. This results in a stronger exchange rate in favor of the U.S. dollar and a weaker exchange rate in the favor of Indian rupee, which could lead to the depreciation of the Indian rupee. Question 18 of 34 18. Question Consider the following statements with reference to External commercial borrowings in India: ECB can be raised only in Indian Rupees It includes trade credits beyond three years. It includes commercial bank loans It includes Investment in the stock market How many of the above statements are correct? a) Only one b) Only two c) Only three d) All four Correct Solution (b) Statement 1 Statement 2 Statement 3 Statement 4 Incorrect Correct Correct Incorrect ECB can be raised in any freely convertible foreign currency as well as in Indian Rupees or any other currency as specified by the Reserve Bank of India in consultation with the Government of India. It includes trade credits beyond three years, floating/ fixed-rate notes/ bonds/ debentures (other than fully and compulsorily convertible instruments), loans including bank loans, financial leases, and plain vanilla Rupee denominated bonds issued overseas. ECBs include commercial bank loans, buyers’ credit, suppliers’ credit, securitized instruments such as floating rate notes and fixed rate bonds, etc., credit from official export credit agencies, and commercial borrowings from the private sector window of multilateral financial Institutions such as International Finance Corporation (Washington), ADB, AFIC, CDC, etc. ECBs cannot be used for investment in the stock market or speculation in real estate. The DEA (Department of Economic Affairs), Ministry of Finance, Government of India along with the Reserve Bank of India, monitors and regulates ECB guidelines and policies. Incorrect Solution (b) Statement 1 Statement 2 Statement 3 Statement 4 Incorrect Correct Correct Incorrect ECB can be raised in any freely convertible foreign currency as well as in Indian Rupees or any other currency as specified by the Reserve Bank of India in consultation with the Government of India. It includes trade credits beyond three years, floating/ fixed-rate notes/ bonds/ debentures (other than fully and compulsorily convertible instruments), loans including bank loans, financial leases, and plain vanilla Rupee denominated bonds issued overseas. ECBs include commercial bank loans, buyers’ credit, suppliers’ credit, securitized instruments such as floating rate notes and fixed rate bonds, etc., credit from official export credit agencies, and commercial borrowings from the private sector window of multilateral financial Institutions such as International Finance Corporation (Washington), ADB, AFIC, CDC, etc. ECBs cannot be used for investment in the stock market or speculation in real estate. The DEA (Department of Economic Affairs), Ministry of Finance, Government of India along with the Reserve Bank of India, monitors and regulates ECB guidelines and policies. Question 19 of 34 19. Question Which of the following is correct about the ‘impossible trinity? a) A country cannot maintain all three policy goals—a stable financial market, global integration and stable exchange rate. b) A country cannot maintain all three policy goals—free capital flows, a fixed exchange rate and an independent monetary policy. c) A country cannot maintain all three policy goals—stable exchange rate, global integration and continuous economic growth. d) A country cannot maintain all three policy goals—small fiscal deficits, social welfare and high economic growth. Correct Solution (b) The impossible trinity also called the Mundell-Fleming trilemma or simply the trilemma expresses the limited options available to countries in setting monetary policy. The impossible trinity (also known as the impossible trilemma or the Unholy Trinity) is a concept in international economics that states that it is impossible to have all three of the following at the same time: A fixed foreign exchange rate Free capital movement (absence of capital controls) An independent monetary policy According to the impossible trinity, a central bank can only pursue two of the above-mentioned three policies simultaneously Incorrect Solution (b) The impossible trinity also called the Mundell-Fleming trilemma or simply the trilemma expresses the limited options available to countries in setting monetary policy. The impossible trinity (also known as the impossible trilemma or the Unholy Trinity) is a concept in international economics that states that it is impossible to have all three of the following at the same time: A fixed foreign exchange rate Free capital movement (absence of capital controls) An independent monetary policy According to the impossible trinity, a central bank can only pursue two of the above-mentioned three policies simultaneously Question 20 of 34 20. Question Which one of the following groups of items is included in India’s foreign-exchange reserves? a) Foreign-currency assets, Special Drawing Rights (SDRs) and loans from foreign countries b) Foreign-currency assets, gold holdings of the RBI and SDRs c) Foreign-currency assets, loans from the World Bank and SDRs d) Foreign-currency assets, gold holdings of the RBI and loans from the World Bank Correct Solution (b) The Foreign Exchange Reserves of India consists of below four categories: Foreign Currency Assets Gold Special Drawing Rights (SDRs) Reserve Tranche Position Incorrect Solution (b) The Foreign Exchange Reserves of India consists of below four categories: Foreign Currency Assets Gold Special Drawing Rights (SDRs) Reserve Tranche Position Question 21 of 34 21. Question It is located in the Mandla and Balaghat districts of Madhya Pradesh. It is nestled in the Maikal range of Satpuras, the heart of India, that forms the central Indian highlands. It is characterized mainly by forested shallow undulations, and hills with varying degrees of slopes, plateaus, and valleys. It is the first tiger reserve in India to officially introduce a mascot, “Bhoorsingh the Barasingha”. It is the largest national park in Madhya Pradesh. The above paragraph describes which of the following? a) Pench National Park b) Bandhavgarh National Park c) Panna National Park d) Kanha National Park Correct Solution (d) Kanha National Park is located in the Mandla and Balaghat districts of Madhya Pradesh. It is nestled in the Maikal range of Satpuras, the heart of India, that forms the central Indian highlands. It is characterized mainly by forested shallow undulations, and hills with varying degrees of slopes, plateaus, and valleys. It is the first tiger reserve in India to officially introduce a mascot, “Bhoorsingh the Barasingha”. It is the largest national park in Madhya Pradesh. Hence option d is correct.   Incorrect Solution (d) Kanha National Park is located in the Mandla and Balaghat districts of Madhya Pradesh. It is nestled in the Maikal range of Satpuras, the heart of India, that forms the central Indian highlands. It is characterized mainly by forested shallow undulations, and hills with varying degrees of slopes, plateaus, and valleys. It is the first tiger reserve in India to officially introduce a mascot, “Bhoorsingh the Barasingha”. It is the largest national park in Madhya Pradesh. Hence option d is correct.   Question 22 of 34 22. Question Consider the following statements regarding the Kosi River: It is formed by the confluence of the Sun Kosi, the Arun Kosi, and the Tamur Kosi in the Himalayan region of Nepal and Tibet. It is a transboundary river that flows through China, Nepal, and India. It is known as the “Sorrow of Bihar”, as it has caused widespread human suffering in the past due to flooding. How many of the above statements are correct? a) Only one b) Only two c) All three d) None Correct Solution (c) The Kosi River is formed by the confluence of the Sun Kosi, the Arun Kosi, and the Tamur Kosi in the Himalayan region of Nepal and Tibet. Hence statement 1 is correct. About 30 miles (48 km) north of the Indian-Nepalese frontier, the Kosi is joined by several major tributaries and breaks southward through the Siwalik Hills at the narrow Chatra Gorge. The river then emerges on the great plain of northern India in Bihar state on its way to the Ganges River, which it enters south of Purnea after a course of about 450 miles (724 km). It has seven major tributaries – Sun Koshi, Tama Koshi or Tamba Koshi, Dudh Koshi, Indravati, Likhu, Arun, and Tamore or Tamar. It is a transboundary river that flows through China, Nepal, and India. Hence statement 2 is correct. It is a prominent tributary of the Ganges. It drains an area of 74,500 sq. km, of which only 11,070 sq. km lie within Indian Territory. The Kosi River valley is bounded by steep margins that disconnect it from the Yarlung Zangbo River to the north, the Mahananda River to the east, the Gandaki to the west, and the Ganga to the south. It is well known for its tendency to change course generally in a westward direction. It is known as the “Sorrow of Bihar”, as it has caused widespread human suffering in the past due to flooding. Hence statement 3 is correct. Incorrect Solution (c) The Kosi River is formed by the confluence of the Sun Kosi, the Arun Kosi, and the Tamur Kosi in the Himalayan region of Nepal and Tibet. Hence statement 1 is correct. About 30 miles (48 km) north of the Indian-Nepalese frontier, the Kosi is joined by several major tributaries and breaks southward through the Siwalik Hills at the narrow Chatra Gorge. The river then emerges on the great plain of northern India in Bihar state on its way to the Ganges River, which it enters south of Purnea after a course of about 450 miles (724 km). It has seven major tributaries – Sun Koshi, Tama Koshi or Tamba Koshi, Dudh Koshi, Indravati, Likhu, Arun, and Tamore or Tamar. It is a transboundary river that flows through China, Nepal, and India. Hence statement 2 is correct. It is a prominent tributary of the Ganges. It drains an area of 74,500 sq. km, of which only 11,070 sq. km lie within Indian Territory. The Kosi River valley is bounded by steep margins that disconnect it from the Yarlung Zangbo River to the north, the Mahananda River to the east, the Gandaki to the west, and the Ganga to the south. It is well known for its tendency to change course generally in a westward direction. It is known as the “Sorrow of Bihar”, as it has caused widespread human suffering in the past due to flooding. Hence statement 3 is correct. Question 23 of 34 23. Question Consider the following statements regarding the SOFIA (Stratospheric Observatory for Infrared Astronomy): It was a telescope mounted on a Boeing 747 SP aircraft that studied ultraviolet light. It is operated jointly by the National Aeronautics and Space Administration and the Indian Space Research Organisation. It is the world’s largest airborne astronomical observatory. How many of the above statements are correct? a) Only one b) Only two c) All three d) None Correct Solution (a) The SOFIA (Stratospheric Observatory for Infrared Astronomy) was a telescope mounted on a Boeing 747 SP aircraft that studied infrared light, essentially heat, emitted by objects in the universe. Hence statement 1 is incorrect. The observatory, fitted with an 8.9-foot-wide (2.7 meter) telescope with a nearly 20-ton mirror, used a door in the side of the aircraft to peer at the sky. The airplane is capable of ten-hour flights, with flight paths chosen to keep ahead of the sunrise and maximize the amount of darkness. Flying into the stratosphere at 38,000-45,000 feet put SOFIA above 99 percent of Earth’s infrared-blocking atmosphere, allowing astronomers to study the solar system and beyond in ways that are not possible with ground-based telescopes. It is operated jointly by the National Aeronautics and Space Administration and the German Space Agency. Hence statement 2 is incorrect. The observatory’s mobility allowed researchers to observe from almost anywhere in the world and enabled studies of transient events that often take place over oceans, where there are no telescopes. It is the world’s largest airborne astronomical observatory, complementing NASA’s space telescopes as well as major Earth-based telescopes. Hence statement 3 is correct. The SOFIA project was prematurely ended in 2022 after operating for 12 years. Incorrect Solution (a) The SOFIA (Stratospheric Observatory for Infrared Astronomy) was a telescope mounted on a Boeing 747 SP aircraft that studied infrared light, essentially heat, emitted by objects in the universe. Hence statement 1 is incorrect. The observatory, fitted with an 8.9-foot-wide (2.7 meter) telescope with a nearly 20-ton mirror, used a door in the side of the aircraft to peer at the sky. The airplane is capable of ten-hour flights, with flight paths chosen to keep ahead of the sunrise and maximize the amount of darkness. Flying into the stratosphere at 38,000-45,000 feet put SOFIA above 99 percent of Earth’s infrared-blocking atmosphere, allowing astronomers to study the solar system and beyond in ways that are not possible with ground-based telescopes. It is operated jointly by the National Aeronautics and Space Administration and the German Space Agency. Hence statement 2 is incorrect. The observatory’s mobility allowed researchers to observe from almost anywhere in the world and enabled studies of transient events that often take place over oceans, where there are no telescopes. It is the world’s largest airborne astronomical observatory, complementing NASA’s space telescopes as well as major Earth-based telescopes. Hence statement 3 is correct. The SOFIA project was prematurely ended in 2022 after operating for 12 years. Question 24 of 34 24. Question Consider the following statements about the Sangam: Digital Twin Initiative: It is a Proof of Concept (PoC) distributed in two stages to be conducted in one of the major cities of India. It is launched by the Department of Telecommunications (DoT). It symbolizes a collaborative leap towards reshaping infrastructure planning and design, combining the prowess of next-gen computational technologies. How many of the above statements are correct? a) Only one b) Only two c) All three d) None Correct Solution (c) The Sangam: Digital Twin Initiative is a Proof of Concept (PoC) distributed in two stages to be conducted in one of the major cities of India. Hence statement 1 is correct. The first stage is exploratory for clarity of horizon and creative exploration to unleash potential. The second stage is for practical demonstration of specific use cases generating a future blueprint that may serve as a roadmap to scale and replicate successful strategies in future infrastructure projects through collaboration. It is launched by the Department of Telecommunications (DoT). Hence statement 2 is correct. The initiative aims to demonstrate the practical implementation of innovative infrastructure planning solutions, to develop a model framework for facilitating faster and more effective collaboration, and to provide a future blueprint that may serve as a roadmap to scale and replicate successful strategies in future infrastructure projects. It symbolizes a collaborative leap towards reshaping infrastructure planning and design, combining the prowess of 5G, IoT, AI, AR/VR, AI native 6G, Digital Twin, and next-gen computational technologies with the collective intelligence of public entities, infrastructure planners, tech giants, start-ups, and academia to break the silos and engage in a whole-of-nation approach. Hence statement 3 is correct. Sangam brings all stakeholders on one platform aiming to transform innovative ideas into tangible solutions, bridging the gap between conceptualization and realization, ultimately paving the way for ground-breaking infrastructure advancements. Incorrect Solution (c) The Sangam: Digital Twin Initiative is a Proof of Concept (PoC) distributed in two stages to be conducted in one of the major cities of India. Hence statement 1 is correct. The first stage is exploratory for clarity of horizon and creative exploration to unleash potential. The second stage is for practical demonstration of specific use cases generating a future blueprint that may serve as a roadmap to scale and replicate successful strategies in future infrastructure projects through collaboration. It is launched by the Department of Telecommunications (DoT). Hence statement 2 is correct. The initiative aims to demonstrate the practical implementation of innovative infrastructure planning solutions, to develop a model framework for facilitating faster and more effective collaboration, and to provide a future blueprint that may serve as a roadmap to scale and replicate successful strategies in future infrastructure projects. It symbolizes a collaborative leap towards reshaping infrastructure planning and design, combining the prowess of 5G, IoT, AI, AR/VR, AI native 6G, Digital Twin, and next-gen computational technologies with the collective intelligence of public entities, infrastructure planners, tech giants, start-ups, and academia to break the silos and engage in a whole-of-nation approach. Hence statement 3 is correct. Sangam brings all stakeholders on one platform aiming to transform innovative ideas into tangible solutions, bridging the gap between conceptualization and realization, ultimately paving the way for ground-breaking infrastructure advancements. Question 25 of 34 25. Question Consider the following statements about the Gulf Stream: It is a cold ocean current that flows along the eastern coast of North America. It carries warm water from the tropics to higher latitudes. It keeps the coastal areas warmer in winter and cooler in summer compared to inland regions at the same latitudes on the eastern coast of North America. How many of the above statements are correct? a) Only one b) Only two c) All three d) None Correct Solution (b) The Gulf Stream is a warm ocean current that flows along the eastern coast of North America. Hence statement 1 is incorrect. It originates in the Gulf of Mexico. It then travels northward along the eastern coast of the United States. It follows a north-eastward path across the western North Atlantic Ocean. The two equatorial sources of the Gulf Stream are the North Equatorial Current (NEC), which flows generally westward along the Tropic of Cancer, and the South Equatorial Current (SEC), which flows westward from southwestern Africa to South America and then northward to the Caribbean Sea. Together, these two warm currents, along with waters from the Gulf of Mexico, form the Gulf Stream. It carries warm water from the tropics (around 25 to 28°C or 77 to 82°F) to higher latitudes. Hence statement 2 is correct. The Gulf Stream is several hundred kilometres wide and can flow at an average speed of about four miles per hour (6.4 kilometers per hour). However, its speed can vary depending on the location and other factors. The current is also very deep, extending to depths of up to 1,000 meters. It moderates the temperatures along the eastern coast of North America, keeping the coastal areas warmer in winter and cooler in summer compared to inland regions at the same latitudes. Since the Gulf Stream also extends toward Europe, it warms Western European countries as well. Hence statement 3 is correct. The warm and moist air above the Gulf Stream can lead to the formation of low-pressure systems, which may develop into storms or hurricanes. It can also contribute to the formation of fog in certain areas. Incorrect Solution (b) The Gulf Stream is a warm ocean current that flows along the eastern coast of North America. Hence statement 1 is incorrect. It originates in the Gulf of Mexico. It then travels northward along the eastern coast of the United States. It follows a north-eastward path across the western North Atlantic Ocean. The two equatorial sources of the Gulf Stream are the North Equatorial Current (NEC), which flows generally westward along the Tropic of Cancer, and the South Equatorial Current (SEC), which flows westward from southwestern Africa to South America and then northward to the Caribbean Sea. Together, these two warm currents, along with waters from the Gulf of Mexico, form the Gulf Stream. It carries warm water from the tropics (around 25 to 28°C or 77 to 82°F) to higher latitudes. Hence statement 2 is correct. The Gulf Stream is several hundred kilometres wide and can flow at an average speed of about four miles per hour (6.4 kilometers per hour). However, its speed can vary depending on the location and other factors. The current is also very deep, extending to depths of up to 1,000 meters. It moderates the temperatures along the eastern coast of North America, keeping the coastal areas warmer in winter and cooler in summer compared to inland regions at the same latitudes. Since the Gulf Stream also extends toward Europe, it warms Western European countries as well. Hence statement 3 is correct. The warm and moist air above the Gulf Stream can lead to the formation of low-pressure systems, which may develop into storms or hurricanes. It can also contribute to the formation of fog in certain areas. Question 26 of 34 26. Question Consider the following statements regarding the Young Scientist Programme (YUVIKA): It is a learning and awareness-creating programme regarding space science by the Ministry of Education. It encourages more students to pursue Science, Technology, Engineering, and Mathematics (STEM) based careers. Students who have finished class 5 and are currently studying in class 6 can apply for the programme. How many of the above statements are correct? a) Only one b) Only two c) All three d) None Correct Solution (a) The Young Scientist Programme (YUVIKA) is a learning and awareness-creating programme regarding space science by the Indian Space Research Organisation(ISRO). Hence statement 1 is incorrect. It aims to impart basic knowledge of space technology, space science, and space applications to younger students with a preference for rural areas. It is a two-week residential programme offered by ISRO. It will include invited talks, experience sharing by eminent scientists, facility and lab visits, exclusive sessions for discussions with experts, and practical and feedback sessions. It encourages more students to pursue Science, Technology, Engineering, and Mathematics (STEM) based careers. Hence statement 2 is correct. It is aimed at creating awareness about the emerging trends in science and technology amongst the youngsters, who are the future building blocks of our nation. ISRO has chalked out this programme to “Catch them young”. Students who have finished class 8 and are currently studying in class 9 can apply for the programme. Hence statement 3 is incorrect. Three students from each state/Union Territory will participate in this programme every year, covering CBSE, ICSE, and state-board syllabus. The selection is based on 8th Standard academic performance and extracurricular activities. Students belonging to the rural areas have been given special weightage in the selection criteria. Incorrect Solution (a) The Young Scientist Programme (YUVIKA) is a learning and awareness-creating programme regarding space science by the Indian Space Research Organisation(ISRO). Hence statement 1 is incorrect. It aims to impart basic knowledge of space technology, space science, and space applications to younger students with a preference for rural areas. It is a two-week residential programme offered by ISRO. It will include invited talks, experience sharing by eminent scientists, facility and lab visits, exclusive sessions for discussions with experts, and practical and feedback sessions. It encourages more students to pursue Science, Technology, Engineering, and Mathematics (STEM) based careers. Hence statement 2 is correct. It is aimed at creating awareness about the emerging trends in science and technology amongst the youngsters, who are the future building blocks of our nation. ISRO has chalked out this programme to “Catch them young”. Students who have finished class 8 and are currently studying in class 9 can apply for the programme. Hence statement 3 is incorrect. Three students from each state/Union Territory will participate in this programme every year, covering CBSE, ICSE, and state-board syllabus. The selection is based on 8th Standard academic performance and extracurricular activities. Students belonging to the rural areas have been given special weightage in the selection criteria. Question 27 of 34 27. Question Consider the following statements regarding the European Free Trade Association: It is an intergovernmental organisation established by the Stockholm Convention. Its member countries are Iceland, Liechtenstein, Norway, and Switzerland. It aims to promote free trade and economic integration between its members within Europe and globally. How many of the above statements are correct? a) Only one b) Only two c) All three d) None Correct Solution (c) The European Free Trade Association is an intergovernmental organisation established in 1960 by the Stockholm Convention. Hence statement 1 is correct. The members of this organization are all open, competitive economies committed to the progressive liberalisation of trade in the multinational arena as well as in free trade agreements. Its member countries are Iceland, Liechtenstein, Norway, and Switzerland. Hence statement 2 is correct. Its highest governing body is the EFTA Council. It generally meets 8 times a year at the ambassadorial level and twice a year at the ministerial level. EFTA Surveillance Authority (ESA) monitors compliance with European Economic Area (EEA) rules in Iceland, Liechtenstein, and Norway. EFTA Court is based in Luxembourg and has the competence and authority to settle internal and external disputes regarding the implementation, application, or interpretation of the EEA agreement. It aims to promote free trade and economic integration between its members within Europe and globally. Hence statement 3 is correct. Incorrect Solution (c) The European Free Trade Association is an intergovernmental organisation established in 1960 by the Stockholm Convention. Hence statement 1 is correct. The members of this organization are all open, competitive economies committed to the progressive liberalisation of trade in the multinational arena as well as in free trade agreements. Its member countries are Iceland, Liechtenstein, Norway, and Switzerland. Hence statement 2 is correct. Its highest governing body is the EFTA Council. It generally meets 8 times a year at the ambassadorial level and twice a year at the ministerial level. EFTA Surveillance Authority (ESA) monitors compliance with European Economic Area (EEA) rules in Iceland, Liechtenstein, and Norway. EFTA Court is based in Luxembourg and has the competence and authority to settle internal and external disputes regarding the implementation, application, or interpretation of the EEA agreement. It aims to promote free trade and economic integration between its members within Europe and globally. Hence statement 3 is correct. Question 28 of 34 28. Question Consider the following statements regarding the Cash Reserve Ratio (CRR): It is the percentage of cash required to be kept in reserves with the Reserve Bank of India as against the bank’s total deposits. The bank can use CRR for lending and investment purposes and gets interest from the RBI. In a high inflation environment, the RBI can increase CRR which helps to control inflation. How many of the above statements are correct? a) Only one b) Only two c) All three d) None Correct Solution (b) The Cash Reserve Ratio (CRR) is the percentage of cash required to be kept in reserves with the Reserve Bank of India as against the bank’s total deposits. Hence statement 1 is correct. Under CRR, commercial banks have to hold a certain minimum amount of deposits as reserves with the RBI. The RBI decides the CRR and is kept with them for financial security. The bank cannot use CRR for lending and investment purposes and does not get any interest from the RBI. Hence statement 2 is incorrect. The CRR applies to scheduled commercial banks, while regional rural banks and NBFCs are excluded. In a high inflation environment, the RBI can increase CRR which helps to control inflation. Hence statement 3 is correct. It ensures banks have a minimum amount of funds readily available to customers, even during huge demand. CRR serves as the reference rate for loans. Also known as the base rate for loans, banks cannot offer loans below this rate. Incorrect Solution (b) The Cash Reserve Ratio (CRR) is the percentage of cash required to be kept in reserves with the Reserve Bank of India as against the bank’s total deposits. Hence statement 1 is correct. Under CRR, commercial banks have to hold a certain minimum amount of deposits as reserves with the RBI. The RBI decides the CRR and is kept with them for financial security. The bank cannot use CRR for lending and investment purposes and does not get any interest from the RBI. Hence statement 2 is incorrect. The CRR applies to scheduled commercial banks, while regional rural banks and NBFCs are excluded. In a high inflation environment, the RBI can increase CRR which helps to control inflation. Hence statement 3 is correct. It ensures banks have a minimum amount of funds readily available to customers, even during huge demand. CRR serves as the reference rate for loans. Also known as the base rate for loans, banks cannot offer loans below this rate. Question 29 of 34 29. Question Consider the following statements regarding Diphtheria: It is a serious contagious fungal infection of the nose and throat. It can spread from person to person, usually through respiratory droplets. It is treated by neutralization of unbound toxin with Diphtheria Antitoxin (DAT). How many of the above statements are correct? a) Only one b) Only two c) All three d) None Correct Solution (b) Diphtheria is a serious contagious bacterial infection of the nose and throat. Hence statement 1 is incorrect. It is caused by strains of bacteria called Corynebacterium diphtheriae that make a toxin. Its symptoms include a thick, gray membrane covering the throat and tonsils, sore throat and hoarseness, swollen glands (enlarged lymph nodes) in the neck, difficulty breathing, etc. It can spread from person to person, usually through respiratory droplets. Hence statement 2 is correct. People can also get sick from touching infected open sores or ulcers. The bacteria can also infect the skin, causing open sores or ulcers. However, diphtheria skin infections rarely result in severe disease. It is treated by neutralization of unbound toxin with Diphtheria Antitoxin (DAT). Hence statement 3 is correct. Although diphtheria can be treated with medications, in advanced stages, the bacterial infection can damage the heart, kidneys, and nervous system.   Incorrect Solution (b) Diphtheria is a serious contagious bacterial infection of the nose and throat. Hence statement 1 is incorrect. It is caused by strains of bacteria called Corynebacterium diphtheriae that make a toxin. Its symptoms include a thick, gray membrane covering the throat and tonsils, sore throat and hoarseness, swollen glands (enlarged lymph nodes) in the neck, difficulty breathing, etc. It can spread from person to person, usually through respiratory droplets. Hence statement 2 is correct. People can also get sick from touching infected open sores or ulcers. The bacteria can also infect the skin, causing open sores or ulcers. However, diphtheria skin infections rarely result in severe disease. It is treated by neutralization of unbound toxin with Diphtheria Antitoxin (DAT). Hence statement 3 is correct. Although diphtheria can be treated with medications, in advanced stages, the bacterial infection can damage the heart, kidneys, and nervous system.   Question 30 of 34 30. Question Mutualism is an association between organisms of two different species in which each benefit. Which of the following exhibit mutualism? a) Penicillium and Staphylococcus b) Orchids growing on branches c) Golden-backed frog and Bonnet mushroom d) Grazing cattle and Insects Correct Solution (c) Mutualism is an association between organisms of two different species in which each benefit. It is exhibited by a Golden-backed frog and Bonnet mushroom. Hence option c is correct. Golden-backed Frog is endemic to the Western Ghats of Karnataka and Kerala. They are primarily insectivorous and feed on a range of small insects and arthropods such as ants, beetles, and crickets. Bonnet mushroom is commonly found as a saprotroph (an organism that feeds on non-living organic matter) on rotting wood. It belongs to the Mycena genus. Incorrect Solution (c) Mutualism is an association between organisms of two different species in which each benefit. It is exhibited by a Golden-backed frog and Bonnet mushroom. Hence option c is correct. Golden-backed Frog is endemic to the Western Ghats of Karnataka and Kerala. They are primarily insectivorous and feed on a range of small insects and arthropods such as ants, beetles, and crickets. Bonnet mushroom is commonly found as a saprotroph (an organism that feeds on non-living organic matter) on rotting wood. It belongs to the Mycena genus. Question 31 of 34 31. Question What is the missing number ‘X’ of the series 7, 11, 19, 35, X? a) 45 b) 67 c) 57 d) 78 Correct Solution (b) Second number of series = [(2 × first number)− 3] 11 = (2 × 7)− 3 19 = (2 × 11) − 3 35 = (2 × 19) − 3 X = (2 × 35)− 3 = 70 − 3 = 67 Incorrect Solution (b) Second number of series = [(2 × first number)− 3] 11 = (2 × 7)− 3 19 = (2 × 11) − 3 35 = (2 × 19) − 3 X = (2 × 35)− 3 = 70 − 3 = 67 Question 32 of 34 32. Question A man purchased 50 pens for Rs. 3000, and sold 15 of them at a gain of 5%. He wants to sell the remaining so as to gain 15% on the whole. What should the approximate selling price of each pen be? a) 90 b) 72 c) 66 d) 78 Correct Solution (b)                         (x – 15) : 10 = 15 : 35 7x – 105 = 30 X = 20 Required selling price = 60 * (120/100 ) = 72 Incorrect Solution (b)                         (x – 15) : 10 = 15 : 35 7x – 105 = 30 X = 20 Required selling price = 60 * (120/100 ) = 72 Question 33 of 34 33. Question The monthly average salary paid to all the employees of a company was Rs. 12000. The monthly average salary paid to male and female employees was Rs. 14000 and Rs. 10000 respectively. Then the percentage of males employed in the company is:   a) 40% b) 60% c) 50% d) 80% Correct Solution (c) Let total no of the male employee be x Total no of the female be y Then total salary of all male = 14000 × x The total salary of all female =10000 × y But as per question total salary of all the employee = 12000(x + y) So [(14000 × x)+ (10000 × y)] = 12000(x + y) By equating we will find x/y = 1/1 So the percentage of female employee= 1 2 × 100 = 50%   Incorrect Solution (c) Let total no of the male employee be x Total no of the female be y Then total salary of all male = 14000 × x The total salary of all female =10000 × y But as per question total salary of all the employee = 12000(x + y) So [(14000 × x)+ (10000 × y)] = 12000(x + y) By equating we will find x/y = 1/1 So the percentage of female employee= 1 2 × 100 = 50%   Question 34 of 34 34. Question If CAT = 60, AS = 19, then BED will be equal to a) 30 b) 15 c) 40 d) 14 Correct Solution (c) Taking A = 1, B = 2, C = 3…., T = 20,….., Z = 26, We have: CAT = C × A x T = 3 × 1 x 20 = 20. AS = A x S = 1 x 19 = 19. Similarly, BED = B x E x D = 2 x 5 x 4 = 40. Incorrect Solution (c) Taking A = 1, B = 2, C = 3…., T = 20,….., Z = 26, We have: CAT = C × A x T = 3 × 1 x 20 = 20. AS = A x S = 1 x 19 = 19. Similarly, BED = B x E x D = 2 x 5 x 4 = 40. window.wpProQuizInitList = window.wpProQuizInitList || []; window.wpProQuizInitList.push({ id: '#wpProQuiz_3673', init: { quizId: 3673, mode: 1, globalPoints: 68, timelimit: 1800, resultsGrade: [0], bo: 704, qpp: 0, catPoints: [68], formPos: 0, lbn: "Test-summary", json: {"32942":{"type":"single","id":32942,"catId":0,"points":2,"correct":[0,0,1,0]},"32943":{"type":"single","id":32943,"catId":0,"points":2,"correct":[0,0,1,0]},"32944":{"type":"single","id":32944,"catId":0,"points":2,"correct":[0,0,1,0]},"32945":{"type":"single","id":32945,"catId":0,"points":2,"correct":[0,1,0,0]},"32948":{"type":"single","id":32948,"catId":0,"points":2,"correct":[0,1,0,0]},"32950":{"type":"single","id":32950,"catId":0,"points":2,"correct":[0,0,1,0]},"32953":{"type":"single","id":32953,"catId":0,"points":2,"correct":[1,0,0,0]},"32956":{"type":"single","id":32956,"catId":0,"points":2,"correct":[1,0,0,0]},"32959":{"type":"single","id":32959,"catId":0,"points":2,"correct":[0,0,1,0]},"32962":{"type":"single","id":32962,"catId":0,"points":2,"correct":[0,1,0,0]},"32964":{"type":"single","id":32964,"catId":0,"points":2,"correct":[0,0,0,1]},"32966":{"type":"single","id":32966,"catId":0,"points":2,"correct":[0,0,1,0]},"32967":{"type":"single","id":32967,"catId":0,"points":2,"correct":[0,1,0,0]},"32968":{"type":"single","id":32968,"catId":0,"points":2,"correct":[0,0,1,0]},"32969":{"type":"single","id":32969,"catId":0,"points":2,"correct":[0,0,1,0]},"32970":{"type":"single","id":32970,"catId":0,"points":2,"correct":[0,0,1,0]},"32973":{"type":"single","id":32973,"catId":0,"points":2,"correct":[0,0,1,0]},"32974":{"type":"single","id":32974,"catId":0,"points":2,"correct":[0,1,0,0]},"32975":{"type":"single","id":32975,"catId":0,"points":2,"correct":[0,1,0,0]},"32976":{"type":"single","id":32976,"catId":0,"points":2,"correct":[0,1,0,0]},"32979":{"type":"single","id":32979,"catId":0,"points":2,"correct":[0,0,0,1]},"32980":{"type":"single","id":32980,"catId":0,"points":2,"correct":[0,0,1,0]},"32982":{"type":"single","id":32982,"catId":0,"points":2,"correct":[1,0,0,0]},"32983":{"type":"single","id":32983,"catId":0,"points":2,"correct":[0,0,1,0]},"32985":{"type":"single","id":32985,"catId":0,"points":2,"correct":[0,1,0,0]},"32986":{"type":"single","id":32986,"catId":0,"points":2,"correct":[1,0,0,0]},"32989":{"type":"single","id":32989,"catId":0,"points":2,"correct":[0,0,1,0]},"32991":{"type":"single","id":32991,"catId":0,"points":2,"correct":[0,1,0,0]},"32994":{"type":"single","id":32994,"catId":0,"points":2,"correct":[0,1,0,0]},"32995":{"type":"single","id":32995,"catId":0,"points":2,"correct":[0,0,1,0]},"32996":{"type":"single","id":32996,"catId":0,"points":2,"correct":[0,1,0,0]},"32999":{"type":"single","id":32999,"catId":0,"points":2,"correct":[0,1,0,0]},"33002":{"type":"single","id":33002,"catId":0,"points":2,"correct":[0,0,1,0]},"33005":{"type":"single","id":33005,"catId":0,"points":2,"correct":[0,0,1,0]}} } }); All the Best IASbaba

DAILY CURRENT AFFAIRS IAS | UPSC Prelims and Mains Exam – 24th April 2024

Archives (PRELIMS & MAINS Focus)   Sea Level Rise Syllabus Prelims & Mains – Environment Context: A quarter of the coastal land of China will sink below sea level within a century, putting hundreds of millions of people at risk of being flooded, due to land subsidence and climate change, according to a new study, published in the journal Science on Friday (April 19). Background:- The land subsidence in China is primarily a result of rapid urbanisation — activities such as excessive groundwater extraction and the weight of buildings lead to subsidence. The situation has been exacerbated by climate change, which is resulting in the rise of sea levels. Sea level rise isn’t just limited to China. It is a global issue that is projected to affect between 130 million to half a billion people depending on different temperature scenarios in the long run, according to a UN report. How fast is the global sea level rising? The global sea level has risen by about 8–9 inches or 21–24 centimetres since 1880, according to a report by the National Oceanic and Atmospheric Administration (NOAA). The more worrying sign is that since 1993, the rate of increase has been accelerating. It has more than doubled from 0.07 inches or 0.18 centimetres per year in 1993 to the current rate of 0.17 inches or 0.42 centimetres per year. Global average sea level rose by about 0.3 inches or 0.76 centimetres from 2022 to 2023. Experts said there was a huge jump in global sea level between the two years mainly due to the development of El Niño, a weather pattern associated with warmer oceans, in June 2023. Now that El Niño is weakening, the rate of sea level rise is expected to slow down. How climate change is leading to sea level rise? The key driver behind rising sea levels is global warming. As temperatures across the world continue to soar, more and more glaciers and ice sheets are melting and adding water to the ocean. Global warming is also making the ocean warmer, which is leading to something called thermal expansion — when water becomes warmer, it expands in volume. This is contributing to sea level rise as well. Why does sea level rise matter? The most visible consequence of sea level rise will be the flooding of coastal areas. Take the example of coastal cities in India. A 2022 analysis by RMSI, a global risk management firm, found that some critical properties and road networks in Mumbai, Kochi, Mangalore, Chennai, Visakhapatnam, and Thiruvananthapuram will be submerged by 2050. Things are worse for island countries like Indonesia. In 2019, Indonesia announced that the country’s capital would be relocated from Jakarta, which is prone to flooding, to the province of East Kalimantan on the island of Borneo. The rise of sea level also leads to more intense storm surges — the rise in seawater level caused solely by a storm — flooding, and damage to coastal areas. That’s because higher sea level can allow more water inland during hurricane-related storm surges. With sea level rise, saltwater contaminates freshwater aquifers, many of which are used by communities for agricultural and drinking purposes. Source: Indian Express Earth Day Syllabus Prelims – Environment Context: Every year, April 22 is observed as Earth Day, an international event celebrated around the world to pledge support for environmental protection. Background: The year 2024 marks the 54th anniversary of the annual celebrations. About Earth DAY : Earth Day was first observed in 1970 when 20 million took to the streets to protest against environmental degradation. The event was triggered by the 1969 Santa Barbara oil spill and other issues such as smog and polluted rivers. For over the next half century, Earth Day continued to play an important role in environmental activism. The landmark Paris Agreement, which brings almost 200 countries together to set a common target to reduce global greenhouse emissions, was signed on Earth Day 2016. In 2009, the United Nations designated Earth Day as International Mother Earth Day. According to earthday.org,Earth Day aims to “build the world’s largest environmental movement to drive transformative change for people and the planet.” The movement’s mission is “to diversify, educate and activate the environmental movement worldwide.” Earth Day is celebrated to remind each of us that the Earth and its ecosystems provide us with life and sustenance. This day also recognises a collective responsibility, as called for in the 1992 Rio Declaration, to promote harmony with nature and the Earth to achieve a just balance among the economic, social and environmental needs of present and future generations of humanity. This year, the theme is ‘Planet vs. Plastics’. The theme highlights the threat plastics pose to the planet and human health. It calls for an end to all plastic waste by demanding a 60% reduction in the production of plastics by the year 2040. Source: Indian Express Iran- Israel Conflict Syllabus Prelims & Mains – International Event Context: Iran launched a significant attack on Israel, deploying over 300 projectiles, including approximately 170 drones, cruise missiles, and over 120 ballistic missiles. Background: Tit-for-tat confrontation between Israel and Iran has sparked concerns about escalation dragging the Middle East into all-out war. What led Iran to attack Israel? In 2018, Israel hails US withdrawal from Iran’s nuclear deal with world powers after years of lobbying against the agreement, calling Trump’s decision “a historic move”. In 2020, Israel welcomed the assassination of General Qassem Soleimani, commander of the overseas arm of Iran’s Revolutionary Guards, in an American drone strike in Baghdad. Iran strikes back with missile attacks on Iraqi bases housing American troops. In October 2023, Hamas (the Iran-backed terrorist group) launched a missile attack targeting Israel. In response, Israel conducted airstrikes on Gaza. In November 2023, Israel started conducting raids and attacking medical facilities as Hamas was reportedly operating from these hospital buildings and carrying out their warfare. In November 2023, The Iran-backed Houthi group, based in Yemen, landed their helicopter on the Galaxy Leader cargo vessel as it was passing via the Red Sea. This marked the start of the ‘Red Sea Crisis,’ which eventually led to supply chain issues. In December 2023, Israel’s ground offenses in the Gaza Strip intensified at a rapid pace. This led to an increase in the death toll and the number of refugees. India urged for an ‘early and durable resolution’ between the warring nations. A suspected Israeli air strike on the Iranian embassy compound in Damascus kills seven officers of the Islamic Revolutionary Guard, including two senior commanders. Israel neither confirmed nor denied responsibility. In April 2024, Iran launched a missile attack on Israel. The attack was, allegedly, in response to a suspected Israeli strike on Iran’s consulate in Syria. This marked the first instance of Iran directly targeting Israel from its domestic territory. The Israel Defence Forces (IDF) claimed that the Israeli air defence system intercepted 99% of the incoming projectiles from Iran. The United States, the United Kingdom, France, and other Middle Eastern allies also helped defend Israel. Impact of the Iran-Israel War on the world: Given the widely held Israeli belief that a nuclear-armed Iran is an existential threat to Israel, retaliation cannot be ruled out of the decision-making matrix. Failure of diplomatic efforts to de-escalate tensions or negotiate a peaceful resolution could leave military action as the only option, increasing the likelihood of regional escalation. Iran is the third-largest producer of crude oil within OPEC (Organization of the Petroleum Exporting Countries). If the tensions between Iran and Israel escalate further, the supply of crude oil prices will be severely disrupted. This will hit Indian share market sentiment as India is the third-largest consumer and importer of crude oil, importing over 80% of its crude oil needs. If geopolitical tensions increase from hereon, commodity prices will move up due to supply disruptions. Globally, inflation will remain high due to geopolitical tension as it will affect crude oil prices and other commodity prices such as copper, zinc, aluminium, nickel, etc. As a result of these concerns, investors are expected to become cautious and may move their money out of riskier assets like Indian stocks and into safer options like gold (bullion). Apart from oil prices being affected, trade and travel could also be hit by the likelihood of an Israel-Iran all-out war. Aviation and shipping sectors could be disrupted. Several countries in the region, including Iran, Jordan, Iraq, Lebanon, and Israel, temporarily closed their airspaces, opening it later, albeit with restrictions. India’s long-standing strategic relationships with both Iran and Israel present challenges on both policy and operational fronts. India values its strategic partnership with Israel, which includes defence cooperation, technology exchange, and intelligence sharing. However, India also maintains historical and economic ties with Iran, including energy imports and infrastructure projects. India seeks to maintain stability in the Middle East to safeguard its interests, including energy security and the welfare of its diaspora. Way Forward: Israel should accept a sustainable ceasefire in Gaza as soon as possible, open the borders for international humanitarian aid to Gaza, and respect the UN resolutions to end the 70-year-old crisis by realizing a two-state solution. The two-state solution is the only feasible way ahead for long-term security, peace, and stability in the region. It is not an easy goal, but both sides are familiar with the challenges and opportunities. An international initiative must mediate a sustainable ceasefire between Israel and Iran. Encouraging both countries to engage in direct talks facilitated by international mediators could help build trust and find common ground. Iran and Israel could engage in direct talks facilitated by a neutral third party, such as the European Union or the United Nations. Iran could adhere to the terms of the Joint Comprehensive Plan of Action (JCPOA) and allow international inspections of its nuclear facilities to ensure compliance with the agreement. In return, Israel could recognise Iran’s right to peaceful nuclear energy and commit to refraining from military strikes against Iranian nuclear facilities. Promoting cooperation between Iran and Israel within the framework of regional organizations, such as the Arab League or the Gulf Cooperation Council, could help address shared security concerns and foster stability in the Middle East. Regional powers could work together to establish a comprehensive security architecture for the Middle East, including confidence-building measures, arms control agreements, and mechanisms for resolving conflicts peacefully. Addressing underlying issues, such as historical grievances, territorial disputes, and religious extremism, can help create an environment conducive to peace and reconciliation. Iran and Israel could take steps towards normalizing diplomatic relations, such as exchanging ambassadors, reopening embassies, and facilitating people-to-people exchanges, similar to the peace agreements between Israel and some Arab states like the United Arab Emirates and Bahrain. Source: AP CHIPKO MOVEMENT Syllabus Prelims – Environment Context: The Chipko Movement, which began in the Uttarakhand region of the Himalayas in early 1973, has now reached its 50th anniversary. Background: The Chipko Movement’s tireless efforts and commitment to the environment continue to inspire new generations of activists. About CHIPKO MOVEMENT : The Chipko Movement, also known as the Chipko Andolan, was a non-violent environmental movement that originated in Uttarakhand (formerly part of Uttar Pradesh) in 1973. The name “Chipko” derives from Hindi, signifying “to hug.” During the movement, villagers embraced trees to prevent them from being cut down by loggers. The movement aimed to protect the Himalayan range trees from the axes of builders and commercial interests. The Chipko Movement is best remembered for the active participation of women in preserving forests. It changed attitudes regarding women’s status in society and highlighted their role in environmental conservation. Sunderlal Bahuguna, a Gandhian and environmentalist, played a pivotal role in the Chipko Movement. He coined the famous Chipko slogan: “Ecology is permanent economy.” Major Achievements: Rights to Forests: The movement made people aware of their rights to forests and demonstrated how grassroots activism could influence policy-making regarding ecology and shared natural resources. Ban on Commercial Felling: In 1981, the movement led to a ban on commercial tree felling above 30 degrees slope and above 1,000 meters above mean sea level (msl). Source: Down To Earth Previous Year Question Q) Chipko movement was basically against Water pollution Noise pollution Deforestation Cultural pollution RASHTRIYA AROGYA NIDHI (RAN) SCHEME Syllabus Prelims – Current Event Context: The Delhi High Court has raised concerns about the low-income threshold for financial assistance under the Rashtriya Arogya Nidhi (RAN) scheme. Background: The court’s observation highlights the need for a re-evaluation of the income threshold to ensure equitable access to healthcare assistance for those in need. About RASHTRIYA AROGYA NIDHI (RAN) SCHEME: The umbrella scheme of Rashtriya Arogya Nidhi (RAN) is a central sector scheme designed to provide financial assistance to poor patients living below the poverty line. It provides one-time financial assistance to poor patients living below State/UT wise threshold poverty line and suffering from life threatening diseases relating to heart, kidney, liver, cancer, etc. for treatment at any of Super Specialty Government hospitals/institutes. RAN offers financial support for treatments such as organ transplants, cancer treatment, and other critical procedures. The assistance is provided as a one-time grant to eligible patients. The Umbrella Scheme of RAN has three components as under: Rashtriya Arogya Nidhi (RAN) – Financial assistance for treatment of life-threatening diseases relating to heart, kidney, liver, etc. at Government hospitals/institutes having Super Specialty facilities; (Maximum financial assistance is Rs. 15 lakhs) Health Minister’s Cancer Patient Fund (HMCPF) – Financial assistance for treatment of cancer at Regional Cancer Centres (RCCs)/ Tertiary Care Cancer Centres (TCCCs) and State Cancer Institutes (SCIs); (Maximum financial assistance is Rs. 15 lakhs) Financial assistance for poor patients suffering from rare diseases – for specified rare diseases for treatment at Government hospitals/institutes having Super Specialty facilities; (Maximum financial assistance is Rs. 20 lakhs) Source: PIB NATIONAL SERVICE SCHEME (NSS) Syllabus Prelims – Current Event Context: Trained National Service Scheme (NSS) volunteers from 153 NSS units are stepping up to make a difference in Kozhikode district during the upcoming polls. Background: These dedicated volunteers will extend their helping hands to differently abled individuals and senior citizens, ensuring their well-being and facilitating their participation in the democratic process. About NATIONAL SERVICE SCHEME (NSS): The National Service Scheme (NSS) is a Central Sector Scheme implemented by the Government of India, under the Ministry of Youth Affairs & Sports. It aims to provide an opportunity for student youth across different educational levels to participate in various government-led community service activities and programs. NSS encourages students to actively engage in community service while pursuing their education. NSS was launched in 1969, during the birth centenary year of Mahatma Gandhi, involving 40,000 students across 37 universities. NSS promotes the development of students’ personality and character through voluntary community service. Source: Hindu Practice MCQs Daily Practice MCQs Q1.) With reference to the National Service Scheme (NSS), consider the following statements: The National Service Scheme (NSS) is a Central Sector Scheme under the Ministry of Youth Affairs & Sports. NSS was launched during the birth centenary year of Jawaharlal Nehru. It aims to provide an opportunity for student youth across different educational levels to participate in various government-led community service activities and programs. How many of the statements given above are correct? Only one Only two All three None Q2.) With reference to the Rashtriya Arogya Nidhi (RAN) scheme, consider the following statements: The Rashtriya Arogya Nidhi (RAN) is designed to provide financial assistance to poor patients living below the poverty line. The assistance is provided as a one-time grant to eligible patients. The scheme covers treatments for heart, kidney, liver diseases, cancer, and specified rare diseases. How many of the statements given above are correct? Only one Only two All three None of the above Q3.) With reference to the Chipko Movement, consider the following statements: The Chipko Movement was a non-violent environmental movement. The movement aimed to protect the Himalayan range trees from the axes of builders and commercial interests. Sunderlal Bahuguna played a pivotal role in the Chipko Movement. How many of the statements given above are not correct? Only one Only two Only three None Comment the answers to the above questions in the comment section below!! ANSWERS FOR ’  24th April  2024 – Daily Practice MCQs’ will be updated along with tomorrow’s Daily Current Affairs.st ANSWERS FOR  23rd April – Daily Practice MCQs Answers- Daily Practice MCQs Q.1) – d Q.2) – c Q.3) – b

Daily Prelims CA Quiz

UPSC Quiz – 2024 : IASbaba’s Daily Current Affairs Quiz 24th April 2024

The Current Affairs questions are based on sources like ‘The Hindu’, ‘Indian Express’ and ‘PIB’, which are very important sources for UPSC Prelims Exam. The questions are focused on both the concepts and facts. The topics covered here are generally different from what is being covered under ‘Daily Current Affairs/Daily News Analysis (DNA) and Daily Static Quiz’ to avoid duplication. The questions would be published from Monday to Saturday before 2 PM. One should not spend more than 10 minutes on this initiative. Gear up and Make the Best Use of this initiative. Do remember that, “the difference between Ordinary and EXTRA-Ordinary is PRACTICE!!” Important Note: Don’t forget to post your marks in the comment section. Also, let us know if you enjoyed today’s test 🙂After completing the 5 questions, click on ‘View Questions’ to check your score, time taken, and solutions.To take the Test Click Here

[DAY 45] 60 DAY RAPID REVISION (RaRe) SERIES for UPSC Prelims 2024 – ECONOMY, CURRENT AFFAIRS & CSAT TEST SERIES!

Archives Hello Friends The 60 Days Rapid Revision (RaRe) Series is IASbaba’s Flagship Initiative recommended by Toppers and loved by the aspirants’ community every year. It is the most comprehensive program which will help you complete the syllabus, revise and practice tests on a daily basis. The Programme on a daily basis includes Daily Prelims MCQs from Static (Monday – Saturday) Daily Static Quiz will cover all the topics of static subjects – Polity, History, Geography, Economics, Environment and Science and technology. 20 questions will be posted daily and these questions are framed from the topics mentioned in the schedule. It will ensure timely and streamlined revision of your static subjects. Daily Current Affairs MCQs (Monday – Saturday) Daily 5 Current Affairs questions, based on sources like ‘The Hindu’, ‘Indian Express’ and ‘PIB’, would be published from Monday to Saturday according to the schedule. Daily CSAT Quiz (Monday – Friday) CSAT has been an Achilles heel for many aspirants. Daily 5 CSAT Questions will be published. Note – Daily Test of 20 static questions, 10 current affairs, and 5 CSAT questions. (35 Prelims Questions) in QUIZ FORMAT will be updated on a daily basis. To Know More about 60 Days Rapid Revision (RaRe) Series – CLICK HERE   60 Day Rapid Revision (RaRe) Series Schedule – CLICK HERE  Important Note Comment your Scores in the Comment Section. This will keep you accountable, responsible and sincere in days to come. It will help us come out with the Cut-Off on a Daily Basis. Let us know if you enjoyed today’s test 🙂  You can post your comments in the given format  (1) Your Score (2) Matrix Meter (3) New Learning from the Test Time limit: 0 Test-summary 0 of 35 questions completed Questions: 1 2 3 4 5 6 7 8 9 10 11 12 13 14 15 16 17 18 19 20 21 22 23 24 25 26 27 28 29 30 31 32 33 34 35 Information The following Test is based on the syllabus of 60 Days Plan-2023 for UPSC IAS Prelims 2022. To view Solutions, follow these instructions: Click on – ‘Start Test’ button Solve Questions Click on ‘Test Summary’ button Click on ‘Finish Test’ button Now click on ‘View Questions’ button – here you will see solutions and links. You have already completed the test before. Hence you can not start it again. Test is loading... You must sign in or sign up to start the test. You have to finish following test, to start this test: Results 0 of 35 questions answered correctly Your time: Time has elapsed You have scored 0 points out of 0 points, (0) Average score     Your score     Categories Not categorized 0% Your result has been entered into leaderboard Loading Name: E-Mail: Captcha: maximum of 70 points Pos. Name Entered on Points Result Table is loading No data available 1 Siddhant 2024/04/24 12:25 PM 24 52.17 % 1 2 3 4 5 6 7 8 9 10 11 12 13 14 15 16 17 18 19 20 21 22 23 24 25 26 27 28 29 30 31 32 33 34 35 Answered Review Question 1 of 35 1. Question Consider the following statements with reference to Maharatna Companies. Only the Navratna Companies are eligible to be considered for grant of Maharatna status. Maharatna companies should have a global-level presence or operation. The average net worth of the company should be more than 1 lakh crore in the last three years. How many of the above statements are correct? a) Only one b) Only two c) All three d) None Correct Solution (b) The Government of India categorises Central Public Sector Enterprises (CPSEs) under three categories of Navratna, Miniratna, and Maharatna companies. A wide range of factors determines the categorization of these CPSEs. This includes annual turnover, net worth, and net profit calculated annually.   Criteria for grant of Maharatna status to CPSEs meeting the following criteria are eligible to be considered for grant of Maharatna status are Having Navratna status Listed on the Indian stock exchange with minimum prescribed public shareholding under SEBI regulations Hence, Statement 1 is correct. An average annual turnover of more than Rs. 25,000 crores during the last 3 years An average annual net worth of more than 15,000 crores during the last 3 years Maharatna Status does not require a net worth of more than 1 lakh crore in the last three years to get the maharatna status. Hence, Statement 3 is not correct. An average annual net profit after tax of more than Rs. 5,000 crores during the last 3 years Should have significant global presence/international operations. Hence, Statement 2 is correct. Incorrect Solution (b) The Government of India categorises Central Public Sector Enterprises (CPSEs) under three categories of Navratna, Miniratna, and Maharatna companies. A wide range of factors determines the categorization of these CPSEs. This includes annual turnover, net worth, and net profit calculated annually.   Criteria for grant of Maharatna status to CPSEs meeting the following criteria are eligible to be considered for grant of Maharatna status are Having Navratna status Listed on the Indian stock exchange with minimum prescribed public shareholding under SEBI regulations Hence, Statement 1 is correct. An average annual turnover of more than Rs. 25,000 crores during the last 3 years An average annual net worth of more than 15,000 crores during the last 3 years Maharatna Status does not require a net worth of more than 1 lakh crore in the last three years to get the maharatna status. Hence, Statement 3 is not correct. An average annual net profit after tax of more than Rs. 5,000 crores during the last 3 years Should have significant global presence/international operations. Hence, Statement 2 is correct. Question 2 of 35 2. Question Consider the following statements regarding Gender Budgeting in India: The term was first used during Union Budget in 1969 when Indira Gandhi became Prime Minister. The first part of the Gender Budgeting Statement in India is fully dedicated to women. Select the correct answer using the code given below. a) 1 only b) 2 only c) Both 1 and 2 d) Neither 1 nor 2 Correct Solution (b) Gender budgeting is a tool that allows governments to promote equality through fiscal policies by taking analyses of a budget’s differing impacts on the sexes as well as setting goals or targets for equality and allocating funds to support those goals Statement 1 Statement 2 Incorrect Correct The term Gender Budget was first introduced by the Union budget in 2001-02. Shri Atal Bihari Vajpayee was the PM during its introduction. Since 2005, the union government releases the Gender Budgeting Statement consisting of two parts.  The first part reflects the women-specific schemes in which 100 per cent allocation is only for women. The second part reflects pro-women schemes in which 30% of the allocation is earmarked for women. Incorrect Solution (b) Gender budgeting is a tool that allows governments to promote equality through fiscal policies by taking analyses of a budget’s differing impacts on the sexes as well as setting goals or targets for equality and allocating funds to support those goals Statement 1 Statement 2 Incorrect Correct The term Gender Budget was first introduced by the Union budget in 2001-02. Shri Atal Bihari Vajpayee was the PM during its introduction. Since 2005, the union government releases the Gender Budgeting Statement consisting of two parts.  The first part reflects the women-specific schemes in which 100 per cent allocation is only for women. The second part reflects pro-women schemes in which 30% of the allocation is earmarked for women. Question 3 of 35 3. Question Consider the following statements with respect to 15th Finance Commission: The Census of 1971 was used as the baseline for fund allocation by the 15th Finance Commission. The 15th Finance Commission proposed a 41% share of the divisible pool to be allocated to the States. Select the correct answer using the code given below. a) 1 only b) 2 only c) Both 1 and 2 d) Neither 1 nor 2 Correct Solution (b) The 14th Finance Commission used the Census of 1971 as the benchmark for fund allocation, not the 15th Finance Commission. The 15th Finance Commission utilized the Census of 2011 for this purpose. (Hence statement 1 is incorrect). The 15th Finance Commission recommended maintaining a 41% vertical devolution for distributing funds to the states from the divisible pool. The 14th Finance Commission suggested a 42% share. (Hence statement 2 is correct). Incorrect Solution (b) The 14th Finance Commission used the Census of 1971 as the benchmark for fund allocation, not the 15th Finance Commission. The 15th Finance Commission utilized the Census of 2011 for this purpose. (Hence statement 1 is incorrect). The 15th Finance Commission recommended maintaining a 41% vertical devolution for distributing funds to the states from the divisible pool. The 14th Finance Commission suggested a 42% share. (Hence statement 2 is correct). Question 4 of 35 4. Question Which of the following items are considered under the non-tax revenue receipts of the government of India? Interest Received Examination fee Dividend and profits Salaries Select the correct answer using the code given below: a) 1, 2 and 3 only b) 2 and 4 only c) 3 and 4 only d) 1, 2, 3 and 4 Correct Solution (a) Non-Tax Revenue is the recurring income earned by the government from sources other than taxes. Interest received, Examination fees, Dividends and Profits are all components of Non Tax receipts. Statement 1 Statement 2 Statement 3 Statement 4 Correct Correct Correct Incorrect Interests refer to interest on loans given to states and union territories for reasons like non-plan schemes (e.g. flood control) and planning schemes with a maturity period of 20 years such as modernization of police forces and also interest on loans advanced to Public Sector Enterprises (PSEs), Port Trusts and other statutory bodies etc. Examination fees: This includes fees paid by applicants of competitive examinations conducted by the Union Public Service Commission (UPSC) and Staff Selection Commission (SSC) to fill up vacancies in government offices. Dividends and profits: include dividends and profits from PSEs as well as the transfer of surplus from the Reserve Bank of India (RBI). Salaries do not come under the non-tax revenue receipts of the government of India. They comprise Revenue expenditure. Incorrect Solution (a) Non-Tax Revenue is the recurring income earned by the government from sources other than taxes. Interest received, Examination fees, Dividends and Profits are all components of Non Tax receipts. Statement 1 Statement 2 Statement 3 Statement 4 Correct Correct Correct Incorrect Interests refer to interest on loans given to states and union territories for reasons like non-plan schemes (e.g. flood control) and planning schemes with a maturity period of 20 years such as modernization of police forces and also interest on loans advanced to Public Sector Enterprises (PSEs), Port Trusts and other statutory bodies etc. Examination fees: This includes fees paid by applicants of competitive examinations conducted by the Union Public Service Commission (UPSC) and Staff Selection Commission (SSC) to fill up vacancies in government offices. Dividends and profits: include dividends and profits from PSEs as well as the transfer of surplus from the Reserve Bank of India (RBI). Salaries do not come under the non-tax revenue receipts of the government of India. They comprise Revenue expenditure. Question 5 of 35 5. Question With reference to taxes and their impact on the economy, consider the following statements: The low levels of tax collection in India are primarily attributed to the existence of a parallel economy. Implementation of a progressive tax system has the potential to alleviate income inequality. Select the correct answer using the code given below. a) 1 only b) 2 only c) Both 1 and 2 d) Neither 1 nor 2 Correct Solution (c) The presence of a parallel economy, associated with black money or unaccounted funds, is a significant challenge to the Indian economy and contributes to substantial losses in tax revenues for the government. (Hence statement 1 is correct). A progressive taxation system is designed to address income inequality by placing a heavier tax burden on wealthier individuals. (Hence statement 2 is correct). Incorrect Solution (c) The presence of a parallel economy, associated with black money or unaccounted funds, is a significant challenge to the Indian economy and contributes to substantial losses in tax revenues for the government. (Hence statement 1 is correct). A progressive taxation system is designed to address income inequality by placing a heavier tax burden on wealthier individuals. (Hence statement 2 is correct). Question 6 of 35 6. Question Which of the following are the primary factors contributing to the significant growth of the services sector in India post-liberalization? Availability of affordable labour. Reduced tariff and non-tariff barriers on imports. Appreciation of the rupee against foreign currencies. How many of the above statements are correct? a) Only one b) Only two c) All three d) None Correct Solution (b) The presence of inexpensive labour and a skilled workforce in India led to a surge in outsourcing by multinational companies, contributing significantly to the services sector’s growth after the reform period. (Hence statement 1 is correct). The reduction in tariff and non-tariff barriers on imports played a pivotal role in the services sector’s high growth. Foreign trade reforms facilitated domestic products to engage and compete globally. (Hence statement 2 is correct). The devaluation of the rupee against foreign currencies resulted in increased foreign exchange flow, encouraging numerous foreign companies to invest in India. This, in turn, contributed to the substantial growth of the services sector post-liberalization. (Hence statement 3 is incorrect). Incorrect Solution (b) The presence of inexpensive labour and a skilled workforce in India led to a surge in outsourcing by multinational companies, contributing significantly to the services sector’s growth after the reform period. (Hence statement 1 is correct). The reduction in tariff and non-tariff barriers on imports played a pivotal role in the services sector’s high growth. Foreign trade reforms facilitated domestic products to engage and compete globally. (Hence statement 2 is correct). The devaluation of the rupee against foreign currencies resulted in increased foreign exchange flow, encouraging numerous foreign companies to invest in India. This, in turn, contributed to the substantial growth of the services sector post-liberalization. (Hence statement 3 is incorrect). Question 7 of 35 7. Question Consider the following statements about the agriculture sector: It is the biggest unorganized sector of the economy. An increase in agricultural growth leads to an increase in industrial output. India has remained consistently a ‘net exporter’ of Agri-products. How many of the above statements are correct? a) Only one b) Only two c) All three d) None Correct Solution (c) Agriculture is the biggest unorganized sector of the economy accounting for more than 90 per cent share in the total unorganized labour-force. (Hence statement 1 is correct). Agriculture is deeply related to industrial growth and the national income in India—1 per cent increase in agricultural growth leads to 0.5 per cent increase in industrial output (growth) and 0.7 per cent increase in the national income of India. (Hence statement 2 is correct). India has remained consistently a ‘net exporter’ of Agri-products (in 2018-19, value of exports being ₹2.7 lakh crore against import of ₹1.37 lakh crore). India occupies a leading position in global agricultural trade having a share of 2.15 percent in the world agricultural trade. (Hence statement 3 is correct). Incorrect Solution (c) Agriculture is the biggest unorganized sector of the economy accounting for more than 90 per cent share in the total unorganized labour-force. (Hence statement 1 is correct). Agriculture is deeply related to industrial growth and the national income in India—1 per cent increase in agricultural growth leads to 0.5 per cent increase in industrial output (growth) and 0.7 per cent increase in the national income of India. (Hence statement 2 is correct). India has remained consistently a ‘net exporter’ of Agri-products (in 2018-19, value of exports being ₹2.7 lakh crore against import of ₹1.37 lakh crore). India occupies a leading position in global agricultural trade having a share of 2.15 percent in the world agricultural trade. (Hence statement 3 is correct). Question 8 of 35 8. Question Consider the following statements with reference to the New Industrial Policy of 1991. Abolition of Industrial licensing of all industries Allowed FDI upto 51% under automatic route for the first time. Indian companies were directly allowed to negotiate the terms of technology transfer with their foreign counterparts How many of the above statements are correct? a) Only one b) Only two c) All three d) None Correct Solution (b) Industrial licensing was abolished for all industries except those specified. These specified industries will continue to be subjected to compulsory licensing for reasons related to security and strategic concerns, social reasons, problems related to safety and overriding environmental issues, manufacture of products of hazardous nature and articles of elitist consumption. Hence, Statement 1 is not correct. Prior to 1991, foreign investment in India was tightly controlled through industrial licensing and foreign exchange restrictions. The NIP significantly liberalised these controls, introducing several key measures: Automatic approval for FDI up to 51% in 47 high-priority industries, eliminating the need for prior government approvals. Streamlined approval process for higher levels of FDI through the Foreign Investment Promotion Board (FIPB). Relaxation of foreign exchange regulations to facilitate investment inflows. Focus on attracting foreign technology and expertise to boost India’s industrial development. Hence, Statement 2 is correct. Indian companies will be free to negotiate the terms of technology transfer with their foreign counterparts according to their own commercial judgement. Actively encourage and assist Indian entrepreneurs in exploiting and meeting emerging domestic and global opportunities and challenges. The bedrock of any such package of measures must be to let the entrepreneurs make investment decisions on the basis of their own commercial judgement. Hence, Statement 3 is correct. Incorrect Solution (b) Industrial licensing was abolished for all industries except those specified. These specified industries will continue to be subjected to compulsory licensing for reasons related to security and strategic concerns, social reasons, problems related to safety and overriding environmental issues, manufacture of products of hazardous nature and articles of elitist consumption. Hence, Statement 1 is not correct. Prior to 1991, foreign investment in India was tightly controlled through industrial licensing and foreign exchange restrictions. The NIP significantly liberalised these controls, introducing several key measures: Automatic approval for FDI up to 51% in 47 high-priority industries, eliminating the need for prior government approvals. Streamlined approval process for higher levels of FDI through the Foreign Investment Promotion Board (FIPB). Relaxation of foreign exchange regulations to facilitate investment inflows. Focus on attracting foreign technology and expertise to boost India’s industrial development. Hence, Statement 2 is correct. Indian companies will be free to negotiate the terms of technology transfer with their foreign counterparts according to their own commercial judgement. Actively encourage and assist Indian entrepreneurs in exploiting and meeting emerging domestic and global opportunities and challenges. The bedrock of any such package of measures must be to let the entrepreneurs make investment decisions on the basis of their own commercial judgement. Hence, Statement 3 is correct. Question 9 of 35 9. Question Consider the following schemes: The Gati Shakti Programme Sagarmala Bharatmala e-sanchit and FASTags Ayushman Bharat-PMJAY How many of the schemes mentioned above are considered supportive initiatives for developing the logistics sector? a) Two only b) Three only c) Four only d) All five Correct Solution (c) Steps taken by the Government to develop the logistics sector: The Gati Shakti Programme: This initiative focuses on implementing infrastructure connectivity projects, including roadways and railways, in a coordinated manner across the nation to enhance logistics efficiency. (Hence 1 is correct) Sagarmala: This initiative aims to utilize the potential of the coastline and waterways to reduce the amount of infrastructure needed for logistics and improve cargo movement. (Hence 2 is correct) Bharatmala: The Bharatmala project focuses on reducing critical infrastructure gaps in the road sector to improve the effectiveness of road traffic circulation and enhance logistics efficiency. (Hence 3 is correct) Comprehensive Logistics Action Plan (CLAP): Aims to rank India among the top 25 countries in the Logistics Performance Index by 2030. e-sanchit and FASTags: Implementing paperless export-import trade operations and enhancing efficiency on roads. (Hence 4 is correct) GST: A unified tax system like GST simplifies logistics-related issues. Ayushman Bharat-PMJAY: Ayushman Bharat-PMJAY is a national public health insurance scheme that provides healthcare benefits to eligible beneficiaries and is not related to enhancing logistics efficiency. (Hence 5 is incorrect) Incorrect Solution (c) Steps taken by the Government to develop the logistics sector: The Gati Shakti Programme: This initiative focuses on implementing infrastructure connectivity projects, including roadways and railways, in a coordinated manner across the nation to enhance logistics efficiency. (Hence 1 is correct) Sagarmala: This initiative aims to utilize the potential of the coastline and waterways to reduce the amount of infrastructure needed for logistics and improve cargo movement. (Hence 2 is correct) Bharatmala: The Bharatmala project focuses on reducing critical infrastructure gaps in the road sector to improve the effectiveness of road traffic circulation and enhance logistics efficiency. (Hence 3 is correct) Comprehensive Logistics Action Plan (CLAP): Aims to rank India among the top 25 countries in the Logistics Performance Index by 2030. e-sanchit and FASTags: Implementing paperless export-import trade operations and enhancing efficiency on roads. (Hence 4 is correct) GST: A unified tax system like GST simplifies logistics-related issues. Ayushman Bharat-PMJAY: Ayushman Bharat-PMJAY is a national public health insurance scheme that provides healthcare benefits to eligible beneficiaries and is not related to enhancing logistics efficiency. (Hence 5 is incorrect) Question 10 of 35 10. Question How many among the following Central Public Sector enterprises are categorised as Maharatna Company? Coal India Limited National Aluminium Company Limited Rural Electrification Corporation Limited Bharat Electronics Limited. Select the correct answer using the code given below a) Only one b) Only two c) Only three d) All four Correct Solution (b) The Government of India categorises Central Public Sector Enterprises (CPSEs) under three categories of Navratna, Miniratna, and Maharatna companies. A wide range of factors determines the categorization of these CPSEs. This includes annual turnover, net worth, and net profit calculated on an annual basis. Along with this, the presence of the company on the stock exchange list as per the Securities and Exchange Board of India (SEBI) rules.   The following Central Public Sector enterprises were categorised as Maharatna Company Bharat Heavy Electricals Limited Bharat Petroleum Corporation Limited Coal India Limited Hence, Statement 1 is correct. GAIL India Limited Hindustan Petroleum Corporation Limited Indian Oil Corporation Limited NTPC Limited Oil & Natural Gas Corporation Limited, Power Finance Corporation Power Grid Corporation of India Limited Rural Electrification Corporation Limited Hence, Statement 3 is correct. Steel Authority of India Limited Recently Rural Electrification Corporation Limited The following Central Public Sector enterprises were categorized as Navaratna companies in India: Bharat Electronics Limited (BEL). Hence, Statement 4 is not correct. Container Corporation of India Limited (CONCOR) Engineers India Limited (EIL) Hindustan Aeronautics Limited (HAL) Hindustan Petroleum Corporation Limited (HPCL) Mahanagar Telephone Nigam Limited (MTNL) National Aluminium Company Limited (NALCO) Hence, Statement 2 is not correct. NBCC (India) Limited NMDC Limited NLC India Limited Oil India Limited (OIL) Power Finance Corporation Limited (PFC) Power Grid Corporation of India Limited (PGCIL) Rashtriya Ispat Nigam Limited (RINL) Incorrect Solution (b) The Government of India categorises Central Public Sector Enterprises (CPSEs) under three categories of Navratna, Miniratna, and Maharatna companies. A wide range of factors determines the categorization of these CPSEs. This includes annual turnover, net worth, and net profit calculated on an annual basis. Along with this, the presence of the company on the stock exchange list as per the Securities and Exchange Board of India (SEBI) rules.   The following Central Public Sector enterprises were categorised as Maharatna Company Bharat Heavy Electricals Limited Bharat Petroleum Corporation Limited Coal India Limited Hence, Statement 1 is correct. GAIL India Limited Hindustan Petroleum Corporation Limited Indian Oil Corporation Limited NTPC Limited Oil & Natural Gas Corporation Limited, Power Finance Corporation Power Grid Corporation of India Limited Rural Electrification Corporation Limited Hence, Statement 3 is correct. Steel Authority of India Limited Recently Rural Electrification Corporation Limited The following Central Public Sector enterprises were categorized as Navaratna companies in India: Bharat Electronics Limited (BEL). Hence, Statement 4 is not correct. Container Corporation of India Limited (CONCOR) Engineers India Limited (EIL) Hindustan Aeronautics Limited (HAL) Hindustan Petroleum Corporation Limited (HPCL) Mahanagar Telephone Nigam Limited (MTNL) National Aluminium Company Limited (NALCO) Hence, Statement 2 is not correct. NBCC (India) Limited NMDC Limited NLC India Limited Oil India Limited (OIL) Power Finance Corporation Limited (PFC) Power Grid Corporation of India Limited (PGCIL) Rashtriya Ispat Nigam Limited (RINL) Question 11 of 35 11. Question Consider the following statements with reference to the New Public Sector Enterprises (PSEI) Policy of 2021. It classifies all the Central Public Sector Enterprises into Strategic and Non-Strategic sectors. The Government will have only a bare minimum presence in the Non- strategic sector. Department of Investment and Public Asset Management is responsible for the implementation of the policy How many of the above statements are correct? a) Only one b) Only two c) All three d) None Correct Solution (a) The Government notified the new Public Sector Enterprise (PSE) Policy on February 4, 2O2l for Atmanirbhar Bharat. The new PSE Policy envisages the classification of CPSEs into Strategic and Non Strategic Sectors and exempts certain CPSEs, such as those set up as not-for-profit companies under the Companies Act 2013 or those supporting vulnerable and weaker sections of society, from the scope of the policy. Hence, Statement 1 is not correct. CPSEs in the Non- Strategic Sector will be privatised, otherwise shall be closed. Only a bare minimum presence of CPSEs in the Strategic Sector is to be maintained. Hence, Statement 2 is not correct. DIPAM (Department of Investment and public asset management) is India’s custodian of disinvestment proceeds. The Department of Public Enterprises (DPE) is the implementation agency of disinvestment policy. DPE has been entrusted with the responsibility to identify CPSEs for closure or privatisation in the non-strategic sector in consultation with administrative ministries/departments and to take in-principle approval from CCEA in respect of such identified CPSEs. Hence, Statement 3 is correct. Incorrect Solution (a) The Government notified the new Public Sector Enterprise (PSE) Policy on February 4, 2O2l for Atmanirbhar Bharat. The new PSE Policy envisages the classification of CPSEs into Strategic and Non Strategic Sectors and exempts certain CPSEs, such as those set up as not-for-profit companies under the Companies Act 2013 or those supporting vulnerable and weaker sections of society, from the scope of the policy. Hence, Statement 1 is not correct. CPSEs in the Non- Strategic Sector will be privatised, otherwise shall be closed. Only a bare minimum presence of CPSEs in the Strategic Sector is to be maintained. Hence, Statement 2 is not correct. DIPAM (Department of Investment and public asset management) is India’s custodian of disinvestment proceeds. The Department of Public Enterprises (DPE) is the implementation agency of disinvestment policy. DPE has been entrusted with the responsibility to identify CPSEs for closure or privatisation in the non-strategic sector in consultation with administrative ministries/departments and to take in-principle approval from CCEA in respect of such identified CPSEs. Hence, Statement 3 is correct. Question 12 of 35 12. Question With respect to the Bombay Plan, which of the following statements is/are correct? The plan advocated for the abolition of the zamindari system. The plan supported the Gandhian concept of village economy and denounced rapid industrialization. It laid emphasis on promoting medium-scale, small-scale and cottage industries. It did not want the state to play any role in the economy. How many of the above statements are correct? a) Only one b) Only two c) Only three d) All four Correct Solution (b) The Bombay Plan laid emphasis on agriculture restructuring. As a part of this, it proposed abolition of intermediaries (zamindari abolition). (Hence statement 1 is correct). The Plan did not support the Gandhian concept of village economy, rather it was in support of rapid industrialization and emphasized heavy capital goods and basic industries. (Hence statement 2 is incorrect). It also gave importance to medium-scale, small-scale and cottage industries as they provide greater employment and require lesser capital. (Hence statement 3 is correct). Bombay plan wanted the state to play an active role. It was in the support of a controlled economy where the government plans, controls and oversees the different areas of the economy (Hence statement 4 is incorrect). Incorrect Solution (b) The Bombay Plan laid emphasis on agriculture restructuring. As a part of this, it proposed abolition of intermediaries (zamindari abolition). (Hence statement 1 is correct). The Plan did not support the Gandhian concept of village economy, rather it was in support of rapid industrialization and emphasized heavy capital goods and basic industries. (Hence statement 2 is incorrect). It also gave importance to medium-scale, small-scale and cottage industries as they provide greater employment and require lesser capital. (Hence statement 3 is correct). Bombay plan wanted the state to play an active role. It was in the support of a controlled economy where the government plans, controls and oversees the different areas of the economy (Hence statement 4 is incorrect). Question 13 of 35 13. Question Consider the following types of taxes: Corporation Tax Tax on Income Wealth Tax Customs Duty Excise Duty Which of the above can be categorized as direct taxes? a) 1, 2 and 3 only b) 1, 4 and 5 only c) 4 and 5 only d) 2, 3 and 5 only Correct Solution (a) Types of Direct Taxes: Income Tax: Imposed on an individual’s or entity’s income, varying with income levels. Progressive tax rates may apply, with higher rates for higher income brackets. Corporation Tax: Applicable to the income of corporations and businesses. The tax rate may differ for domestic and foreign companies. Wealth Tax: Levied on an individual’s net wealth, including assets like property, jewellery, and financial instruments. Phased out in India in recent years. Gift Tax: Applied when an individual receives a gift, with the giver being responsible for the tax payment. In some jurisdictions, gift tax is merged with other taxes. Incorrect Solution (a) Types of Direct Taxes: Income Tax: Imposed on an individual’s or entity’s income, varying with income levels. Progressive tax rates may apply, with higher rates for higher income brackets. Corporation Tax: Applicable to the income of corporations and businesses. The tax rate may differ for domestic and foreign companies. Wealth Tax: Levied on an individual’s net wealth, including assets like property, jewellery, and financial instruments. Phased out in India in recent years. Gift Tax: Applied when an individual receives a gift, with the giver being responsible for the tax payment. In some jurisdictions, gift tax is merged with other taxes. Question 14 of 35 14. Question Which of the following suggestion did the Narasimhan Committee made, which was established in 1991? Reduce the quantity of Public Sector Banks by way of acquisitions and mergers. Establish Asset Recovery Firms. Gradually phase off interest subsidies for small-scale and agricultural industries. The Cash Reserve Ratio (CRR) should be the main tool for managing money. How many of the above statements are correct? a) Only one b) Only two c) Only three d) All four Correct Solution (c) The committee proposed a significant decrease in the number of public sector banks (PSBs) through mergers and acquisitions, aiming to enhance efficiency in banking operations. It also suggested empowering the RBI as the primary regulator of the banking system, while advocating for the autonomy of PSBs. (Hence statement 1 is correct). In response to the challenge posed by the increasing non-performing assets (NPAs) in banks and financial institutions, the committee recommended the establishment of asset reconstruction companies or funds. (Hence statement 2 is correct). The committee proposed a gradual phasing out of the directed credit programme. It argued that sectors like agriculture and small-scale industries (SSIs) had matured sufficiently and no longer required special support, suggesting that concessional interest rates could be discontinued after two decades of interest subsidies. (Hence statement 3 is correct). Contrary to the statement, the committee advised against using the Cash Reserve Ratio (CRR) as the primary tool for monetary and credit control. Instead, it recommended an increased reliance on open market operations (OMOs). Additionally, the committee suggested a gradual reduction of the CRR from its current high level of 15 percent to a range of 3 to 5 percent. (Hence statement 4 is incorrect). Incorrect Solution (c) The committee proposed a significant decrease in the number of public sector banks (PSBs) through mergers and acquisitions, aiming to enhance efficiency in banking operations. It also suggested empowering the RBI as the primary regulator of the banking system, while advocating for the autonomy of PSBs. (Hence statement 1 is correct). In response to the challenge posed by the increasing non-performing assets (NPAs) in banks and financial institutions, the committee recommended the establishment of asset reconstruction companies or funds. (Hence statement 2 is correct). The committee proposed a gradual phasing out of the directed credit programme. It argued that sectors like agriculture and small-scale industries (SSIs) had matured sufficiently and no longer required special support, suggesting that concessional interest rates could be discontinued after two decades of interest subsidies. (Hence statement 3 is correct). Contrary to the statement, the committee advised against using the Cash Reserve Ratio (CRR) as the primary tool for monetary and credit control. Instead, it recommended an increased reliance on open market operations (OMOs). Additionally, the committee suggested a gradual reduction of the CRR from its current high level of 15 percent to a range of 3 to 5 percent. (Hence statement 4 is incorrect). Question 15 of 35 15. Question Consider the following: Farmers Scientist Poors Women Youth Teachers Which of the above are mentioned as the 4 pillars of Viksit Bharat in the Interim Budget 2024? a) 1 and 2 only b) 1 ,3 ,4 and 5 only c) 2, 3, and 6 only d) 1, 2, 4, 5 and 6 only Correct Solution (b) Garib (Poor): This pillar focuses on the welfare of the poor and marginalized sections of society. Key initiatives include Garib Kalyan programs and measures aimed at overall welfare and development of the nation. Mahilayen (Women): This pillar emphasizes women empowerment and gender equality. Nari Shakti programs and policies are central to this pillar. Yuva (Youth): It aims to provide opportunities for skill development, education, and employment to the youth population. Initiatives under this pillar aim to harness the potential of young individuals and channel their energy towards constructive endeavours. Annadata (Farmer): This pillar is dedicated to the welfare and support of farmers, who are the backbone of the nation. It includes policies and programs aimed at improving agricultural productivity, ensuring fair prices for agricultural produce, and providing financial support and assistance to farmers. Incorrect Solution (b) Garib (Poor): This pillar focuses on the welfare of the poor and marginalized sections of society. Key initiatives include Garib Kalyan programs and measures aimed at overall welfare and development of the nation. Mahilayen (Women): This pillar emphasizes women empowerment and gender equality. Nari Shakti programs and policies are central to this pillar. Yuva (Youth): It aims to provide opportunities for skill development, education, and employment to the youth population. Initiatives under this pillar aim to harness the potential of young individuals and channel their energy towards constructive endeavours. Annadata (Farmer): This pillar is dedicated to the welfare and support of farmers, who are the backbone of the nation. It includes policies and programs aimed at improving agricultural productivity, ensuring fair prices for agricultural produce, and providing financial support and assistance to farmers. Question 16 of 35 16. Question Consider the following statements with respect to Feldman–Mahalanobis model: The model extensively highlights the crucial sources of capital, such as taxation. According to this model, the growth rate is determined by both the proportion of investment in the capital goods sector and the proportion of investment in the consumer goods sector. Due to the necessity for ample raw material reserves for sustainability, the model is less suitable for smaller countries. How many of the above statements are correct? a) Only one b) Only two c) All three d) None Correct Solution (b) The Feldman-Mahalanobis model, a Neo-Marxist theory of economic development, was independently formulated by Soviet economist G. A. Feldman in 1928 and Indian statistician Prasanta Chandra Mahalanobis in 1953. Mahalanobis’ model gives little attention to the savings constraint, which is assumed to originate from the industrial sector. However, in developing countries, initial savings often stem from the agricultural sector, and taxation, one of the essential sources of capital, is not addressed in this model. (Hence statement 1 is incorrect). According to the model, the growth rate depends on both the share of investment in the capital goods sector, denoted by k, and the share of investment in the consumer goods sector, denoted by c. If the value of k surpasses c, it may lead to slower short-term growth, but in the long run, it can yield higher consumption levels. (Hence statement 2 is correct). The model presupposes that a country must possess sufficient resources to sustain itself, making it less applicable to smaller countries. Therefore, this aspect of the model does not hold true for nations lacking in adequate raw material resources. (Hence statement 3 is correct). Incorrect Solution (b) The Feldman-Mahalanobis model, a Neo-Marxist theory of economic development, was independently formulated by Soviet economist G. A. Feldman in 1928 and Indian statistician Prasanta Chandra Mahalanobis in 1953. Mahalanobis’ model gives little attention to the savings constraint, which is assumed to originate from the industrial sector. However, in developing countries, initial savings often stem from the agricultural sector, and taxation, one of the essential sources of capital, is not addressed in this model. (Hence statement 1 is incorrect). According to the model, the growth rate depends on both the share of investment in the capital goods sector, denoted by k, and the share of investment in the consumer goods sector, denoted by c. If the value of k surpasses c, it may lead to slower short-term growth, but in the long run, it can yield higher consumption levels. (Hence statement 2 is correct). The model presupposes that a country must possess sufficient resources to sustain itself, making it less applicable to smaller countries. Therefore, this aspect of the model does not hold true for nations lacking in adequate raw material resources. (Hence statement 3 is correct). Question 17 of 35 17. Question Match the following pairs: List I                                      List II Merit goods                      Goods with the non- exclusion principle Public goods                    Provided to targeted individuals. Demerit goods                 Consumption of these goods negatively impacts society. Select the correct option using the code given below: a) A: 3; B: 1; C: 2 b) A: 2; B: 1; C: 3 c) A:1; B: 2; C: 3 d) A:1; B: 3; C: 2 Correct Solution (b)                   List I                                        List II Merit goods          Provided to targeted individuals. Public goods         Goods with the non- exclusion principle Demerit goods     Consumption of these goods negatively impacts society. (Hence b is correct code). Incorrect Solution (b)                   List I                                        List II Merit goods          Provided to targeted individuals. Public goods         Goods with the non- exclusion principle Demerit goods     Consumption of these goods negatively impacts society. (Hence b is correct code). Question 18 of 35 18. Question Consider the following statements with reference to Special Economic Zones (SEZs): SEZs are designated duty-free enclaves treated as part of the customs territory of India. The SEZ Act 2005 establishes a legal framework for Special Economic Zones in India. Domestic sales from SEZs are subject to full customs duty. The Baba Kalyani committee is related to India’s SEZ policy. How many of the above statements are correct? a) Only one b) Only two c) Only three d) All four Correct Solution (d) SEZs are designated as duty-free enclaves treated as part of India’s customs territory. Goods and services produced within SEZs are exempt from customs duties, providing a competitive advantage for businesses operating within these zones. (Hence statement 1 is correct). The SEZ Act of 2005 establishes the legal framework for Special Economic Zones in India. It provides the necessary guidelines and regulations for the establishment, operation, and management of SEZs, contributing to a conducive business environment. (Hence statement 2 is correct). Domestic sales from SEZs are subject to full customs duty. SEZs enjoy exemptions on customs duties for both international and domestic transactions, fostering a favourable environment for economic activities within these zones. (Hence statement 3 is correct). The Baba Kalyani-led committee was constituted by the Ministry of Commerce and Industry to evaluate and provide recommendations on India’s SEZ policy. The committee’s focus was on aligning the policy with World Trade Organization (WTO) standards and implementing global best practices to optimize SEZs’ capacity utilization and potential output. (Hence statement 4 is correct). Incorrect Solution (d) SEZs are designated as duty-free enclaves treated as part of India’s customs territory. Goods and services produced within SEZs are exempt from customs duties, providing a competitive advantage for businesses operating within these zones. (Hence statement 1 is correct). The SEZ Act of 2005 establishes the legal framework for Special Economic Zones in India. It provides the necessary guidelines and regulations for the establishment, operation, and management of SEZs, contributing to a conducive business environment. (Hence statement 2 is correct). Domestic sales from SEZs are subject to full customs duty. SEZs enjoy exemptions on customs duties for both international and domestic transactions, fostering a favourable environment for economic activities within these zones. (Hence statement 3 is correct). The Baba Kalyani-led committee was constituted by the Ministry of Commerce and Industry to evaluate and provide recommendations on India’s SEZ policy. The committee’s focus was on aligning the policy with World Trade Organization (WTO) standards and implementing global best practices to optimize SEZs’ capacity utilization and potential output. (Hence statement 4 is correct). Question 19 of 35 19. Question With reference to direct and indirect tax, consider the following statements: Direct tax has incidence and impact on the same individual. In indirect tax, the burden of tax can be shifted to someone else. Indirect taxes are progressive in nature. How many of the above statements are correct? a) Only one b) Only two c) All three d) None Correct Solution (b) Direct taxes, like income tax or property tax, have incidence and impact at the same point and cannot be shifted by the taxpayer to someone else. (Hence statement 1 is correct). Indirect taxes, such as customs duty or value-added tax, have incidence and impact at different points, allowing the taxpayer to shift the burden to someone else. (Hence statement 2 is correct). Indirect taxes are regressive, impacting lower incomes more than higher incomes, as the tax burden is the same for everyone, regardless of income level. (Hence statement 3 is incorrect). Incorrect Solution (b) Direct taxes, like income tax or property tax, have incidence and impact at the same point and cannot be shifted by the taxpayer to someone else. (Hence statement 1 is correct). Indirect taxes, such as customs duty or value-added tax, have incidence and impact at different points, allowing the taxpayer to shift the burden to someone else. (Hence statement 2 is correct). Indirect taxes are regressive, impacting lower incomes more than higher incomes, as the tax burden is the same for everyone, regardless of income level. (Hence statement 3 is incorrect). Question 20 of 35 20. Question Consider the following statements regarding Five Year Plans: The First Five-Year Plan was based on the Mahalanobis Model. The Third Five-Year Plan is also known as the ‘Gadgil Yojana/Plan’. The Fifth Five-Year Plan was terminated in 1978 due to severe drought and famine. The Sixth Five-Year Plan aimed at poverty eradication and employment generation, with the famous slogan ‘Garibi Hatao’. How many of the above statements are correct? a) Only one b) Only two c) Only three d) All four Correct Solution (b) The First Five-Year Plan was based on the Harrod-Domar model, not the Mahalanobis Model. It emphasized agricultural development, including irrigation and power projects, laying the foundation for economic growth in post-independence India. (Hence statement 1 is incorrect). The Third Five-Year Plan, known as the ‘Gadgil Yojana/Plan,’ aimed at economic self-reliance, balanced regional development, and achieving a self-sustained take-off for the Indian economy. (Hence statement 2 is correct). The Fifth Five-Year Plan was terminated in 1978, but not due to drought or famine. The termination was a result of a change in the political regime with the Janata Party coming to power, causing disruptions during the plan period. (Hence statement 3 is incorrect). The Sixth Five-Year Plan focused on poverty eradication and employment generation, with the iconic slogan ‘Garibi Hatao’ coined by the then-Prime Minister Indira Gandhi. The plan was successful, surpassing its targeted growth rate and contributing to socio-economic development. (Hence statement 4 is correct). Incorrect Solution (b) The First Five-Year Plan was based on the Harrod-Domar model, not the Mahalanobis Model. It emphasized agricultural development, including irrigation and power projects, laying the foundation for economic growth in post-independence India. (Hence statement 1 is incorrect). The Third Five-Year Plan, known as the ‘Gadgil Yojana/Plan,’ aimed at economic self-reliance, balanced regional development, and achieving a self-sustained take-off for the Indian economy. (Hence statement 2 is correct). The Fifth Five-Year Plan was terminated in 1978, but not due to drought or famine. The termination was a result of a change in the political regime with the Janata Party coming to power, causing disruptions during the plan period. (Hence statement 3 is incorrect). The Sixth Five-Year Plan focused on poverty eradication and employment generation, with the iconic slogan ‘Garibi Hatao’ coined by the then-Prime Minister Indira Gandhi. The plan was successful, surpassing its targeted growth rate and contributing to socio-economic development. (Hence statement 4 is correct). Question 21 of 35 21. Question Vaccine Safety Net is a global network of websites that provides reliable information on vaccine safety. It is established by the? a) World Health Organization b) United Nations Conference on Trade and Development c) World Trade Organization d) None Correct Solution (a) Vaccine Safety Net is a global network of websites that provides reliable information on vaccine safety. It is established by the World Health Organization (WHO). Hence option a is correct. It is a network of a diverse group of digital information resources (websites and social media), VSN members, located in countries around the world, and providing scientifically based information on vaccine safety in various languages. Incorrect Solution (a) Vaccine Safety Net is a global network of websites that provides reliable information on vaccine safety. It is established by the World Health Organization (WHO). Hence option a is correct. It is a network of a diverse group of digital information resources (websites and social media), VSN members, located in countries around the world, and providing scientifically based information on vaccine safety in various languages. Question 22 of 35 22. Question Consider the following statements regarding the ‘Mekong River’: It is the longest river in Southeast Asia. It originates from the Sanjianyuang in the Tibetan Plateau. It flows through China, Vietnam, Laos, Myanmar, Thailand, and Cambodia. Vientiane, the capital of Laos, and Phnom Penh, the capital of Cambodia, both stand on its banks. How many of the above statements are correct? a) Only one b) Only two c) Only three d) All four Correct Solution (d) The Mekong River is the longest river in Southeast Asia. Hence statement 1 is correct. It is the 7th longest in Asia and the 12th longest in the world. It has a length of about 2,700 miles (4,350 km). It originates from the Sanjianyuang in the Tibetan Plateau. Hence statement 2 is correct. The left-bank tributaries (draining high rainfall areas) include Nam Ou, Tha, and Nam Khan, while the right-bank tributaries (draining the lower relief region) are Ruak, Kok, Tonle Sap, and Mun. It flows through China, Vietnam, Laos, Myanmar, Thailand, and Cambodia. Hence statement 3 is correct. Its biodiversity is only second to the Amazon River Basin and contains about 20,000 plant species, 1,200 birds, 430 mammals, 800 amphibians and reptiles, and 850 fish species. The Mekong creates a huge delta, or triangular piece of land, in southern Vietnam. The delta has rich soil and is one of the world’s great producers of rice. Vientiane, the capital of Laos, and Phnom Penh, the capital of Cambodia, both stand on its banks. Hence statement 4 is correct. The river drains into the South China Sea south of Ho Chi Minh City (Vietnam).   Incorrect Solution (d) The Mekong River is the longest river in Southeast Asia. Hence statement 1 is correct. It is the 7th longest in Asia and the 12th longest in the world. It has a length of about 2,700 miles (4,350 km). It originates from the Sanjianyuang in the Tibetan Plateau. Hence statement 2 is correct. The left-bank tributaries (draining high rainfall areas) include Nam Ou, Tha, and Nam Khan, while the right-bank tributaries (draining the lower relief region) are Ruak, Kok, Tonle Sap, and Mun. It flows through China, Vietnam, Laos, Myanmar, Thailand, and Cambodia. Hence statement 3 is correct. Its biodiversity is only second to the Amazon River Basin and contains about 20,000 plant species, 1,200 birds, 430 mammals, 800 amphibians and reptiles, and 850 fish species. The Mekong creates a huge delta, or triangular piece of land, in southern Vietnam. The delta has rich soil and is one of the world’s great producers of rice. Vientiane, the capital of Laos, and Phnom Penh, the capital of Cambodia, both stand on its banks. Hence statement 4 is correct. The river drains into the South China Sea south of Ho Chi Minh City (Vietnam).   Question 23 of 35 23. Question Consider the following statements regarding the Nagoya Protocol on Access to Genetic Resources and the Fair and Equitable Sharing of Benefits Arising from their Utilization: It is a legally binding global agreement that implements the access and benefit-sharing obligations of the United Nations Framework Convention on Climate Change (UNFCCC). It establishes a framework that helps researchers access genetic resources for biotechnology research, development, and other activities, in return for a fair share of any benefits from their use. Choose the correct code: a) 1 only b) 2 only c) Both 1 and 2 d) Neither 1 nor 2 Correct Solution (b) The Nagoya Protocol on Access to Genetic Resources and the Fair and Equitable Sharing of Benefits Arising from their Utilization is a legally binding global agreement that implements the access and benefit-sharing obligations of the United Nations Convention on Biological Diversity (CBD). Hence statement 1 is incorrect. It was adopted by the CBD in Nagoya, Japan, in October 2010 and entered into force on October 12, 2014, 90 days after the deposit of the fiftieth instrument of ratification. It provides a transparent legal framework for the effective implementation of one of the three objectives of the CBD: the fair and equitable sharing of benefits arising out of the utilization of genetic resources. It applies to genetic resources that are covered by the CBD, and to the benefits arising from their utilization. It also covers traditional knowledge (TK) associated with genetic resources that are covered by the CBD and the benefits arising from their utilization. It establishes a framework that helps researchers access genetic resources for biotechnology research, development, and other activities, in return for a fair share of any benefits from their use. Hence statement 2 is correct. Indigenous and local communities may receive benefits through a legal framework that respects the value of traditional knowledge associated with genetic resources.   Incorrect Solution (b) The Nagoya Protocol on Access to Genetic Resources and the Fair and Equitable Sharing of Benefits Arising from their Utilization is a legally binding global agreement that implements the access and benefit-sharing obligations of the United Nations Convention on Biological Diversity (CBD). Hence statement 1 is incorrect. It was adopted by the CBD in Nagoya, Japan, in October 2010 and entered into force on October 12, 2014, 90 days after the deposit of the fiftieth instrument of ratification. It provides a transparent legal framework for the effective implementation of one of the three objectives of the CBD: the fair and equitable sharing of benefits arising out of the utilization of genetic resources. It applies to genetic resources that are covered by the CBD, and to the benefits arising from their utilization. It also covers traditional knowledge (TK) associated with genetic resources that are covered by the CBD and the benefits arising from their utilization. It establishes a framework that helps researchers access genetic resources for biotechnology research, development, and other activities, in return for a fair share of any benefits from their use. Hence statement 2 is correct. Indigenous and local communities may receive benefits through a legal framework that respects the value of traditional knowledge associated with genetic resources.   Question 24 of 35 24. Question Consider the following statements regarding the Directorate General of GST Intelligence (DGGI): It is an apex intelligence organization functioning under the Central Board of Indirect Taxes and Customs. It is responsible for taking legal action against offenders, but it cannot impose penalties. It is entrusted with the task of collection, collation, and dissemination of intelligence relating to the evasion of Goods and Services Tax (GST) and the duties of Central Excise and Service Tax on an all-India basis. How many of the above statements are correct? a) Only one b) Only two c) All three d) None Correct Solution (b) The Directorate General of GST Intelligence (DGGI) is an apex intelligence organization functioning under the Central Board of Indirect Taxes and Customs, Department of Revenue, and Ministry of Finance. Hence statement 1 is correct. The Directorate General of Central Excise Intelligence (DGCEI) is now renamed as Directorate General of GST Intelligence (DGGI). It has the power to conduct investigations into suspected cases of GST evasion or non-compliance. This may involve summoning persons, examining records, and carrying out searches and seizures. It is responsible for enforcing the provisions of the GST law. This includes taking legal action against offenders, imposing penalties, and recovering any taxes or duties due. Hence statement 2 is incorrect. It is responsible for gathering intelligence about potential violations of the GST law. This includes collecting information from various sources, such as GST returns, financial statements, and other documents. It develops intelligence, especially in new areas of tax evasion through its intelligence network across the country and disseminates such information, by issuing Modus Operandi Circulars and Alert Circulars to sensitize the field formations about the latest trends in duty evasion. It is entrusted with the task of collection, collation, and dissemination of intelligence relating to the evasion of Goods and Services Tax (GST) and the duties of Central Excise and Service Tax on an all-India basis. Hence statement 3 is correct. Incorrect Solution (b) The Directorate General of GST Intelligence (DGGI) is an apex intelligence organization functioning under the Central Board of Indirect Taxes and Customs, Department of Revenue, and Ministry of Finance. Hence statement 1 is correct. The Directorate General of Central Excise Intelligence (DGCEI) is now renamed as Directorate General of GST Intelligence (DGGI). It has the power to conduct investigations into suspected cases of GST evasion or non-compliance. This may involve summoning persons, examining records, and carrying out searches and seizures. It is responsible for enforcing the provisions of the GST law. This includes taking legal action against offenders, imposing penalties, and recovering any taxes or duties due. Hence statement 2 is incorrect. It is responsible for gathering intelligence about potential violations of the GST law. This includes collecting information from various sources, such as GST returns, financial statements, and other documents. It develops intelligence, especially in new areas of tax evasion through its intelligence network across the country and disseminates such information, by issuing Modus Operandi Circulars and Alert Circulars to sensitize the field formations about the latest trends in duty evasion. It is entrusted with the task of collection, collation, and dissemination of intelligence relating to the evasion of Goods and Services Tax (GST) and the duties of Central Excise and Service Tax on an all-India basis. Hence statement 3 is correct. Question 25 of 35 25. Question Consider the following statements regarding ABHYAS: It is designed for autonomous flying with the help of an autopilot indigenously. It is designed by the Defence Research and Development Organisation’s (DRDO) Aeronautical Development Establishment (ADE). It has a radar cross-section and a visual and infrared augmentation system required  for weapon practice. How many of the above statements are correct? a) Only one b) Only two c) All three d) None Correct Solution (c) ABHYAS is designed for autonomous flying with the help of an autopilot indigenously. Hence statement 1 is correct. It offers a realistic threat scenario for the practice of weapon systems. It is the ideal platform for the validation of Armed Forces equipment slated for induction (only those that require aerial engagement). Hence statement 2 is correct. It is designed by the Defence Research and Development Organisation’s (DRDO) Aeronautical Development Establishment (ADE). The target drone has a laptop-based Ground Control System with which the aircraft can be integrated and pre-flight checks, data recording during the flight, replays after the flight, and post-flight analysis can be carried out. It has a radar cross-section and a visual and infrared augmentation system required for weapon practice. Hence statement 3 is correct. Incorrect Solution (c) ABHYAS is designed for autonomous flying with the help of an autopilot indigenously. Hence statement 1 is correct. It offers a realistic threat scenario for the practice of weapon systems. It is the ideal platform for the validation of Armed Forces equipment slated for induction (only those that require aerial engagement). Hence statement 2 is correct. It is designed by the Defence Research and Development Organisation’s (DRDO) Aeronautical Development Establishment (ADE). The target drone has a laptop-based Ground Control System with which the aircraft can be integrated and pre-flight checks, data recording during the flight, replays after the flight, and post-flight analysis can be carried out. It has a radar cross-section and a visual and infrared augmentation system required for weapon practice. Hence statement 3 is correct. Question 26 of 35 26. Question Consider the following statements regarding the Index of Industrial Production (IIP): It is published annually by the National Statistical Office (NSO). It uses mining, manufacturing, and electricity as broad sectors. It uses basic goods, capital goods, and intermediate goods as base sectors. How many of the above statements are correct? a) Only one b) Only two c) All three d) None Correct Solution (b) The Index of Industrial Production (IIP) is published monthly by the National Statistical Office (NSO), Ministry of Statistics and Programme Implementation. Hence statement 1 is incorrect. It uses 2011-2012 as its base year. It uses mining, manufacturing, and electricity as broad sectors. Hence statement 2 is correct. The relative weights of these three sectors are 6% (manufacturing), 14.4% (mining), and 8% (electricity). Eight core industries comprise about 40 percent of the weight of items included in the IIP. They are electricity, crude oil, coal, cement, steel, refinery products, natural gas, and fertilizers. The eight core sector industries in decreasing order of their weightage: Refinery Products> Electricity> Steel> Coal> Crude Oil> Natural Gas> Cement> Fertilizers. It uses basic goods, capital goods, and intermediate goods as base sectors. Hence statement 3 is correct. Incorrect Solution (b) The Index of Industrial Production (IIP) is published monthly by the National Statistical Office (NSO), Ministry of Statistics and Programme Implementation. Hence statement 1 is incorrect. It uses 2011-2012 as its base year. It uses mining, manufacturing, and electricity as broad sectors. Hence statement 2 is correct. The relative weights of these three sectors are 6% (manufacturing), 14.4% (mining), and 8% (electricity). Eight core industries comprise about 40 percent of the weight of items included in the IIP. They are electricity, crude oil, coal, cement, steel, refinery products, natural gas, and fertilizers. The eight core sector industries in decreasing order of their weightage: Refinery Products> Electricity> Steel> Coal> Crude Oil> Natural Gas> Cement> Fertilizers. It uses basic goods, capital goods, and intermediate goods as base sectors. Hence statement 3 is correct. Question 27 of 35 27. Question The Red Sea is an arm of the Indian Ocean, lying between Africa and Asia. It is bordered by which of the following countries? Israel Egypt Jordan Sudan Yemen Choose the correct code: a) Only two b) Only three c) Only four d) All five Correct Solution (d) The Red Sea is an arm of the Indian Ocean, lying between Africa and Asia. It is bordered by the following countries: Eastern shore: Israel, Jordan, Saudi Arabia and Yemen. Western shore: Egypt, Sudan, Eritrea, Djibouti. Hence option d is correct. It is connected to the ocean in the south, through the Bab-el-Mandeb strait and the Gulf of Aden. To its north lie the Sinai Peninsula, the Gulf of Aqaba, and the Gulf of Suez (leading to the Suez Canal). The Red Sea is connected to the Mediterranean Sea through the Suez Canal. The Red Sea is one of the most densely packed shipping channels in the world, carrying maritime traffic between Europe and Asia and east Africa. About 12% of global trade passes through the Red Sea, including 30% of global container traffic. The Red Sea also serves as a strategic zone for both regional and Great Powers projecting their military might or openly engaging in conflict. Incorrect Solution (d) The Red Sea is an arm of the Indian Ocean, lying between Africa and Asia. It is bordered by the following countries: Eastern shore: Israel, Jordan, Saudi Arabia and Yemen. Western shore: Egypt, Sudan, Eritrea, Djibouti. Hence option d is correct. It is connected to the ocean in the south, through the Bab-el-Mandeb strait and the Gulf of Aden. To its north lie the Sinai Peninsula, the Gulf of Aqaba, and the Gulf of Suez (leading to the Suez Canal). The Red Sea is connected to the Mediterranean Sea through the Suez Canal. The Red Sea is one of the most densely packed shipping channels in the world, carrying maritime traffic between Europe and Asia and east Africa. About 12% of global trade passes through the Red Sea, including 30% of global container traffic. The Red Sea also serves as a strategic zone for both regional and Great Powers projecting their military might or openly engaging in conflict. Question 28 of 35 28. Question Consider the following statements regarding the ‘Aldabra Giant Tortoise’: It is the largest species of land tortoise in the world. It is endemic to the Aldabra Atoll of the Seychelles. Its largest populations are found on grasslands called “platins”. It is listed as vulnerable on the IUCN Red List. How many of the above statements are correct? a) Only one b) Only two c) Only three d) All four Correct Solution (c) The Aldabra Giant Tortoise is the second-largest species of land tortoise in the world. Hence statement 1 is incorrect. The Galapagos Giant Tortoise is the largest species of land tortoise in the world. It can live for 100 years and has a fascinating history. This tortoise evolved from the ancestors of Aldabrachelys abrupta, one of two giant tortoises that inhabited Madagascar for 15 million years. Four million years ago, the Aldabrachelys abrupta lineage migrated, likely via a combination of drifting with floating vegetation and assisted by their natural buoyancy and good swimming abilities, to Seychelles. It is endemic to the Aldabra Atoll of the Seychelles,  an archipelago nation in the western Indian Ocean about 930 miles east of Africa and northeast of Madagascar. Hence statement 2 is correct. Its largest populations are found on grasslands called “platins”. Hence statement 3 is correct. These are terrestrial and occur in a wide variety of habitats, including scrub forests, mangrove swamps, and coastal dunes and beaches, each with their respective vegetation. It is listed as vulnerable on the IUCN Red List. Hence statement 4 is correct. It is listed in Appendix II of CITES. Incorrect Solution (c) The Aldabra Giant Tortoise is the second-largest species of land tortoise in the world. Hence statement 1 is incorrect. The Galapagos Giant Tortoise is the largest species of land tortoise in the world. It can live for 100 years and has a fascinating history. This tortoise evolved from the ancestors of Aldabrachelys abrupta, one of two giant tortoises that inhabited Madagascar for 15 million years. Four million years ago, the Aldabrachelys abrupta lineage migrated, likely via a combination of drifting with floating vegetation and assisted by their natural buoyancy and good swimming abilities, to Seychelles. It is endemic to the Aldabra Atoll of the Seychelles,  an archipelago nation in the western Indian Ocean about 930 miles east of Africa and northeast of Madagascar. Hence statement 2 is correct. Its largest populations are found on grasslands called “platins”. Hence statement 3 is correct. These are terrestrial and occur in a wide variety of habitats, including scrub forests, mangrove swamps, and coastal dunes and beaches, each with their respective vegetation. It is listed as vulnerable on the IUCN Red List. Hence statement 4 is correct. It is listed in Appendix II of CITES. Question 29 of 35 29. Question Consider the following statements regarding the World Sustainable Development (WSDS) Summit: It is the annual flagship event of The Energy and Resources Institute (TERI). The theme of WSDS 2024 is ‘Mainstreaming Sustainable Development and Climate Resilience for Collective Action’. Choose the correct code: a) 1 only b) 2 only c) Both 1 and 2 d) Neither 1 nor 2 Correct Solution (a) The World Sustainable Development (WSDS) Summit is the annual flagship event of The Energy and Resources Institute (TERI). Hence statement 1 is correct. It is the only independently convened international summit on sustainable development and environment, based in the Global South. It strives to provide long-term solutions for the benefit of global communities by assembling the world’s most enlightened leaders and thinkers on a single platform. The theme of WSDS 2024 is ‘Leadership for the Sustainable Development and Climate Justice’. Hence statement 2 is incorrect. Note: TERI is a leading think tank dedicated to conducting research for sustainable development of India and the Global South. It is an independent, multi-dimensional organization, with capabilities in research, policy, consultancy, and implementation.   Incorrect Solution (a) The World Sustainable Development (WSDS) Summit is the annual flagship event of The Energy and Resources Institute (TERI). Hence statement 1 is correct. It is the only independently convened international summit on sustainable development and environment, based in the Global South. It strives to provide long-term solutions for the benefit of global communities by assembling the world’s most enlightened leaders and thinkers on a single platform. The theme of WSDS 2024 is ‘Leadership for the Sustainable Development and Climate Justice’. Hence statement 2 is incorrect. Note: TERI is a leading think tank dedicated to conducting research for sustainable development of India and the Global South. It is an independent, multi-dimensional organization, with capabilities in research, policy, consultancy, and implementation.   Question 30 of 35 30. Question The 3M22 Zircon, or SS-N-33, is a scramjet-powered manoeuvring anti-ship hypersonic cruise missile. It is developed by which of the following country? a) The United States b) France c) Russia d) China Correct Solution (c) The 3M22 Zircon, or SS-N-33, is a scramjet-powered manoeuvring anti-ship hypersonic cruise missile. It is developed by Russia. Hence option c is correct. It is reportedly capable of flying with speeds up to Mach 9 and a range of 1000 km. It’s a two-stage missile that uses solid fuel in the first stage and a scramjet motor in the second stage. Zircon cruise missiles use cooled supersonic combustion ramjet engines, which power combustion by compressing air flowing at supersonic speeds due to the missile’s forward motion.   Incorrect Solution (c) The 3M22 Zircon, or SS-N-33, is a scramjet-powered manoeuvring anti-ship hypersonic cruise missile. It is developed by Russia. Hence option c is correct. It is reportedly capable of flying with speeds up to Mach 9 and a range of 1000 km. It’s a two-stage missile that uses solid fuel in the first stage and a scramjet motor in the second stage. Zircon cruise missiles use cooled supersonic combustion ramjet engines, which power combustion by compressing air flowing at supersonic speeds due to the missile’s forward motion.   Question 31 of 35 31. Question Find the lowest 4-digit number which when divided by 3, 4 or 5 leaves a remainder of 2 in each case? a) 1020 b) 1026 c) 1022 d) 1030 Correct Solution (c) Lowest 4-digit number is 1000. LCM of 3, 4 and 5 = 3 × 4 × 5 = 60. Dividing 1000 by 60, we get the remainder 40. Thus, the lowest 4-digit number that exactly divisible by 3, 4 and 5 is 1000 + (60 – 40) = 1020. Now, add the remainder 2 that’s required. Thus, the answer is (1020 + 2 = 1022). Solution (c) Lowest 4-digit number is 1000. LCM of 3, 4 and 5 = 3 × 4 × 5 = 60. Dividing 1000 by 60, we get the remainder 40. Thus, the lowest 4-digit number that exactly divisible by 3, 4 and 5 is 1000 + (60 – 40) = 1020. Now, add the remainder 2 that’s required. Thus, the answer is (1020 + 2 = 1022). Incorrect Solution (c) Lowest 4-digit number is 1000. LCM of 3, 4 and 5 = 3 × 4 × 5 = 60. Dividing 1000 by 60, we get the remainder 40. Thus, the lowest 4-digit number that exactly divisible by 3, 4 and 5 is 1000 + (60 – 40) = 1020. Now, add the remainder 2 that’s required. Thus, the answer is (1020 + 2 = 1022). Solution (c) Lowest 4-digit number is 1000. LCM of 3, 4 and 5 = 3 × 4 × 5 = 60. Dividing 1000 by 60, we get the remainder 40. Thus, the lowest 4-digit number that exactly divisible by 3, 4 and 5 is 1000 + (60 – 40) = 1020. Now, add the remainder 2 that’s required. Thus, the answer is (1020 + 2 = 1022). Question 32 of 35 32. Question The average weight of a group of 5 boys is 26. If we replace a boy of weight 20 in the group with another boy so that new average increases by 3.8 kgs, find the weight of the new boy. a) 56 b) 39 c) 40 d) 33 Correct Solution (b) The weight of new boy = Weight of the removed boy + No. of boys ×Increase in average. = 20+5×3.8 =20+19 = 39 kgs Incorrect Solution (b) The weight of new boy = Weight of the removed boy + No. of boys ×Increase in average. = 20+5×3.8 =20+19 = 39 kgs Question 33 of 35 33. Question A man whose bowling average is 12.4 takes 5 wickets for 26 runs and thereby decreases his average by 0.4. The number of wickets, taken by him, before his last match is a) 85 b) 78 c) 72 d) 64 Correct Solution (a) The bowling average indicates the number of runs given by a bowler to take a wicket. Let the bowler of 12.4 bowling average has taken x wickets before his last match. So, number of runs conceded by him are 12.4x. Number of runs conceded including present match are 12.4x+26 and total wickets taken are x+5 and the new bowling average is 12.4−0.4=12. To find x, Bowling average = Number of runs conceded / Number of wickets taken ​ or, 12.4x+26 /x+5 ​ =12 or,x=85 Therefore number of wickets taken by him his last match is 85     Incorrect Solution (a) The bowling average indicates the number of runs given by a bowler to take a wicket. Let the bowler of 12.4 bowling average has taken x wickets before his last match. So, number of runs conceded by him are 12.4x. Number of runs conceded including present match are 12.4x+26 and total wickets taken are x+5 and the new bowling average is 12.4−0.4=12. To find x, Bowling average = Number of runs conceded / Number of wickets taken ​ or, 12.4x+26 /x+5 ​ =12 or,x=85 Therefore number of wickets taken by him his last match is 85     Question 34 of 35 34. Question In a conference, out of a total 80 participants, 65 are Indians. If 40 of the total participants are vegetarian, then which of the following statements is/are correct? At least 25 Indian participants are vegetarian. At least 25 Indian participants are non- vegetarian.  Select the correct answer using the codes given below:   a) 1 only b) 2 only c) Both 1 and 2 d) Neither 1 nor 2 Correct Solution (c) Let’s try to maximise the number of Indian-Vegetarians. Out of 65 Indians, all vegetarians (i.e., 40) can be Indians. So, at least 25 Indians will be there who will be non- vegetarians. This number can increase depending on the number of vegetarian-Indians. Let’s try to minimise the number of Indian-Vegetarians. For that we have maximise the number of non-Indian-Vegetarians. Out of 15 Non-Indians, at max all can be vegetarian. Still 25 vegetarians remain which will fall under Indian category. So, at least 25 Indians will be there who will be vegetarians. Hence both statements are correct. Incorrect Solution (c) Let’s try to maximise the number of Indian-Vegetarians. Out of 65 Indians, all vegetarians (i.e., 40) can be Indians. So, at least 25 Indians will be there who will be non- vegetarians. This number can increase depending on the number of vegetarian-Indians. Let’s try to minimise the number of Indian-Vegetarians. For that we have maximise the number of non-Indian-Vegetarians. Out of 15 Non-Indians, at max all can be vegetarian. Still 25 vegetarians remain which will fall under Indian category. So, at least 25 Indians will be there who will be vegetarians. Hence both statements are correct. Question 35 of 35 35. Question Shamanth and Manoj go for a swim after a gap of every 3 days and every 4 days respectively. If on 1st January both of them went for a swim together, when will they go together next?   a) 7th January b) 8th January c) 21st January d) 13th January Correct Solution (d) Shamanth goes to swim every 3th day and Manoj goes to swim every 4th day. They will go to swim together every 12th day (LCM of 3 and 4 is 12). So, on the 12th day after January 1, they will go together to swim i.e., on January 13.   Incorrect Solution (d) Shamanth goes to swim every 3th day and Manoj goes to swim every 4th day. They will go to swim together every 12th day (LCM of 3 and 4 is 12). So, on the 12th day after January 1, they will go together to swim i.e., on January 13.   window.wpProQuizInitList = window.wpProQuizInitList || []; window.wpProQuizInitList.push({ id: '#wpProQuiz_3670', init: { quizId: 3670, mode: 1, globalPoints: 70, timelimit: 1800, resultsGrade: [0], bo: 704, qpp: 0, catPoints: [70], formPos: 0, lbn: "Test-summary", json: {"32862":{"type":"single","id":32862,"catId":0,"points":2,"correct":[0,1,0,0]},"32863":{"type":"single","id":32863,"catId":0,"points":2,"correct":[0,1,0,0]},"32866":{"type":"single","id":32866,"catId":0,"points":2,"correct":[0,1,0,0]},"32868":{"type":"single","id":32868,"catId":0,"points":2,"correct":[1,0,0,0]},"32871":{"type":"single","id":32871,"catId":0,"points":2,"correct":[0,0,1,0]},"32873":{"type":"single","id":32873,"catId":0,"points":2,"correct":[0,1,0,0]},"32874":{"type":"single","id":32874,"catId":0,"points":2,"correct":[0,0,1,0]},"32876":{"type":"single","id":32876,"catId":0,"points":2,"correct":[0,1,0,0]},"32877":{"type":"single","id":32877,"catId":0,"points":2,"correct":[0,0,1,0]},"32878":{"type":"single","id":32878,"catId":0,"points":2,"correct":[0,1,0,0]},"32881":{"type":"single","id":32881,"catId":0,"points":2,"correct":[1,0,0,0]},"32883":{"type":"single","id":32883,"catId":0,"points":2,"correct":[0,1,0,0]},"32887":{"type":"single","id":32887,"catId":0,"points":2,"correct":[1,0,0,0]},"32889":{"type":"single","id":32889,"catId":0,"points":2,"correct":[0,0,1,0]},"32892":{"type":"single","id":32892,"catId":0,"points":2,"correct":[0,1,0,0]},"32893":{"type":"single","id":32893,"catId":0,"points":2,"correct":[0,1,0,0]},"32895":{"type":"single","id":32895,"catId":0,"points":2,"correct":[0,1,0,0]},"32897":{"type":"single","id":32897,"catId":0,"points":2,"correct":[0,0,0,1]},"32898":{"type":"single","id":32898,"catId":0,"points":2,"correct":[0,1,0,0]},"32899":{"type":"single","id":32899,"catId":0,"points":2,"correct":[0,1,0,0]},"32902":{"type":"single","id":32902,"catId":0,"points":2,"correct":[1,0,0,0]},"32904":{"type":"single","id":32904,"catId":0,"points":2,"correct":[0,0,0,1]},"32905":{"type":"single","id":32905,"catId":0,"points":2,"correct":[0,1,0,0]},"32908":{"type":"single","id":32908,"catId":0,"points":2,"correct":[0,1,0,0]},"32910":{"type":"single","id":32910,"catId":0,"points":2,"correct":[0,0,1,0]},"32913":{"type":"single","id":32913,"catId":0,"points":2,"correct":[0,1,0,0]},"32914":{"type":"single","id":32914,"catId":0,"points":2,"correct":[0,0,0,1]},"32915":{"type":"single","id":32915,"catId":0,"points":2,"correct":[0,0,1,0]},"32918":{"type":"single","id":32918,"catId":0,"points":2,"correct":[1,0,0,0]},"32920":{"type":"single","id":32920,"catId":0,"points":2,"correct":[0,0,1,0]},"32922":{"type":"single","id":32922,"catId":0,"points":2,"correct":[0,0,1,0]},"32925":{"type":"single","id":32925,"catId":0,"points":2,"correct":[0,1,0,0]},"32928":{"type":"single","id":32928,"catId":0,"points":2,"correct":[1,0,0,0]},"32930":{"type":"single","id":32930,"catId":0,"points":2,"correct":[0,0,1,0]},"32933":{"type":"single","id":32933,"catId":0,"points":2,"correct":[0,0,0,1]}} } }); All the Best IASbaba

DAILY CURRENT AFFAIRS IAS | UPSC Prelims and Mains Exam – 23rd April 2024

Archives (PRELIMS & MAINS Focus)   Indelible ink used in election Syllabus Prelims – Current Event Context: With the first phase of voting for the 2024 Lok Sabha elections beginning on April 19, the classic symbol of Indian polls is visible everywhere – a left hand with only its index finger extended, marked by a purple-black indelible ink. Background:- Devised as a method to prevent a person from casting more than one vote, the indelible ink has been used for decades in India. It has also travelled to other parts of the world to help in conducting elections. About the Indelible ink used in elections The Section 61 of Representation of the People Act (RoPA) of 1951 mentions the ink. It states that rules may be made under the Act “for the marking with indelible ink of the thumb or any other finger of every elector who applies for a ballot paper or ballot papers for the purpose of voting at a polling station before delivery of such paper or papers to him.” Why papers? Because before EVMs, ballot papers existed at polling stations. They contained the list of contesting candidates alongside their party symbols and names. Voters had to mark their preference on the paper and then drop it in ballot boxes to vote. Indelible ink contains silver nitrate. It is a colourless compound which becomes visible when exposed to ultraviolet light, including sunlight. The water-based ink also contains a solvent like alcohol to allow its faster drying. Silver nitrate is an inorganic compound with the chemical formula AgNO3.The higher silver nitrate’s concentration, say around 20 percent, the higher will be the ink’s quality, according to a report from the United Nations Development Programme. For up to 72 hours after application it can remain resistant to soap, liquids, home-cleansing, detergents, etc. The indelible ink was first manufactured at the ECI’s request by the government’s Council of Scientific & Industrial Research (CSIR). It was later patented by the National Research Development Corporation (NRDC), New Delhi. Mysore Paints & Varnish Ltd. has been licensed to manufacture the ink and has been in the business since 1962. Earlier called Mysore Lac & Paint Works Ltd, it was established in 1937 by Nalwadi Krishnaraja Wodeyar, then the Maharaja of Mysore. This Karnataka Government Undertaking is the sole manufacturer of the ink in India. Today, once a voter has her credentials checked at the polling booth, and before she casts her vote by pressing a button on the Electronic Voting Machine (EVM), the indelible ink is applied on her finger. This has been the case for decades, although the mode of voting has changed. Currently, the indelible ink is exported to more than 25 countries that include Canada, Ghana, Nigeria, Mongolia, Malaysia, Nepal, South Africa and the Maldives. Source: Hindu Fourth global mass coral bleaching Syllabus Prelims & Mains – Environment Context: The fourth global mass coral bleaching event has been triggered by extraordinary ocean temperatures, the US National Oceanic and Atmospheric Administration (NOAA) said on Monday (April 15). This could have serious consequences for ocean life and millions of people who rely on reefs for food, jobs, and coastal defence. Background: Since mid-March 2023, the average sea surface temperature (SST) has been abnormally high. What are corals and coral reefs? Corals are essentially animals, which are sessile, meaning they permanently attach themselves to the ocean floor. They use their tiny tentacle-like hands to catch food from the water and sweep into their mouth. Each individual coral animal is known as a polyp and it lives in groups of hundreds to thousands of genetically identical polyps that form a ‘colony’. Corals are largely classified as either hard coral or soft coral. It is the hard corals that are the architects of coral reefs — complex three-dimensional structures built up over thousands of years. “Unlike soft corals, hard corals have stony skeletons made out of limestone that are produced by coral polyps. When polyps die, their skeletons are left behind and used as foundations for new polyps,” according to NOAA. Coral reefs, also referred to as “rainforests of the sea”, have existed on the Earth for nearly 450 million years. Australia’s Great Barrier Reef is the largest in the world, stretching across 2,028 kilometres. What is the significance of corals? Coral reefs have a crucial role in marine ecosystems. Thousands of marine species can be found living on one reef. For instance, the Great Barrier Reef contains over 400 coral species, 1,500 fish species, 4,000 mollusc species and six of the world’s seven sea turtle species. Research has shown that there could be millions of undiscovered species of organisms living in and around reefs. These massive structures also provide economic goods and services worth about $375 billion each year. More than 500 million people across the world depend on coral reefs for food, income and coastal protection from storms and floods. Coral reefs can absorb up to 97% of the energy from waves, storms, and floods, which prevents loss of life, property damage, and soil erosion. Therefore, the absence of coral reefs would not only result in severe ramifications for marine life but also for humans. What is coral bleaching? Most corals contain algae called zooxanthellae — they are plant-like organisms — in their tissues. Corals and zooxanthellae have a symbiotic relationship. While corals provide zooxanthellae a safe place to live, zooxanthellae provide oxygen and organic products of photosynthesis that help corals to grow and thrive. Zooxanthellae also give bright and unique colours to corals. Corals are very sensitive to light and temperature and even a small change in their living conditions can stress them. When stressed, they expel zooxanthellae and turn entirely white. This is called coral bleaching. Coral bleaching doesn’t immediately lead to the death of corals. They rather go under more stress and are subject to mortality. Coral bleaching reduces the reproductivity of corals and makes them more vulnerable to fatal diseases. If the bleaching is not too severe, corals have been known to recover. Global mass bleaching of coral reefs is when significant coral bleaching is confirmed in the Atlantic, Indian and Pacific oceans, according to a report published by The Conversation. Such events are a relatively new phenomenon. The first one took place in 1998 in which 20% of the world’s reef areas suffered bleaching-level heat stress. The next two global bleaching events occurred in 2010 (35% of reefs affected) and between 2014 and 2017 (56% of reefs affected). What is happening right now? NOAA has confirmed that the fourth global bleaching event is currently underway. Nearly 54 countries, territories and local economies — from Florida, the US, Saudi Arabia to Fiji — have confirmed bleaching, according to a report by The New York Times. The Great Barrier Reef is witnessing its most severe bleaching event. “About a third of the reefs surveyed by air showed prevalence of very high or extreme bleaching, and at least three quarters showed some bleaching,” the NYT report said. On Monday (April 15), bleaching was also confirmed in the Western Indian Ocean, including Tanzania, Kenya, Mauritius, Seychelles, and off the western coast of Indonesia. The key driver behind the current event is higher ocean temperatures. However, the situation has been exacerbated by El Niño, a weather pattern which is associated with warmer oceans. Given that El Niño is weakening and a cooler La Niña may set in by the end of the year, the event may not last for very long. Source: Indian Express Previous Year Question Q) Consider the following statements: Most of the world’s coral reefs are in tropical waters. More than one-third of the world’s coral reefs are located in territories of Australia, Indonesia and Philippines. Coral reefs host far more number of animal phyla than those hosted by tropical rainforests. Which of the statements given above is/are correct? 1 and 2 only 3 only 1 and 3 only 1,2 and 3 NATIONAL ORGAN AND TISSUE TRANSPLANT ORGANISATION (NOTTO) Syllabus Prelims – Current Event Context: As per the recent directive by the Union Health Ministry, all cases of organ transplants will now be assigned a unique National Organ and Tissue Transplant Organisation (NOTTO)-ID for both the donor and the recipient. Background: This initiative aims to streamline and enhance the process of organ allocation and transplantation across the country. About NATIONAL ORGAN AND TISSUE TRANSPLANT ORGANISATION (NOTTO): The National Organ and Tissue Transplant Organisation (NOTTO) is the apex body in India for the coordination and networking of organ and tissue procurement and transplantation. It operates under the Directorate General of Health Services, Ministry of Health and Family Welfare, Government of India. NOTTO also oversees the National Human Organ and Tissue Removal and Storage Network and the National Biomaterial Centre (National Tissue Bank), which are crucial for maintaining the supply and quality of transplantable tissues. Primary Functions: Coordination and Networking: Facilitating the procurement and distribution of organs and tissues across the country. Registry Maintenance: Keeping a registry of organ and tissue donations and transplants. Policy and Protocol Development: Establishing guidelines, protocols, and standard operating procedures for tissue procurement, distribution, and transplantation. Training and Assistance: Providing training and assisting in the registration of other tissue banks. Quality Assurance: Ensuring the quality of tissues available for transplantation. Public Awareness: Promoting organ and tissue donation awareness. Source: Hindu IRAN Syllabus Prelims – Geography Context: Iran recently launched a series of air strikes on Israel, employing 200-300 drones and ballistic missiles. Background: The attacks, executed by Iran’s para-military force, the Iranian Revolutionary Guard Corps, were in response to Israeli war jets targeting an Iranian consulate in Syria. Iran dubbed the attack “Operation True Promise”. About Iran : Iran, officially known as the Islamic Republic of Iran, is situated in Western Asia. The vast majority of Iranians adhere to Islam. Specifically, they follow the Ithnā ʿAsharī (Twelver) branch of Shi’a Islam, which is the official state religion. Iran has one of the longest land borders of any country in the region, covering approximately 3,662 miles. Iran shares this extensive land border with seven countries: Iraq: To the west. Turkmenistan: To the north. Afghanistan: To the east. Pakistan: To the southeast. Turkey: To the northwest. Armenia: To the north. Azerbaijan: To the north. Additionally, Iran has maritime borders with several countries, including Bahrain, Kuwait, Oman, Qatar, Saudi Arabia, and the United Arab Emirates. Source: Hindu MOUNT RUANG VOLCANO Syllabus Prelims – Geography Context: Indonesia witnessed a series of eruptions from Mount Ruang. Background: At least 16 eruptions have been recorded from Mount Ruang, with the first one occurring in 1808. About MOUNT RUANG VOLCANO: Mount Ruang is situated in Indonesia’s North Sulawesi province. It is a stratovolcano, characterized by a tall, steep, cone-shaped structure. Stratovolcanoes are known for their explosive eruptions due to the build-up of gas pressures in their more viscous lavas. The volcano is part of the Pacific Ring of Fire, which is prone to frequent seismic and volcanic activity due to its position on subduction zones. Source: Times of India Global Methane Tracker 2024 Syllabus Prelims & Mains – Environment Context: The International Energy Agency’s Global Methane Tracker 2024 indicates that methane emissions from fuel usage in 2023 were nearly at their highest level on record, representing a slight increase compared to 2022. Background: Methane is responsible for around 30 per cent of the rise in global temperatures since the preindustrial era. The energy sector— including oil, natural gas, coal and bioenergy — accounts for over a third of methane emissions from human activity. Key highlights of the Global Methane Tracker 2024: In 2023, methane emissions from fossil fuels totalled close to 120 million tonnes (Mt). Bioenergy (largely from biomass use) contributed a further 10 Mt methane emissions. This level has stayed constant since 2019. Major methane emissions events increased by over 50% in 2023 compared to 2022. These events included more than 5 million metric tons of methane emissions from significant fossil fuel leaks globally. One prominent incident was a major well blowout in Kazakhstan that lasted over 200 days. Nearly 70% of methane emissions from fossil fuels come from the top 10 emitting countries. The United States is the largest emitter of methane from oil and gas operations, closely followed by Russia. China is the highest emitter of methane in the coal sector. Cutting methane emissions from fossil fuels by 75% by 2030 is crucial for limiting global warming to 1.5 °C. The IEA estimated that this goal would require about USD 170 billion in spending. This is less than 5% of the income generated by the fossil fuel industry in 2023. Around 40% of emissions from fossil fuels in 2023 could have been avoided at no net cost. Methane: It is the simplest hydrocarbon, consisting of one carbon atom and four hydrogen atoms (CH4). It is the primary component of natural gas. It is odourless, colourless, and tasteless gas. It is lighter than air and burns with a blue flame in complete combustion, yielding carbon dioxide (CO2) and water (H2O) in the presence of oxygen. Methane ranks as the second most important greenhouse gas (GHG) after carbon dioxide (CO2). Its 20-year global warming potential (GWP) is 84, indicating that it traps 84 times more heat per mass unit than CO2 over a 20-year period, making it a potent GHG. It is a significant contributor to global warming, accounting for about 30% of the rise in global temperatures since the preindustrial era. It contributes to the formation of ground-level ozone. Major sources of Methane Emission: Wetlands, both natural and human-made, are significant sources of methane emissions due to anaerobic decomposition of organic matter. Growing paddy fields release methane due to anaerobic conditions in flooded rice paddies. Excreta from cattle and other livestock undergo enteric fermentation, producing methane as a byproduct. Burning of fossil fuels, including oil and natural gas, releases methane emissions. Biomass burning, such as wood and agricultural residues, also contributes to methane levels. Industrial activities like landfills and wastewater treatment plants generate methane during organic waste decomposition in anaerobic environments. Fertilizer factories and other industrial processes can also release methane during production and transportation. Source: Down To Earth Practice MCQs Daily Practice MCQs Q.1) With reference to the National Organ and Tissue Transplant Organisation (NOTTO), consider the following statement: NOTTO is the apex body in India for the coordination and networking of organ and tissue procurement and transplantation. It operates under the Directorate General of Health Services, Ministry of Health and Family Welfare. Which of the statements given above are not correct? 1 Only 2 Only Both 1 and 2 Neither 1 nor 2 Q2.) Consider the following countries: Iraq Israel Jordan Pakistan Turkey Armenia How many of the above-mentioned countries share a land border with Iran? Only two Only three Only four Only five Q3.)  With reference to the Mount Ruang, consider the following statements: It is a stratovolcano situated in Italy. It is a part of the Pacific Ring of Fire. Which of the statements given above is/are correct? 1 only 2 only Both 1 and 2 Neither 1 nor Comment the answers to the above questions in the comment section below!! ANSWERS FOR ’  23rd April  2024 – Daily Practice MCQs’ will be updated along with tomorrow’s Daily Current Affairs.st ANSWERS FOR  22nd April – Daily Practice MCQs Answers- Daily Practice MCQs Q.1) – d Q.2) – c Q.3) – a

Daily Prelims CA Quiz

UPSC Quiz – 2024 : IASbaba’s Daily Current Affairs Quiz 23rd April 2024

The Current Affairs questions are based on sources like ‘The Hindu’, ‘Indian Express’ and ‘PIB’, which are very important sources for UPSC Prelims Exam. The questions are focused on both the concepts and facts. The topics covered here are generally different from what is being covered under ‘Daily Current Affairs/Daily News Analysis (DNA) and Daily Static Quiz’ to avoid duplication. The questions would be published from Monday to Saturday before 2 PM. One should not spend more than 10 minutes on this initiative. Gear up and Make the Best Use of this initiative. Do remember that, “the difference between Ordinary and EXTRA-Ordinary is PRACTICE!!” Important Note: Don’t forget to post your marks in the comment section. Also, let us know if you enjoyed today’s test 🙂After completing the 5 questions, click on ‘View Questions’ to check your score, time taken, and solutions.To take the Test Click Here

[DAY 44] 60 DAY RAPID REVISION (RaRe) SERIES for UPSC Prelims 2024 – GEOGRAPHY, CURRENT AFFAIRS & CSAT TEST SERIES!

Archives Hello Friends The 60 Days Rapid Revision (RaRe) Series is IASbaba’s Flagship Initiative recommended by Toppers and loved by the aspirants’ community every year. It is the most comprehensive program which will help you complete the syllabus, revise and practice tests on a daily basis. The Programme on a daily basis includes Daily Prelims MCQs from Static (Monday – Saturday) Daily Static Quiz will cover all the topics of static subjects – Polity, History, Geography, Economics, Environment and Science and technology. 20 questions will be posted daily and these questions are framed from the topics mentioned in the schedule. It will ensure timely and streamlined revision of your static subjects. Daily Current Affairs MCQs (Monday – Saturday) Daily 5 Current Affairs questions, based on sources like ‘The Hindu’, ‘Indian Express’ and ‘PIB’, would be published from Monday to Saturday according to the schedule. Daily CSAT Quiz (Monday – Friday) CSAT has been an Achilles heel for many aspirants. Daily 5 CSAT Questions will be published. Note – Daily Test of 20 static questions, 10 current affairs, and 5 CSAT questions. (35 Prelims Questions) in QUIZ FORMAT will be updated on a daily basis. To Know More about 60 Days Rapid Revision (RaRe) Series – CLICK HERE   60 Day Rapid Revision (RaRe) Series Schedule – CLICK HERE  Important Note Comment your Scores in the Comment Section. This will keep you accountable, responsible and sincere in days to come. It will help us come out with the Cut-Off on a Daily Basis. Let us know if you enjoyed today’s test 🙂  You can post your comments in the given format  (1) Your Score (2) Matrix Meter (3) New Learning from the Test Time limit: 0 Test-summary 0 of 35 questions completed Questions: 1 2 3 4 5 6 7 8 9 10 11 12 13 14 15 16 17 18 19 20 21 22 23 24 25 26 27 28 29 30 31 32 33 34 35 Information The following Test is based on the syllabus of 60 Days Plan-2023 for UPSC IAS Prelims 2022. To view Solutions, follow these instructions: Click on – ‘Start Test’ button Solve Questions Click on ‘Test Summary’ button Click on ‘Finish Test’ button Now click on ‘View Questions’ button – here you will see solutions and links. You have already completed the test before. Hence you can not start it again. Test is loading... You must sign in or sign up to start the test. You have to finish following test, to start this test: Results 0 of 35 questions answered correctly Your time: Time has elapsed You have scored 0 points out of 0 points, (0) Average score     Your score     Categories Not categorized 0% Your result has been entered into leaderboard Loading Name: E-Mail: Captcha: maximum of 70 points Pos. Name Entered on Points Result Table is loading No data available 1 2 3 4 5 6 7 8 9 10 11 12 13 14 15 16 17 18 19 20 21 22 23 24 25 26 27 28 29 30 31 32 33 34 35 Answered Review Question 1 of 35 1. Question Consider the following statements regarding demographic transition theory: It can be used to predict the future population of any area. The lower birth and death rates are characteristics of urban, industrial, and literate societies. It explains demographic transition in two stages. How many of the above statements are correct? a) Only one b) Only two c) All three d) None Correct Solution (b) Statement 1 Statement 2 Statement 3 Correct Correct Incorrect Demographic transition theory can be used to describe and predict the future population of any area. The theory tells us that the population of any region changes from high births and high deaths to low births and low deaths as society progresses from rural agrarian and illiterate to urban industrial and literate society. These changes occur in the stages which are collectively known as the demographic cycle. Three Stage demographic transition theory: The first stage has high fertility and high mortality because people produce more to compensate for the deaths due to epidemics and variable food supply. The population growth is slow and most of the people are engaged in agriculture where large families are an asset. Life expectancy is low, people are mostly illiterate and have low levels of technology. Second Stage: The fertility rate remains high at the beginning of the second stage but it declines with time. This is accompanied by a reduced mortality rate. Improvements in sanitation and health conditions lead to a decline in mortality. Because of this gap, the net addition to the population is high. In the last stage, both fertility and mortality decline considerably. The population is either stable or grows slowly. The population becomes urbanized, literate and has the high technical know-how, and deliberately controls the family size. Incorrect Solution (b) Statement 1 Statement 2 Statement 3 Correct Correct Incorrect Demographic transition theory can be used to describe and predict the future population of any area. The theory tells us that the population of any region changes from high births and high deaths to low births and low deaths as society progresses from rural agrarian and illiterate to urban industrial and literate society. These changes occur in the stages which are collectively known as the demographic cycle. Three Stage demographic transition theory: The first stage has high fertility and high mortality because people produce more to compensate for the deaths due to epidemics and variable food supply. The population growth is slow and most of the people are engaged in agriculture where large families are an asset. Life expectancy is low, people are mostly illiterate and have low levels of technology. Second Stage: The fertility rate remains high at the beginning of the second stage but it declines with time. This is accompanied by a reduced mortality rate. Improvements in sanitation and health conditions lead to a decline in mortality. Because of this gap, the net addition to the population is high. In the last stage, both fertility and mortality decline considerably. The population is either stable or grows slowly. The population becomes urbanized, literate and has the high technical know-how, and deliberately controls the family size. Question 2 of 35 2. Question Which of the following statements is/ are correct regarding different forms of Urbanization? Conurbation: It is essentially a metropolitan area including an urban region and its adjacent towns and suburbs. Counter-urbanization: The outward growth of towns and cities to engulf surrounding villages and rural areas. Select the correct answer using the code given below. a) 1 only b) 2 only c) Both 1 and 2 d) Neither 1 nor 2 Correct Solution (a) Urbanization Urbanization means the increase in the proportion population of a country who lives in urban areas. The most important cause of urbanization is rural-urban migration. Statement 1 Statement 2 Correct Incorrect The term conurbation was coined by Patrick Geddes in 1915 and applied to a large area of urban development that resulted from the merging of originally separate towns Counter-urbanization is a process involving the movement of population away from inner urban areas to a new town, a new estate, a commuter town or a village on the edge or just beyond the city limits or rural-urban fringe. Incorrect Solution (a) Urbanization Urbanization means the increase in the proportion population of a country who lives in urban areas. The most important cause of urbanization is rural-urban migration. Statement 1 Statement 2 Correct Incorrect The term conurbation was coined by Patrick Geddes in 1915 and applied to a large area of urban development that resulted from the merging of originally separate towns Counter-urbanization is a process involving the movement of population away from inner urban areas to a new town, a new estate, a commuter town or a village on the edge or just beyond the city limits or rural-urban fringe. Question 3 of 35 3. Question Which among the following is/ are the indicator(s) used in calculating the Multi-Dimensional Poverty Index? Years of schooling Nutrition level Sanitation Drinking Water Type of cooking fuel Electricity Select the correct answer using the code given below: a) 1, 2 and 3 only b) 1, 2, 3 and 6 only c) 1, 2, 3, 5 and 6 only d) 1, 2, 3, 4, 5 and 6 Correct Solution (d) Multi-dimensional Poverty Index The Multi-dimensional Poverty Index(MPI) has been developed by the Oxford Poverty and Human Development Initiative (OPHI) for the United Nations Development Programme . It is composed of ten indicators. Years of schooling and child enrollment (education); Child mortality and nutrition (health) Electricity, flooring, drinking water, sanitation, cooking fuel and assets (standard of living). Each education and health indicator has a 1/6 weight, each standard. So all the above-mentioned indicators are used in the Multi-dimensional Poverty Index (MPI). Incorrect Solution (d) Multi-dimensional Poverty Index The Multi-dimensional Poverty Index(MPI) has been developed by the Oxford Poverty and Human Development Initiative (OPHI) for the United Nations Development Programme . It is composed of ten indicators. Years of schooling and child enrollment (education); Child mortality and nutrition (health) Electricity, flooring, drinking water, sanitation, cooking fuel and assets (standard of living). Each education and health indicator has a 1/6 weight, each standard. So all the above-mentioned indicators are used in the Multi-dimensional Poverty Index (MPI). Question 4 of 35 4. Question There are various problems associated with urbanisation in India. So, which of the following recommendations can be incorporated for mitigating the adverse effects of urbanization in India? Activity mapping for local bodies. Populist policies to increase community participation Municipal bonds as the ratio of own revenue User charge fees for water supply Centralized planning Select the correct answer using the code given below: a) 1, 2 and 3 only b) 1, 3 and 4 only c) 2, 4 and 5 only d) 1, 2, 3, 4 and 5 Correct Solution (b) Urbanization According to the census 2011 data 31% of the total population of India lives in urban areas. About 60% population of the country will live in cities by 2050 at current India’s rate of urbanization.   Statement 1 Statement 2 Statement 3 Statement 4 Statement 5 Correct Incorrect Correct Correct Incorrect The use of geographical information system to map all the properties in a city can have a huge impact on the assessment rate of properties that are not in the tax net Populist policies instead worsen the adverse effects of urbanization Municipal bonds are also famous, which work on the concept of pooled financing. There is a need to broaden the user charge fee for water supply, sewerage and garbage disposal. For financing urban projects The decentralized approach has proven to be helpful in mitigating the adverse impacts of urbanization. Incorrect Solution (b) Urbanization According to the census 2011 data 31% of the total population of India lives in urban areas. About 60% population of the country will live in cities by 2050 at current India’s rate of urbanization.   Statement 1 Statement 2 Statement 3 Statement 4 Statement 5 Correct Incorrect Correct Correct Incorrect The use of geographical information system to map all the properties in a city can have a huge impact on the assessment rate of properties that are not in the tax net Populist policies instead worsen the adverse effects of urbanization Municipal bonds are also famous, which work on the concept of pooled financing. There is a need to broaden the user charge fee for water supply, sewerage and garbage disposal. For financing urban projects The decentralized approach has proven to be helpful in mitigating the adverse impacts of urbanization. Question 5 of 35 5. Question Consider the following statements: In this type of village, the general living area is distinct and separated from surrounding farms and pastures. The close build-up area and its intervening streets present some recognizable pattern or geometric shape. This type of settlement sometimes forms for defence and security reasons. Such settlements are generally found in fertile alluvial plains. The above statements represent which of the following type of rural settlements in India? a) Hamletted settlement b) Dispersed settlement c) Semi-clustered settlement d) Clustered settlement. Correct Solution (d) Statement 1 Statement 2 Statement 3 Statement 4 Incorrect Incorrect Incorrect Correct Hamletted Settlements: Sometimes settlement is fragmented into several units physically separated from each other bearing a common name. These units are locally called Panna, Para, Palli, Nagla, Dhani, etc. in various parts of the country. Dispersed or isolated settlement pattern in India appears in the form of isolated huts or hamlets of few huts in remote jungles, or on small hills with farms or pasture on the slopes. Extreme dispersion of settlement is often caused by the extremely fragmented nature of the terrain and land resource base of habitable areas. Semi-clustered or fragmented settlements may result from tendency of clustering in a restricted area of dispersed settlement. More often such a pattern may also result from the segregation or fragmentation of a large compact village. The clustered rural settlement is a compact or closely built-up area of houses. In this type of village, the general living area is distinct and separated from the surrounding farms, barns and pastures. The closely built-up area and its intervening streets present some recognizable pattern or geometric shape, such as rectangular, radial, linear, etc. Such settlements are generally found in fertile alluvial plains and in the northeastern states. Sometimes, people live in a compact village for security or defence reasons, such as in the Bundelkhand region of central India and in Nagaland In Rajasthan, scarcity of water has necessitated compact settlement for maximum utilization of available water resources. Incorrect Solution (d) Statement 1 Statement 2 Statement 3 Statement 4 Incorrect Incorrect Incorrect Correct Hamletted Settlements: Sometimes settlement is fragmented into several units physically separated from each other bearing a common name. These units are locally called Panna, Para, Palli, Nagla, Dhani, etc. in various parts of the country. Dispersed or isolated settlement pattern in India appears in the form of isolated huts or hamlets of few huts in remote jungles, or on small hills with farms or pasture on the slopes. Extreme dispersion of settlement is often caused by the extremely fragmented nature of the terrain and land resource base of habitable areas. Semi-clustered or fragmented settlements may result from tendency of clustering in a restricted area of dispersed settlement. More often such a pattern may also result from the segregation or fragmentation of a large compact village. The clustered rural settlement is a compact or closely built-up area of houses. In this type of village, the general living area is distinct and separated from the surrounding farms, barns and pastures. The closely built-up area and its intervening streets present some recognizable pattern or geometric shape, such as rectangular, radial, linear, etc. Such settlements are generally found in fertile alluvial plains and in the northeastern states. Sometimes, people live in a compact village for security or defence reasons, such as in the Bundelkhand region of central India and in Nagaland In Rajasthan, scarcity of water has necessitated compact settlement for maximum utilization of available water resources. Question 6 of 35 6. Question Which of the following is/are push factors for migration? Unpleasant climate Natural disasters and epidemics Better job opportunities Peace and stability Select the correct answer using the code given below. a) 1 and 2 only b) 3 and 4 only c) 3 only d) 1, 2, 3 and 4 Correct Solution (a) Statement 1 and Statement 2 Statement 3 and Statement 4 Correct Incorrect Push factors are negative things that make people want to move to a new area. Reasons include less economic opportunities, poor living conditions, political turmoil, unpleasant climate, epidemics, natural disasters, etc. Pull factors are positive aspects that attract people to move to a place. It includes better job opportunities and living conditions, peace and stability, security of life and property, and a pleasant climate. Incorrect Solution (a) Statement 1 and Statement 2 Statement 3 and Statement 4 Correct Incorrect Push factors are negative things that make people want to move to a new area. Reasons include less economic opportunities, poor living conditions, political turmoil, unpleasant climate, epidemics, natural disasters, etc. Pull factors are positive aspects that attract people to move to a place. It includes better job opportunities and living conditions, peace and stability, security of life and property, and a pleasant climate. Question 7 of 35 7. Question Consider the following pairs: Expansive pyramid – Populations with a large number of adults and a longer average life expectancy. Constrictive pyramid – Populations with high fertility rates and Below-average life expectancies. Stationary pyramids – Very even age/ sex distribution. How many of the above pairs are correctly matched? a) Only one b) Only two c) Only three d) None Correct Solution (a) Statement 1 Statement 2 Statement 3 Incorrect Incorrect Correct These types of pyramids are usually found in populations with very large fertility rates and lower-than-average life expectancies. The age-sex distributions of Latin America and many Third World countries would probably display expansive population pyramids. These pyramids typically reflect populations with a large number of adults and a longer average life expectancy, like those found in the United States and some European countries. These pyramids usually reflect nations with a very even age/sex distribution. Several European countries fall under this population Incorrect Solution (a) Statement 1 Statement 2 Statement 3 Incorrect Incorrect Correct These types of pyramids are usually found in populations with very large fertility rates and lower-than-average life expectancies. The age-sex distributions of Latin America and many Third World countries would probably display expansive population pyramids. These pyramids typically reflect populations with a large number of adults and a longer average life expectancy, like those found in the United States and some European countries. These pyramids usually reflect nations with a very even age/sex distribution. Several European countries fall under this population Question 8 of 35 8. Question Which of the following statements is/ are correct regarding the Census of India Act, 1948? Under this act central government can ask for the service of any citizen for census work. Every citizen of a country has to compulsorily answer the questions as a part of the census truthfully and honestly. It has provisions for penalties to punish false answers or not giving answers to the census questionnaire. How many of the above statements are correct? a) Only one b) Only two c) All three d) None Correct Solution (c) Statement 1 Statement 2 Statement 3 Correct Correct Correct Census of India Act, 1948 It gives the central government powers to notify a date for census and to ask for the service of any citizen for census work. The act lays down those services of any citizen can be requisitioned for census work The Census of India Act, 1948 obligatory on every person occupying a house, enclosure etc. to allow access to census officers, It makes it compulsory for every citizen of a country to answer the questions as a part of census truthfully and honestly. It has provisions for penalties to punish false answers or not giving answers to the census questionnaire. According to it, the census records are not to be inspected and also, they are not admissible as evidence. Incorrect Solution (c) Statement 1 Statement 2 Statement 3 Correct Correct Correct Census of India Act, 1948 It gives the central government powers to notify a date for census and to ask for the service of any citizen for census work. The act lays down those services of any citizen can be requisitioned for census work The Census of India Act, 1948 obligatory on every person occupying a house, enclosure etc. to allow access to census officers, It makes it compulsory for every citizen of a country to answer the questions as a part of census truthfully and honestly. It has provisions for penalties to punish false answers or not giving answers to the census questionnaire. According to it, the census records are not to be inspected and also, they are not admissible as evidence. Question 9 of 35 9. Question Which of the following is/are the government deficits in urban governance? Fragmented functional assignments resulting in unclear accountability at the city level. Inappropriately targeted inter-governmental transfers resulting in inadequate local government financial resources. Insufficient provision and maintenance of municipal services and networks. Accountability deficit in transparency. Select the correct answer using the code given below: a) 1, 2 and 4 only b) 2, 3 and 4 only c) 3 and 4 only d) 1, 2, 3 and 4 Correct Solution (d) According to the World Bank, the challenges in urban governance have arisen from the following governance deficits: Statement 1 Statement 2 Statement 3 Statement 4 Correct Correct Correct Correct Empowerment deficit: Limited, overlapping and fragmented functional assignments resulting in unclear accountability at the city level. Resource deficit: Limited revenue-generation powers and inappropriately targeted intergovernmental transfers resulting in inadequate local government financial resources Delivery deficit: Insufficient provision and maintenance of municipal services and networks. Accountability deficit: Despite elections and the right to information, transparency structures, roles and mandates are unclear.   Incorrect Solution (d) According to the World Bank, the challenges in urban governance have arisen from the following governance deficits: Statement 1 Statement 2 Statement 3 Statement 4 Correct Correct Correct Correct Empowerment deficit: Limited, overlapping and fragmented functional assignments resulting in unclear accountability at the city level. Resource deficit: Limited revenue-generation powers and inappropriately targeted intergovernmental transfers resulting in inadequate local government financial resources Delivery deficit: Insufficient provision and maintenance of municipal services and networks. Accountability deficit: Despite elections and the right to information, transparency structures, roles and mandates are unclear.   Question 10 of 35 10. Question Consider the following pairs: Age sex pyramid – Representative Population Triangular Pyramid with flat base: Expanding population Bell shaped pyramid: Declining population Narrow base with tapered top: Constant population How many of the above pairs are correctly matched? a) Only one b) Only two c) Only three d) None Correct Solution (a) Age-Sex Pyramid: The age-sex structure of a population refers to the number of females and males in different age groups. A population pyramid is used to show the age-sex structure of the population. The shape of the population pyramid reflects the characteristics of the population. The left side shows the percentage of males while the right side shows the percentage of women in each age group. Statement 1 Statement 2 Statement 3 Correct Incorrect Incorrect Expanding Populations: The age-sex pyramid of Nigeria for example is a triangular shaped pyramid with a wide base and is typical of less developed countries. These have larger populations in lower age groups due to high birth rates. Ex: Bangladesh and Mexico etc. Constant Population: In the case of Australia age-sex pyramid is bell shaped and tapered towards the top. This shows birth and death rates are almost equal leading to a near constant population. Declining Populations: The Japan pyramid has a narrow base and a tapered top showing low birth and death rates. The population growth in developed countries is usually zero or negative. Incorrect Solution (a) Age-Sex Pyramid: The age-sex structure of a population refers to the number of females and males in different age groups. A population pyramid is used to show the age-sex structure of the population. The shape of the population pyramid reflects the characteristics of the population. The left side shows the percentage of males while the right side shows the percentage of women in each age group. Statement 1 Statement 2 Statement 3 Correct Incorrect Incorrect Expanding Populations: The age-sex pyramid of Nigeria for example is a triangular shaped pyramid with a wide base and is typical of less developed countries. These have larger populations in lower age groups due to high birth rates. Ex: Bangladesh and Mexico etc. Constant Population: In the case of Australia age-sex pyramid is bell shaped and tapered towards the top. This shows birth and death rates are almost equal leading to a near constant population. Declining Populations: The Japan pyramid has a narrow base and a tapered top showing low birth and death rates. The population growth in developed countries is usually zero or negative. Question 11 of 35 11. Question Majority of the population in India resides in rural areas. In this context consider the following statements associated with the concept of Rural settlements: Dispersed rural settlement demands that the farmer should live on his own land. Caste structure or a functional need of people decides the shape and size of rural settlements. A compact settlement is found on the highly productive alluvial plains of the country. How many of the above statements are correct? a) Only one b) Only two c) All three d) None Correct Solution (c) As per the census, 2011 data about 69% of the total population in India resides in rural areas. Rural settlements are of 3 types: Compact, Semi-compact and dispersed. Statement 1 Statement 2 Statement 3 Correct Correct Correct Dispersed settlements are generally found in hills, plateaus and grasslands. These are found in areas where it is essential that the farmer should live on his own land. Overpopulation is one of the reasons for dispersed settlement. Compact settlement is based on farming. These are mostly found in highly productive alluvial plains like Indo-Gangetic Plains, the Hwang Ho Valley and the Nile valley.   Socio-cultural factors like the caste structure of the people living in a village and the functional needs of the people also have a close bearing on the shapes and size of rural settlements. Incorrect Solution (c) As per the census, 2011 data about 69% of the total population in India resides in rural areas. Rural settlements are of 3 types: Compact, Semi-compact and dispersed. Statement 1 Statement 2 Statement 3 Correct Correct Correct Dispersed settlements are generally found in hills, plateaus and grasslands. These are found in areas where it is essential that the farmer should live on his own land. Overpopulation is one of the reasons for dispersed settlement. Compact settlement is based on farming. These are mostly found in highly productive alluvial plains like Indo-Gangetic Plains, the Hwang Ho Valley and the Nile valley.   Socio-cultural factors like the caste structure of the people living in a village and the functional needs of the people also have a close bearing on the shapes and size of rural settlements. Question 12 of 35 12. Question Which of the following statements best represents the features and pattern of Indian urbanization? Lopsided urbanization induces the growth of class I cities. High decadal growth in urban population is attributed to large-scale migration from rural to urban area. In terms of the absolute number of persons living in urban areas, the maximum lives in Maharashtra state. How many of the above statements are correct? a) Only one b) Only two c) All three d) None Correct Solution (c) Statement 1 Statement 2 Statement 3 Correct Correct Correct Following are the some basic feature of Urbanization in India: Lopsided urbanization induces the growth of class I cities. Urbanization occurs without industrialization and a strong economic base. Urbanization is mainly a product of demographic explosion and poverty-induced rural-urban migration. Rapid urbanization leads to the massive growth of slums followed by misery, poverty, unemployment, exploitation, inequalities, and degradation in the quality of urban life. Urbanization occurs not due to urban pull but due to rural push. Poor quality of rural-urban migration leads to poor quality of urbanization. Distress migration initiates urban decay. The decadal growth of 2001-11 suggests that there has been 31.80% growth in the urban population in contrast to only 12.18% in the rural population during the same time. This high urban growth is attributed to the large-scale rural-urban migration. Statement 3 is correct: In terms of the absolute number of persons living in urban areas, Maharashtra continues to lead with 50.8 million persons which comprise 13.5 per cent of the total urban population of the country. Uttar Pradesh accounts for about 44.4 million, followed by Tamil Nadu at 34.9 million Incorrect Solution (c) Statement 1 Statement 2 Statement 3 Correct Correct Correct Following are the some basic feature of Urbanization in India: Lopsided urbanization induces the growth of class I cities. Urbanization occurs without industrialization and a strong economic base. Urbanization is mainly a product of demographic explosion and poverty-induced rural-urban migration. Rapid urbanization leads to the massive growth of slums followed by misery, poverty, unemployment, exploitation, inequalities, and degradation in the quality of urban life. Urbanization occurs not due to urban pull but due to rural push. Poor quality of rural-urban migration leads to poor quality of urbanization. Distress migration initiates urban decay. The decadal growth of 2001-11 suggests that there has been 31.80% growth in the urban population in contrast to only 12.18% in the rural population during the same time. This high urban growth is attributed to the large-scale rural-urban migration. Statement 3 is correct: In terms of the absolute number of persons living in urban areas, Maharashtra continues to lead with 50.8 million persons which comprise 13.5 per cent of the total urban population of the country. Uttar Pradesh accounts for about 44.4 million, followed by Tamil Nadu at 34.9 million Question 13 of 35 13. Question Consider the following pairs related to rural settlement patterns: Settlement pattern                       Regions found Linear settlement pattern – along mountain ranges Rectangular Pattern – an agricultural area Radial pattern – for defence purpose How many of the above pairs are correctly matched? a) Only one b) Only two c) Only three d) None Correct Solution (a) Statement 1 Statement 2 Statement 3 Incorrect Correct Incorrect Linear pattern: In such settlement’s houses are located along a road, railway line, river, canal edge of a valley or along a levee. Rectangular area: Such patterns of rural settlements are found in plain areas, agricultural area and wide intermontane valleys. The roads are rectangular and cut each other at right angles. Radial Pattern: In this type, a number of streets converge in one centre which may be a source of water (pond, well), a temple or mosque, a centre of commercial activity or simply an open space. Incorrect Solution (a) Statement 1 Statement 2 Statement 3 Incorrect Correct Incorrect Linear pattern: In such settlement’s houses are located along a road, railway line, river, canal edge of a valley or along a levee. Rectangular area: Such patterns of rural settlements are found in plain areas, agricultural area and wide intermontane valleys. The roads are rectangular and cut each other at right angles. Radial Pattern: In this type, a number of streets converge in one centre which may be a source of water (pond, well), a temple or mosque, a centre of commercial activity or simply an open space. Question 14 of 35 14. Question India is home to the three big religions of the world: Hindu, Islam and Christianity, and ‘other religions. In light of this, consider the following statements: India has the highest percentage of the Muslim population in the world. India has about 99% of the Hindu population of the world. India has the largest share of all members of the ‘other religions of the world. How many of the above statements are correct? a) Only one b) Only two c) All three d) None Correct Solution (a) Statement 1 Statement 2 Statement 3 Incorrect Incorrect Correct India has the 2nd percentage of the Muslim population in the world after Indonesia. 13% live in Indonesia and 11% live in India. 3 countries India, Mauritius and Nepal together have 97% of the Hindu population of the world. One interesting fact is that Nepal’s 81.3% population is Hindu while India’s 80.5% population is Hindu. India has the largest share of all members of other religions of the world (47%) including Sikhs and Jains. Incorrect Solution (a) Statement 1 Statement 2 Statement 3 Incorrect Incorrect Correct India has the 2nd percentage of the Muslim population in the world after Indonesia. 13% live in Indonesia and 11% live in India. 3 countries India, Mauritius and Nepal together have 97% of the Hindu population of the world. One interesting fact is that Nepal’s 81.3% population is Hindu while India’s 80.5% population is Hindu. India has the largest share of all members of other religions of the world (47%) including Sikhs and Jains. Question 15 of 35 15. Question Which of the following pairs of tribes is/are correctly matched? Malasar tribe – Andhra Pradesh Jatapu tribes – Kerala PahadiKorva tribes – Chhattisgarh Asur tribe – Jharkhand How many of the above pairs are correctly matched? a) Only one b) Only two c) All three d) None Correct Solution (b) Statement 1 Statement 2 Statement 3 Statement 4 Incorrect Incorrect Correct Correct Malasar Tribe – Kerala Malasar is a designated Scheduled Tribe in the Indian states of Kerala and Tamil Nadu. The Malasar are one of the earliest known inhabitants of the Western Ghats, in Anaimalai Hills. Malasar is an unclassified Southern Dravidian language spoken by a Scheduled tribe of India. Jatapu Tribe – Andhra Pradesh The Jatapu people are designated Scheduled Tribe in the Indian states of Andhra Pradesh and Odisha Jatapus are an Adivasi tribe and are traditionally pastoral farmers. Through acculturation, the Jatapus speak Telugu and have in many ways adopted the culture of the surrounding Telugu people. PahadiKorva tribes Chattisgarh The Korwa people are a Munda, a Scheduled Tribe ethnic group of India. They live mainly on the border between Chhattisgarh and Jharkhand. A small number of Korwa are also found in the Mirzapur district of Uttar Pradesh. The Government has implemented several facilities for them, such as roads to their settlements, boys’ hostels for education, providing agricultural aid, etc. They are a hunter-gatherer community. The tribe is divided into several subdivisions: the Agaria, Dandh, Dil and Pahadi Koreas Asur tribe Jharkhand Asur people are a very small Austroasiatic ethnic group living primarily in the Indian state of Jharkhand, mostly in the Gumla, Lohardaga, Palamu and Latehar districts. They speak Asur language, which belongs to Munda family of Austro-asiatic languages Incorrect Solution (b) Statement 1 Statement 2 Statement 3 Statement 4 Incorrect Incorrect Correct Correct Malasar Tribe – Kerala Malasar is a designated Scheduled Tribe in the Indian states of Kerala and Tamil Nadu. The Malasar are one of the earliest known inhabitants of the Western Ghats, in Anaimalai Hills. Malasar is an unclassified Southern Dravidian language spoken by a Scheduled tribe of India. Jatapu Tribe – Andhra Pradesh The Jatapu people are designated Scheduled Tribe in the Indian states of Andhra Pradesh and Odisha Jatapus are an Adivasi tribe and are traditionally pastoral farmers. Through acculturation, the Jatapus speak Telugu and have in many ways adopted the culture of the surrounding Telugu people. PahadiKorva tribes Chattisgarh The Korwa people are a Munda, a Scheduled Tribe ethnic group of India. They live mainly on the border between Chhattisgarh and Jharkhand. A small number of Korwa are also found in the Mirzapur district of Uttar Pradesh. The Government has implemented several facilities for them, such as roads to their settlements, boys’ hostels for education, providing agricultural aid, etc. They are a hunter-gatherer community. The tribe is divided into several subdivisions: the Agaria, Dandh, Dil and Pahadi Koreas Asur tribe Jharkhand Asur people are a very small Austroasiatic ethnic group living primarily in the Indian state of Jharkhand, mostly in the Gumla, Lohardaga, Palamu and Latehar districts. They speak Asur language, which belongs to Munda family of Austro-asiatic languages Question 16 of 35 16. Question Consider the following statements concerning mineral reserves in India The peninsular rocks contain most of the reserves of coal in India Sedimentary rocks on the Western and Eastern flanks of the peninsula, in Gujarat and Assam have most of the petroleum deposits in India The vast alluvial plains of North India contain huge reserves of economic minerals How many of the above statements are correct? a) Only one b) Only two c) All three d) None Correct Solution (b) India is fortunate to have fairly rich and varied mineral resources. However, these are unevenly distributed. Statement 1 Statement 2 Statement 3 Correct Correct Incorrect Peninsular rocks contain most of the reserves of coal(mostly bituminous) metallic minerals, mica, and many other non-metallic minerals. Sedimentary rocks on the Western and Eastern flanks of the peninsula, in Gujarat and Assam, have most of the petroleum deposits. In Assam petroleum reserves can be found around the Digboi region and  Ankleshwar and Khambhat are two major areas in western India. The vast alluvial plains of North India are almost devoid of economic minerals. These variations exist largely because of the differences in the geological structure, processes, and time involved in the formation of minerals. Also the alluvial plains being the youngest geological structure keeps getting covered with alluvium leaving very little scope for Mineral formation. Incorrect Solution (b) India is fortunate to have fairly rich and varied mineral resources. However, these are unevenly distributed. Statement 1 Statement 2 Statement 3 Correct Correct Incorrect Peninsular rocks contain most of the reserves of coal(mostly bituminous) metallic minerals, mica, and many other non-metallic minerals. Sedimentary rocks on the Western and Eastern flanks of the peninsula, in Gujarat and Assam, have most of the petroleum deposits. In Assam petroleum reserves can be found around the Digboi region and  Ankleshwar and Khambhat are two major areas in western India. The vast alluvial plains of North India are almost devoid of economic minerals. These variations exist largely because of the differences in the geological structure, processes, and time involved in the formation of minerals. Also the alluvial plains being the youngest geological structure keeps getting covered with alluvium leaving very little scope for Mineral formation. Question 17 of 35 17. Question The process of ‘sub-urbanisation’ is related to: a) Mushrooming of slums in urban areas. b) Construction of planned gated communities within urban areas for middle- and upper-class people. c) People moving away from congested urban areas to cleaner areas outside the city for better quality of living. d) Emergence of mega cities or megalopolis. Correct Solution (c) Statement 1 Statement 2 Statement 3 Statement 4 Incorrect Incorrect Correct Incorrect This is referred to as Ghettoization. This refers to an isolated and underprivileged urban area. It refers to the segregation/isolation of a group and placement of that group into a figurative or literal position of little power which leads to mushrooming if urban settlements. A gated community (or walled community) is a form of residential community or housing estate containing strictly controlled entrances for pedestrians, bicycles, and automobiles, and often characterized by a closed perimeter of walls and fences. Sub Urbanisation is a new trend of people moving away from congested urban areas to cleaner areas outside the city in search of a better quality of living. Important suburbs develop around major cities and everyday thousands of people commute from their homes in the suburbs to their work places in the city. Like in case of Delhi, people residing in Gautam Buddha Nagar (Noida) and commuting everyday to work in their offices based in Delhi. Megalopolis: This Greek word meaning “great city”, was popularised by Jean Gottman (1957) and signifies ‘super- metropolitan’ region extending, as union of conurbations. The urban landscape stretching from Boston in the north to south of Washington in U.S.A. is the best known example of a megalopolis. Incorrect Solution (c) Statement 1 Statement 2 Statement 3 Statement 4 Incorrect Incorrect Correct Incorrect This is referred to as Ghettoization. This refers to an isolated and underprivileged urban area. It refers to the segregation/isolation of a group and placement of that group into a figurative or literal position of little power which leads to mushrooming if urban settlements. A gated community (or walled community) is a form of residential community or housing estate containing strictly controlled entrances for pedestrians, bicycles, and automobiles, and often characterized by a closed perimeter of walls and fences. Sub Urbanisation is a new trend of people moving away from congested urban areas to cleaner areas outside the city in search of a better quality of living. Important suburbs develop around major cities and everyday thousands of people commute from their homes in the suburbs to their work places in the city. Like in case of Delhi, people residing in Gautam Buddha Nagar (Noida) and commuting everyday to work in their offices based in Delhi. Megalopolis: This Greek word meaning “great city”, was popularised by Jean Gottman (1957) and signifies ‘super- metropolitan’ region extending, as union of conurbations. The urban landscape stretching from Boston in the north to south of Washington in U.S.A. is the best known example of a megalopolis. Question 18 of 35 18. Question Consider the following pairs Mines        Minerals Kudremukh – Bauxite Khetri – Copper Sundergarh – Iron Bilaspur – Manganese Which of the statements given above is/are correct? a) 1 only b) 1 and 2 only c) 1, 2 and 3 d) 2 only Correct Solution (d) Statement 1 Statement 2 Statement 3 Statement 4 Incorrect Correct Incorrect Incorrect The Ballari-Chitradurga-Chikkamagaluru-Tumakuru belt in Karnataka has large reserves of iron ore. The Kudremukh mines located in the Western Ghats of Karnataka are a 100 percent export unit. Kudremukh deposits are known to be one of the largest in the world. The ore is transported as slurry through a pipeline to a port near Mangaluru.   The Balaghat mines in Madhya Pradesh, the Khetri mines in Rajasthan and the Singhbhum district of Jharkhand are leading producers of copper. Sundergarh mines in the state of Odisha is one of the important mines for manganese deposits. India’s bauxite deposits are mainly found in the Amarkantak plateau, Maikal hills, and the plateau region of Bilaspur-Katni Incorrect Solution (d) Statement 1 Statement 2 Statement 3 Statement 4 Incorrect Correct Incorrect Incorrect The Ballari-Chitradurga-Chikkamagaluru-Tumakuru belt in Karnataka has large reserves of iron ore. The Kudremukh mines located in the Western Ghats of Karnataka are a 100 percent export unit. Kudremukh deposits are known to be one of the largest in the world. The ore is transported as slurry through a pipeline to a port near Mangaluru.   The Balaghat mines in Madhya Pradesh, the Khetri mines in Rajasthan and the Singhbhum district of Jharkhand are leading producers of copper. Sundergarh mines in the state of Odisha is one of the important mines for manganese deposits. India’s bauxite deposits are mainly found in the Amarkantak plateau, Maikal hills, and the plateau region of Bilaspur-Katni Question 19 of 35 19. Question Which of the following statements is/are a correct representation of the population density pattern of the world? Physiological density refers to the total population of an area divided by total arable land. North-East USA and Europe, and South and Southeast Asia have high population densities. Rainfall zones of the equator and West Australia represent a sparse population density pattern. How many of the above statements are correct? a) Only one b) Only two c) All three d) None Correct Solution (c) Population density refers to the ratio of the number of people to the size of land.  95% of the world’s population is concentrated on just 10% of the world’s land. Arithmetic Density- Total Population of an area divided by the total area of the land they inhabit. Statement 1 Statement 2 Statement 3 Correct Correct Correct Physiological density = Total population of area divided by total arable land Northeast USA has a very high population density. While the Southeast specialized in cultivating cash crops (especially cotton) for export to Europe, the Northeast industrialized, and its cities grew rapidly. The Equator and West Australia represent sparse population density patterns. Incorrect Solution (c) Population density refers to the ratio of the number of people to the size of land.  95% of the world’s population is concentrated on just 10% of the world’s land. Arithmetic Density- Total Population of an area divided by the total area of the land they inhabit. Statement 1 Statement 2 Statement 3 Correct Correct Correct Physiological density = Total population of area divided by total arable land Northeast USA has a very high population density. While the Southeast specialized in cultivating cash crops (especially cotton) for export to Europe, the Northeast industrialized, and its cities grew rapidly. The Equator and West Australia represent sparse population density patterns. Question 20 of 35 20. Question Consider the following statements regarding the ‘Placer deposits’ These are the minerals that occur as alluvial deposits in the sands of valley floors and the base of hills These deposits generally contain minerals, which are not corroded by water Tin is one of the important minerals found in Placer deposits How many of the above statements are correct? a) Only one b) Only two c) All three d) None Correct Solution (c) Explanation: Minerals generally occur in various forms In igneous and metamorphic rocks minerals may occur in the cracks, crevices, faults, or joints. In most cases, they are formed when minerals in liquid/ molten and gaseous forms are forced upward through cavities toward the earth’s surface. In sedimentary rocks several minerals occur in beds or layers. They have been formed as a result of deposition, accumulation, and concentration in horizontal strata. Other groups of sedimentary minerals include gypsum, potash salt, and sodium salt. These are formed as a result of evaporation, especially in arid regions. Statement 1 Statement 2 Statement 3 Correct Correct Correct Placer Deposits minerals occur as alluvial deposits in the sands of valley floors and the base of the hills. Minerals that form placer deposits have high specific gravity, are chemically resistant to weathering, and are durable. The deposits  called ‘placer deposits’ generally contain minerals, which are not corroded by water The most important among such minerals are such minerals include gold, platinum, cassiterite, magnetite, chromite, ilmenite, rutile, native copper, zircon, monazite, tin  and various gemstones.   Incorrect Solution (c) Explanation: Minerals generally occur in various forms In igneous and metamorphic rocks minerals may occur in the cracks, crevices, faults, or joints. In most cases, they are formed when minerals in liquid/ molten and gaseous forms are forced upward through cavities toward the earth’s surface. In sedimentary rocks several minerals occur in beds or layers. They have been formed as a result of deposition, accumulation, and concentration in horizontal strata. Other groups of sedimentary minerals include gypsum, potash salt, and sodium salt. These are formed as a result of evaporation, especially in arid regions. Statement 1 Statement 2 Statement 3 Correct Correct Correct Placer Deposits minerals occur as alluvial deposits in the sands of valley floors and the base of the hills. Minerals that form placer deposits have high specific gravity, are chemically resistant to weathering, and are durable. The deposits  called ‘placer deposits’ generally contain minerals, which are not corroded by water The most important among such minerals are such minerals include gold, platinum, cassiterite, magnetite, chromite, ilmenite, rutile, native copper, zircon, monazite, tin  and various gemstones.   Question 21 of 35 21. Question Which of the following is correct regarding the ‘Wheat Blast Disease’ (WB)? a) It is a highly destructive bacterial infection that primarily affects wheat crops. b) It is a highly destructive fungal infection that primarily affects wheat crops. c) It is a highly destructive viral infection that primarily affects wheat crops. d) None Correct Solution (b) Wheat blast disease is a highly destructive fungal infection that primarily affects wheat crops. It is caused by the fungus Magnaporthe oryzae pathotype Triticum (MoT). It poses a significant threat to food security and safety, particularly in tropical regions of South America and South Asia. Hence option b is correct.   Incorrect Solution (b) Wheat blast disease is a highly destructive fungal infection that primarily affects wheat crops. It is caused by the fungus Magnaporthe oryzae pathotype Triticum (MoT). It poses a significant threat to food security and safety, particularly in tropical regions of South America and South Asia. Hence option b is correct.   Question 22 of 35 22. Question Consider the following statements regarding the 42nd Amendment Act of 1976: It added three new words – socialist, secular, and integrity in the Preamble. It provided for administrative tribunals and tribunals for other matters. It curtailed the power of judicial review and writ jurisdiction of the Supreme Court and high courts. It shifted five subjects from the state list to the concurrent list – education, forests, protection of wild animals and birds, weights and measures and administration of justice, constitution, and organisation of all courts except the Supreme Court and the high courts. How many of the above statements are correct? a) Only one b) Only two c) Only three d) All four Correct Solution (d) The 42nd Amendment Act of 1976 made the following provisions: It added three new words – socialist, secular, and integrity in the Preamble. It provided for administrative tribunals and tribunals for other matters. It curtailed the power of judicial review and writ jurisdiction of the Supreme Court and high courts. It shifted five subjects from the state list to the concurrent list – education, forests, protection of wild animals and birds, weights and measures and administration of justice, constitution, and organisation of all courts except the Supreme Court and the high courts. It empowered the Parliament to decide from time to time the rights and privileges of its members and committees. It provided for the creation of the All-India Judicial Service. It shortened the procedure for disciplinary action by taking away the right of a civil servant to make representation at the second stage after the inquiry (i.e., on the penalty proposed). It empowered the centre to deploy its armed forces in any state to deal with a grave situation of law and order. It facilitated the proclamation of a national emergency in a part of the territory of India. It extended the one-time duration of the President’s rule in a state from 6 months to one year. It added three new Directive Principles viz., equal justice and free legal aid, the participation of workers in the management of industries, and protection of the environment, forests, and wildlife. It provided that the laws made for the implementation of Directive Principles cannot be declared invalid by the courts on the grounds of violation of some Fundamental Rights. Hence option d is correct. Incorrect Solution (d) The 42nd Amendment Act of 1976 made the following provisions: It added three new words – socialist, secular, and integrity in the Preamble. It provided for administrative tribunals and tribunals for other matters. It curtailed the power of judicial review and writ jurisdiction of the Supreme Court and high courts. It shifted five subjects from the state list to the concurrent list – education, forests, protection of wild animals and birds, weights and measures and administration of justice, constitution, and organisation of all courts except the Supreme Court and the high courts. It empowered the Parliament to decide from time to time the rights and privileges of its members and committees. It provided for the creation of the All-India Judicial Service. It shortened the procedure for disciplinary action by taking away the right of a civil servant to make representation at the second stage after the inquiry (i.e., on the penalty proposed). It empowered the centre to deploy its armed forces in any state to deal with a grave situation of law and order. It facilitated the proclamation of a national emergency in a part of the territory of India. It extended the one-time duration of the President’s rule in a state from 6 months to one year. It added three new Directive Principles viz., equal justice and free legal aid, the participation of workers in the management of industries, and protection of the environment, forests, and wildlife. It provided that the laws made for the implementation of Directive Principles cannot be declared invalid by the courts on the grounds of violation of some Fundamental Rights. Hence option d is correct. Question 23 of 35 23. Question Consider the following statements regarding the Cassini Spacecraft: It consisted of NASA’s Cassini orbiter, which was the first space probe to orbit Saturn. It consisted of the ESA’s Huygens probe, which landed on Titan, Jupiter’s largest moon. Choose the correct code: a) 1 only b) 2 only c) Both 1 and 2 d) Neither 1 nor 2 Correct Solution (a) Cassini-Huygens was a joint NASA/European Space Agency (ESA)/Italian Space Agency (ASI) space mission to Saturn. It was launched on October 15, 1997. The Cassini Spacecraft consisted of NASA’s Cassini orbiter, which was the first space probe to orbit Saturn. Hence statement 1 is correct. The Cassini orbiter weighed 2,125 kg (4,685 pounds) and was 6.7 metres (22 feet) long and 4 metres (13 feet) wide. The instruments on board Cassini included radar to map the cloud-covered surface of Titan and a magnetometer to study Saturn’s magnetic field. The disk-shaped Huygens probe was mounted on the side of Cassini. It weighed 349 kg (769 pounds), was 2.7 metres (8.9 feet) across, and carried six instruments designed to study the atmosphere and surface of Titan. It consisted of the ESA’s Huygens probe, which landed on Titan, Saturn’s largest moon. Hence statement 2 is incorrect. It orbited Saturn from 2004 to 2017, circling the planet 294 times and teaching us almost everything we know about Saturn. It measured the structure of Saturn’s atmosphere and rings, as well as how they interact with the planet’s moons. It also discovered six named moons and revealed Enceladus and Titan as promising locations to search for extraterrestrial life. Incorrect Solution (a) Cassini-Huygens was a joint NASA/European Space Agency (ESA)/Italian Space Agency (ASI) space mission to Saturn. It was launched on October 15, 1997. The Cassini Spacecraft consisted of NASA’s Cassini orbiter, which was the first space probe to orbit Saturn. Hence statement 1 is correct. The Cassini orbiter weighed 2,125 kg (4,685 pounds) and was 6.7 metres (22 feet) long and 4 metres (13 feet) wide. The instruments on board Cassini included radar to map the cloud-covered surface of Titan and a magnetometer to study Saturn’s magnetic field. The disk-shaped Huygens probe was mounted on the side of Cassini. It weighed 349 kg (769 pounds), was 2.7 metres (8.9 feet) across, and carried six instruments designed to study the atmosphere and surface of Titan. It consisted of the ESA’s Huygens probe, which landed on Titan, Saturn’s largest moon. Hence statement 2 is incorrect. It orbited Saturn from 2004 to 2017, circling the planet 294 times and teaching us almost everything we know about Saturn. It measured the structure of Saturn’s atmosphere and rings, as well as how they interact with the planet’s moons. It also discovered six named moons and revealed Enceladus and Titan as promising locations to search for extraterrestrial life. Question 24 of 35 24. Question Consider the following statements regarding the Baltic Sea: It is a semi-enclosed inland sea located in Northern Europe. It is connected to the North Sea’s German Bight via the White Sea Canal. It connects to the Atlantic Ocean through the Danish Straits. How many of the above statements are correct? a) Only one b) Only two c) All three d) None Correct Solution (b) The Baltic Sea is a semi-enclosed inland sea located in Northern Europe. Hence statement 1 is correct. It is an arm of the North Atlantic Ocean, extending northward from the latitude of southern Denmark almost to the Arctic Circle and separating the Scandinavian Peninsula from the rest of continental Europe. It has a coastline of approximately 8,000 km, shared by several countries, including Sweden, Poland, Lithuania, Latvia, Finland, Estonia, Germany, Denmark, and Russia. It is connected to the North Sea’s German Bight via the Kiel Canal. Hence statement 2 is incorrect. It is connected to the White Sea via the White Sea Canal. The Baltic Sea contains three major gulfs: the Gulf of Bothnia to the north, the Gulf of Finland to the east, and the Gulf of Riga slightly to the south. It connects to the Atlantic Ocean through the Danish Straits. Hence statement 3 is correct. It is often cited as the world’s largest brackish inland water body. Its water salinity levels are lower than that of the World Oceans due to the inflow of fresh water from the surrounding land and the sea’s shallowness. Incorrect Solution (b) The Baltic Sea is a semi-enclosed inland sea located in Northern Europe. Hence statement 1 is correct. It is an arm of the North Atlantic Ocean, extending northward from the latitude of southern Denmark almost to the Arctic Circle and separating the Scandinavian Peninsula from the rest of continental Europe. It has a coastline of approximately 8,000 km, shared by several countries, including Sweden, Poland, Lithuania, Latvia, Finland, Estonia, Germany, Denmark, and Russia. It is connected to the North Sea’s German Bight via the Kiel Canal. Hence statement 2 is incorrect. It is connected to the White Sea via the White Sea Canal. The Baltic Sea contains three major gulfs: the Gulf of Bothnia to the north, the Gulf of Finland to the east, and the Gulf of Riga slightly to the south. It connects to the Atlantic Ocean through the Danish Straits. Hence statement 3 is correct. It is often cited as the world’s largest brackish inland water body. Its water salinity levels are lower than that of the World Oceans due to the inflow of fresh water from the surrounding land and the sea’s shallowness. Question 25 of 35 25. Question Consider the following statements regarding Micellar Water: It contains micelles which are formed by special molecules known as surfactants. It contains clusters of molecules that are very effective at removing oily substances. It is used as a skincare product that helps remove impurities and makeup from your skin. How many of the above statements are correct? a) Only one b) Only two c) All three d) None Correct Solution (c) Micellar Water contains micelles which are formed by special molecules known as surfactants. Hence statement 1 is correct. The compounds that lower the surface tension between two phases, like between two liquids, a gas and a liquid, or a liquid and a solid are called surface active agents or surfactants. These micelles have a hydrophilic exterior, meaning that they are very happy to remain mixed throughout the water. It contains clusters of molecules that are very effective at removing oily substances. Hence statement 2 is correct. When you apply some micellar water to a cotton pad, another convenient interaction occurs. Wet cotton is hydrophilic (loves water). Consequently, some of the micelles will unravel, with the hydrophilic heads being attracted to the wet cotton pad. Now, sticking out from the surface will be a layer of hydrophobic tail groups. These hydrophobic tails cannot wait to attract themselves to makeup, sunscreen, oils, dirt, grease, and other contaminants on your face. As you sweep the cotton pad across your skin, these contaminants bind to the hydrophobic tails and are removed from the skin. It is used as a skincare product that helps remove impurities and makeup from your skin. Hence statement 3 is correct.   Incorrect Solution (c) Micellar Water contains micelles which are formed by special molecules known as surfactants. Hence statement 1 is correct. The compounds that lower the surface tension between two phases, like between two liquids, a gas and a liquid, or a liquid and a solid are called surface active agents or surfactants. These micelles have a hydrophilic exterior, meaning that they are very happy to remain mixed throughout the water. It contains clusters of molecules that are very effective at removing oily substances. Hence statement 2 is correct. When you apply some micellar water to a cotton pad, another convenient interaction occurs. Wet cotton is hydrophilic (loves water). Consequently, some of the micelles will unravel, with the hydrophilic heads being attracted to the wet cotton pad. Now, sticking out from the surface will be a layer of hydrophobic tail groups. These hydrophobic tails cannot wait to attract themselves to makeup, sunscreen, oils, dirt, grease, and other contaminants on your face. As you sweep the cotton pad across your skin, these contaminants bind to the hydrophobic tails and are removed from the skin. It is used as a skincare product that helps remove impurities and makeup from your skin. Hence statement 3 is correct.   Question 26 of 35 26. Question Consider the following statements regarding the National Generic Document Registration System: It is a common, generic, and configurable application developed for registration departments across the country. It is initiated by the Department of Land Resources, Ministry of Home Affairs. It facilitates states to create state-specific instances and configure the software as per requirements. How many of the above statements are correct? a) Only one b) Only two c) All three d) None Correct Solution (b) The National Generic Document Registration System (NGDRS) is a common, generic, and configurable application developed for registration departments across the country. Hence statement 1 is correct. The NGDRS application is developed by the National Informatics Centre in Pune. It is initiated by the Department of Land Resources, Ministry of Rural Development. Hence statement 2 is incorrect. The application is specifically designed for the use of sub-registrars, citizens, and apex users from registration departments. It offers a complete user interface for property and document registration, the application enables citizens to proceed with land buying online. They can find out the circle rate for land, calculate property valuation as per prevailing rates, and understand the type of land. Transactions of prohibited properties are restricted for sale like government land, tribal land, mortgaged land, etc., and are also available, which ultimately helps them decide where and what type of land they should buy. Thereafter, they can apply online for document submission, make instant payments, and take prior appointments. It facilitates states to create state-specific instances and configure the software as per requirements. Hence statement 3 is correct. Incorrect Solution (b) The National Generic Document Registration System (NGDRS) is a common, generic, and configurable application developed for registration departments across the country. Hence statement 1 is correct. The NGDRS application is developed by the National Informatics Centre in Pune. It is initiated by the Department of Land Resources, Ministry of Rural Development. Hence statement 2 is incorrect. The application is specifically designed for the use of sub-registrars, citizens, and apex users from registration departments. It offers a complete user interface for property and document registration, the application enables citizens to proceed with land buying online. They can find out the circle rate for land, calculate property valuation as per prevailing rates, and understand the type of land. Transactions of prohibited properties are restricted for sale like government land, tribal land, mortgaged land, etc., and are also available, which ultimately helps them decide where and what type of land they should buy. Thereafter, they can apply online for document submission, make instant payments, and take prior appointments. It facilitates states to create state-specific instances and configure the software as per requirements. Hence statement 3 is correct. Question 27 of 35 27. Question Consider the following statements regarding ‘Brumation’: It describes a period of dormancy or slowed activity in reptiles. It typically occurs during warmer months, when temperatures rise. It allows reptiles to conserve energy and minimise their resource requirements. How many of the above statements are correct? a) Only one b) Only two c) All three d) None Correct Solution (b) Brumation describes a period of dormancy or slowed activity in reptiles. Hence statement 1 is correct. It is much like hibernation in mammals. Brumation is crucial for reptiles to survive cold climates and endure challenging environmental conditions until they can re-emerge to feed and reproduce in more favourable climes. It typically occurs during colder months, when temperatures drop. Hence statement 2 is incorrect. During brumation, reptiles may retreat to underground burrows, rock crevices, or other sheltered areas where temperatures are relatively more stable. Their metabolism slows significantly, allowing them to go weeks or even months without eating. It allows reptiles to conserve energy and minimise their resource requirements. Hence statement 3 is correct. Incorrect Solution (b) Brumation describes a period of dormancy or slowed activity in reptiles. Hence statement 1 is correct. It is much like hibernation in mammals. Brumation is crucial for reptiles to survive cold climates and endure challenging environmental conditions until they can re-emerge to feed and reproduce in more favourable climes. It typically occurs during colder months, when temperatures drop. Hence statement 2 is incorrect. During brumation, reptiles may retreat to underground burrows, rock crevices, or other sheltered areas where temperatures are relatively more stable. Their metabolism slows significantly, allowing them to go weeks or even months without eating. It allows reptiles to conserve energy and minimise their resource requirements. Hence statement 3 is correct. Question 28 of 35 28. Question Consider the following statements: A bond is a fixed-income instrument that represents a loan made by an investor to a borrower for a set period of time in return for regular interest payments. The time from when the bond is issued to when the borrower has agreed to pay the loan back is called its ‘term to maturity’. Bond Yield is the return an investor expects to receive each year over its term to maturity. As the price of a bond goes up, its yield goes down, and as yield goes up, the price of the bond goes down. Choose the correct code: a) Only one b) Only two c) Only three d) All four Correct Solution (d) A bond is a fixed-income instrument that represents a loan made by an investor to a borrower for a set period of time in return for regular interest payments. Hence statement 1 is correct. The time from when the bond is issued to when the borrower has agreed to pay the loan back is called its ‘term to maturity’. Hence statement 2 is correct. The bond issuer uses the money raised from bonds to undertake various activities, such as funding expansion projects, refinancing existing debt, undertaking welfare activities, etc. Bond Yield is the return an investor expects to receive each year over its term to maturity. Hence statement 3 is correct. It partially depends on coupon payments, which refer to the periodic interest income obtained as a reward for holding bonds. The bondholders receive the bond’s face value at the end of the bond’s life. However, one may buy bonds at par value, discount (at a price lower than par value), or premium (at a price higher than par value) as they trade in the secondary market. Therefore, the prevailing market price of bonds also affects the bond yield. As the price of a bond goes up, its yield goes down, and as yield goes up, the price of the bond goes down. Hence statement 4 is correct. Incorrect Solution (d) A bond is a fixed-income instrument that represents a loan made by an investor to a borrower for a set period of time in return for regular interest payments. Hence statement 1 is correct. The time from when the bond is issued to when the borrower has agreed to pay the loan back is called its ‘term to maturity’. Hence statement 2 is correct. The bond issuer uses the money raised from bonds to undertake various activities, such as funding expansion projects, refinancing existing debt, undertaking welfare activities, etc. Bond Yield is the return an investor expects to receive each year over its term to maturity. Hence statement 3 is correct. It partially depends on coupon payments, which refer to the periodic interest income obtained as a reward for holding bonds. The bondholders receive the bond’s face value at the end of the bond’s life. However, one may buy bonds at par value, discount (at a price lower than par value), or premium (at a price higher than par value) as they trade in the secondary market. Therefore, the prevailing market price of bonds also affects the bond yield. As the price of a bond goes up, its yield goes down, and as yield goes up, the price of the bond goes down. Hence statement 4 is correct. Question 29 of 35 29. Question Consider the following statements regarding ‘Hastsal Minar’: Its construction was finished during the reign of Mughal emperor Akbar. It was constructed using lakhori bricks and clad with red sandstone. Choose the correct code: a) 1 only b) 2 only c) Both 1 and 2 d) Neither 1 nor 2 Correct Solution (b) Hastsal Minar construction was finished during the reign of Mughal emperor Shah Jahan. Hence statement 1 is incorrect. It is located in a small village in western Delhi. It is also known as Mini Qutab Minar. Among the locals, it is also known as Hastsal Ki Laat and Kaushal Minar. It was constructed using lakhori bricks and clad with red sandstone. Hence statement 2 is correct. The minar is 17 metres (five storeys) tall, standing on a square platform, and with an octagonal body. A staircase inside the five-storey tower led to the domed Chhatri pavilion at the top. It has three storeys, each with a reducing diameter, and is accessible through a narrow staircase Shah Jahan utilised Hastsal as one of his hunting lodges during the 17th century. Incorrect Solution (b) Hastsal Minar construction was finished during the reign of Mughal emperor Shah Jahan. Hence statement 1 is incorrect. It is located in a small village in western Delhi. It is also known as Mini Qutab Minar. Among the locals, it is also known as Hastsal Ki Laat and Kaushal Minar. It was constructed using lakhori bricks and clad with red sandstone. Hence statement 2 is correct. The minar is 17 metres (five storeys) tall, standing on a square platform, and with an octagonal body. A staircase inside the five-storey tower led to the domed Chhatri pavilion at the top. It has three storeys, each with a reducing diameter, and is accessible through a narrow staircase Shah Jahan utilised Hastsal as one of his hunting lodges during the 17th century. Question 30 of 35 30. Question It is the smallest tiger reserve in India by area located in the Wardha District of Maharashtra. It is a wildlife sanctuary which was declared a tiger reserve in July 2014. The above paragraph is related to which tiger reserve? a) Bor Tiger Reserve b) Satpura Tiger Reserve c) Pench Tiger Reserve d) Melghat Tiger Reserve Correct Solution (a) Bor Tiger Reserve is the smallest tiger reserve in India by area located in the Wardha District of Maharashtra. It is a wildlife sanctuary which was declared a tiger reserve in July 2014. It is centrally located among several other Bengal Tiger habitats. Towards the northeast lies the Pench Tiger Reserve, towards the east is Nagzira Navegaon Tiger Reserve, and to the southeast is Karhandla Wildlife Sanctuary. The Tadoba Andhari Tiger Reserve is located to the southeast, the Melghat Tiger Reserve stands to the west, and the Satpura Tiger Reserve lies to the northwest. The area is populated by dry deciduous forest types. Hence option a is correct. Incorrect Solution (a) Bor Tiger Reserve is the smallest tiger reserve in India by area located in the Wardha District of Maharashtra. It is a wildlife sanctuary which was declared a tiger reserve in July 2014. It is centrally located among several other Bengal Tiger habitats. Towards the northeast lies the Pench Tiger Reserve, towards the east is Nagzira Navegaon Tiger Reserve, and to the southeast is Karhandla Wildlife Sanctuary. The Tadoba Andhari Tiger Reserve is located to the southeast, the Melghat Tiger Reserve stands to the west, and the Satpura Tiger Reserve lies to the northwest. The area is populated by dry deciduous forest types. Hence option a is correct. Question 31 of 35 31. Question A simple mathematical operation with respect to the difference between cube and square in each number results in a sequence 4, 18, 48, 100, 180, 294, ….. . Which one of the following is the next number in the sequence? a) 512 b) 500 c) 448 d) 440 Correct Solution (c) As per the given condition in question: 23 – 22 = 8 – 4 = 4 33 – 32 = 27 – 9 = 18 43 – 42 = 64 – 16 = 48 53 – 52 = 125 – 25 = 100 63 – 62 = 216 – 36 = 180 73 – 72 = 343 – 49 = 294 83 – 82 = 512 – 64 = 448 Next number is 448 Hence, option (c) is the correct answer. Incorrect Solution (c) As per the given condition in question: 23 – 22 = 8 – 4 = 4 33 – 32 = 27 – 9 = 18 43 – 42 = 64 – 16 = 48 53 – 52 = 125 – 25 = 100 63 – 62 = 216 – 36 = 180 73 – 72 = 343 – 49 = 294 83 – 82 = 512 – 64 = 448 Next number is 448 Hence, option (c) is the correct answer. Question 32 of 35 32. Question A number n > 7 is divisible by 7 but not divisible by 14. Which one of the following is divisible by 11? a) 2n b) 3n c) 2n + 1 d) 3n + 3 Correct Solution (d) Given that n is a number greater than 7, and is divisible by 7 but not divisible by 14. Let n = 21 Now checking with options, (a) 2n = 2 × 21 = 42, which is not divisible by 11. (b) 3n = 3 × 21 = 63, which is not divisible by 11. (c) 2n + 1 = 2 × 21 + 1 = 43, which is not divisible by 11. (d) 3n + 3 = 3 × 21 + 3 = 66, which is divisible by 11. Hence, the correct answer is option (d). Incorrect Solution (d) Given that n is a number greater than 7, and is divisible by 7 but not divisible by 14. Let n = 21 Now checking with options, (a) 2n = 2 × 21 = 42, which is not divisible by 11. (b) 3n = 3 × 21 = 63, which is not divisible by 11. (c) 2n + 1 = 2 × 21 + 1 = 43, which is not divisible by 11. (d) 3n + 3 = 3 × 21 + 3 = 66, which is divisible by 11. Hence, the correct answer is option (d). Question 33 of 35 33. Question Consider the following information: P is a perfect number and numbers a, b, c and d are all the possible factors of number P. Sum of a, b, c and d is equal to p Sum of a, b, c and d is equal to 2p Which of the above is/are true? a) 1 only b) 2 only c) 1 and 2 only d) None of the above Correct Solution (b) A number for which sum of all its factors is equal to twice the number is called a perfect number. Incorrect Solution (b) A number for which sum of all its factors is equal to twice the number is called a perfect number. Question 34 of 35 34. Question Madhavi’s salary is 75% more than Kamal’s. Madhavi got a raise of 40% on her salary while Kamal got a raise of 25% on her salary. By what percent is Madhavi’s salary more than Kamal’s? a) 96% b) 93% c) 55.6% d) 52.1% Correct Solution (a) Let Kamal’s salary = 100; Kamal’s salary after rise = 125 Then Madhavi’s salary = 175 Madhavi’s salary after rise of 40% = 245 [As 10% of Madhavi’s salary is 17.5 then 40% = 17.5 × 4 = 70] Difference between Madhavi’s salary and Kamal’s salary = 245 – 125 = 120 % more Madhavi’s salary than Kamal’s = ((120*100)/125) = 96% Incorrect Solution (a) Let Kamal’s salary = 100; Kamal’s salary after rise = 125 Then Madhavi’s salary = 175 Madhavi’s salary after rise of 40% = 245 [As 10% of Madhavi’s salary is 17.5 then 40% = 17.5 × 4 = 70] Difference between Madhavi’s salary and Kamal’s salary = 245 – 125 = 120 % more Madhavi’s salary than Kamal’s = ((120*100)/125) = 96% Question 35 of 35 35. Question The average temperature for Wednesday, Thursday and Friday was 400C. The average for Thursday, Friday and Saturday was 41oC. If temperature on Saturday was 42oC, what was the temperature on Wednesday? a) 39o C b) 44o C c) 38o C d) 41o C Correct Solution (a) Average temperature for Wednesday, Thursday and Friday = 40° C Total temperature = 3 × 40 = 120° C Average temperature for Thursday, Friday and Saturday = 41° C Total temperature = 41 × 3 = 123° C Temperature on Saturday = 42° C Now, (Thursday + Friday + Saturday) – (Wednesday + Thursday + Friday) = 123 – 120; Saturday – Wednesday = 3 Wednesday = 42 – 3 = 39° C   Incorrect Solution (a) Average temperature for Wednesday, Thursday and Friday = 40° C Total temperature = 3 × 40 = 120° C Average temperature for Thursday, Friday and Saturday = 41° C Total temperature = 41 × 3 = 123° C Temperature on Saturday = 42° C Now, (Thursday + Friday + Saturday) – (Wednesday + Thursday + Friday) = 123 – 120; Saturday – Wednesday = 3 Wednesday = 42 – 3 = 39° C   window.wpProQuizInitList = window.wpProQuizInitList || []; window.wpProQuizInitList.push({ id: '#wpProQuiz_3667', init: { quizId: 3667, mode: 1, globalPoints: 70, timelimit: 1800, resultsGrade: [0], bo: 704, qpp: 0, catPoints: [70], formPos: 0, lbn: "Test-summary", json: {"32789":{"type":"single","id":32789,"catId":0,"points":2,"correct":[0,1,0,0]},"32792":{"type":"single","id":32792,"catId":0,"points":2,"correct":[1,0,0,0]},"32793":{"type":"single","id":32793,"catId":0,"points":2,"correct":[0,0,0,1]},"32795":{"type":"single","id":32795,"catId":0,"points":2,"correct":[0,1,0,0]},"32796":{"type":"single","id":32796,"catId":0,"points":2,"correct":[0,0,0,1]},"32798":{"type":"single","id":32798,"catId":0,"points":2,"correct":[1,0,0,0]},"32799":{"type":"single","id":32799,"catId":0,"points":2,"correct":[1,0,0,0]},"32802":{"type":"single","id":32802,"catId":0,"points":2,"correct":[0,0,1,0]},"32803":{"type":"single","id":32803,"catId":0,"points":2,"correct":[0,0,0,1]},"32805":{"type":"single","id":32805,"catId":0,"points":2,"correct":[1,0,0,0]},"32807":{"type":"single","id":32807,"catId":0,"points":2,"correct":[0,0,1,0]},"32808":{"type":"single","id":32808,"catId":0,"points":2,"correct":[0,0,1,0]},"32810":{"type":"single","id":32810,"catId":0,"points":2,"correct":[1,0,0,0]},"32811":{"type":"single","id":32811,"catId":0,"points":2,"correct":[1,0,0,0]},"32812":{"type":"single","id":32812,"catId":0,"points":2,"correct":[0,1,0,0]},"32813":{"type":"single","id":32813,"catId":0,"points":2,"correct":[0,1,0,0]},"32814":{"type":"single","id":32814,"catId":0,"points":2,"correct":[0,0,1,0]},"32815":{"type":"single","id":32815,"catId":0,"points":2,"correct":[0,0,0,1]},"32816":{"type":"single","id":32816,"catId":0,"points":2,"correct":[0,0,1,0]},"32818":{"type":"single","id":32818,"catId":0,"points":2,"correct":[0,0,1,0]},"32821":{"type":"single","id":32821,"catId":0,"points":2,"correct":[0,1,0,0]},"32824":{"type":"single","id":32824,"catId":0,"points":2,"correct":[0,0,0,1]},"32827":{"type":"single","id":32827,"catId":0,"points":2,"correct":[1,0,0,0]},"32830":{"type":"single","id":32830,"catId":0,"points":2,"correct":[0,1,0,0]},"32832":{"type":"single","id":32832,"catId":0,"points":2,"correct":[0,0,1,0]},"32834":{"type":"single","id":32834,"catId":0,"points":2,"correct":[0,1,0,0]},"32836":{"type":"single","id":32836,"catId":0,"points":2,"correct":[0,1,0,0]},"32838":{"type":"single","id":32838,"catId":0,"points":2,"correct":[0,0,0,1]},"32841":{"type":"single","id":32841,"catId":0,"points":2,"correct":[0,1,0,0]},"32843":{"type":"single","id":32843,"catId":0,"points":2,"correct":[1,0,0,0]},"32845":{"type":"single","id":32845,"catId":0,"points":2,"correct":[0,0,1,0]},"32846":{"type":"single","id":32846,"catId":0,"points":2,"correct":[0,0,0,1]},"32847":{"type":"single","id":32847,"catId":0,"points":2,"correct":[0,1,0,0]},"32850":{"type":"single","id":32850,"catId":0,"points":2,"correct":[1,0,0,0]},"32851":{"type":"single","id":32851,"catId":0,"points":2,"correct":[1,0,0,0]}} } }); All the Best IASbaba